Você está na página 1de 144

Autores

Equipe Programa Cientista-Chefe em Educação Básica

UFC/FUNCAP/SEDUC
Todos os direitos reservados à
Secretaria da Educação do Estado do Ceará – Centro Administrativo Virgílio Távora
Av. General Afonso Albuquerque Lima, s/n – Cambeba. Fortaleza/CE – CEP: 60.822-325

GOVERNADOR
Elmano de Freitas da Costa
VICE-GOVERNADORA
Jade Afonso Romero

Secretária da Educação Eliana Nunes Estrela

Secretária Executiva de Ensino Médio e Maria Jucineide da Costa Fernandes


Profissional

Coordenadora de Educação em Tempo Gezenira Rodrigues da Silva


Integral
Coordenadora de Gestão Pedagógica do Ideigiane Terceiro Nobre
Ensino Médio

Coordenadora de Avaliação e Kelem Carla Santos de Freitas


Desenvolvimento Escolar para Resultados
de Aprendizagem

Coordenadora Estadual da Formação Vagna Brito de Lima


Docente e Educação a Distância

Coordenador da Educação Profissional Rodolfo Sena da Penha

Cientista-Chefe da Educação Jorge Herbert Soares de Lira

Elaboração e revisão de texto Jorge Herbert Soares de Lira


Roberta Eliane Gadelha Aleixo (revisora)
Annelise Maymone (revisora)

0
1
|Apresentação
Cara/o professora/or, apresentamos a seguir o Material Didático Estruturado (MDE) da Matemática
“Foco na Aprendizagem”, que consiste no desenvolvimento de ações integradas voltadas à recomposição
das aprendizagens, implementação do Documento Curricular Referencial do Ceará (DCRC) do Novo
Ensino Médio (NEM) e do modelo estadual de Educação Híbrida, articulando ações didático-pedagógicas
por meio da avaliação diagnóstica-formativa, Tutoria em Língua Portuguesa e Matemática, e formação
continuada de professores.
Nesse sentido, o intuito da Coordenadoria de Gestão Pedagógica do Ensino Médio (Cogem) é de
subsidiar suporte pedagógico para as atividades de recomposição das aprendizagens da iniciativa Foco
na Aprendizagem, com sugestões para o uso do MDE como mais um material de apoio, dentre os já
existentes de seu acervo de recursos didáticos nas aulas das áreas de conhecimento e componentes
curriculares de sua atuação, respeitando a autonomia didática de cada docente e as especificidades de
cada escola.
De modo geral, cada MDE possui suas especificidades, sendo os de Língua Portuguesa e Matemática
estruturados por componentes curriculares, e divididos em unidades temáticas voltadas ao desenvolvi-
mento de um objeto de aprendizagem; e os de Ciências Humanas e Ciências da Natureza, estruturados
por áreas do conhecimento, divididos em componentes curriculares e subdivididos em capítulos temáticos
voltados ao desenvolvimento de um objeto de aprendizagem e em diálogo com elementos do tempo
presente das/os estudantes. Nesse sentido, há de se enfatizar a finalidade que atravessa toda a estrutura
deste material: possuir uma estrutura padrão em que cada uma das seções didáticas está destinada a
favorecer diferentes estratégias de recomposição da aprendizagem.

Coordenadoria da Gestão do Ensino Médio - COGEM


Coordenadoria Estadual de Formação Docente e Ensino a Distância - CODED
Coordenadoria de Avaliação e Desenvolvimento para Resultados de Aprendizagem - COADE
Programa Cientista-Chefe em Educação - FUNCAP/UFC/SEDUC

Proposta Pedagógica

Cara/o estudante,
Este caderno foi pensado com muito critério para que se tornasse um material que possa apoiá-lo em
sua formação no Ensino Médio. Esperamos que você possa estudar cada página com o mesmo carinho e
dedicação com que ela foi escrita. Os elaboradores e toda a equipe pedagógica envolvida levaram em
conta os desafios que os últimos anos representaram nas vidas pessoais e escolares de cada um de vocês.
O Ensino Médio é uma etapa desafiadora não só por que somos expostos a conhecimentos mais
amplos e por que somos demandados em habilidades mais complexas. Esses desafios cognitivos vêm
junto com a transição para a vida adulta, com todas as incertezas e oportunidades que estão à frente. E,
no caso desta geração, vivenciamos, ainda, os efeitos da Pandemia e toda a repercussão que o necessário
isolamento social teve sobre a aprendizagem.
Por isso, esse caderno cuida da necessidade de recompor conhecimentos em Matemática que serão
indispensáveis no seu futuro, seja acadêmico, seja profissional. A Matemática não é um punhado de
fórmulas, um amontado de cálculos e uma disciplina “difícil”, que causa ansiedade e, depois, nada tem a
ver com nosso cotidiano. Muito ao contrário: as profissões mais demandadas exigem, cada vez mais,
habilidades relacionadas ao pensamento matemático. Além disso, não podemos exercer plenamente nossa
cidadania sem entendermos a linguagem matemática: basta ver como decisões econômicas, baseadas em
Matemática, afetam o orçamento de nossas famílias. Estamos imersos na sociedade da informação, em
que o uso de dados e de algoritmos determina aspectos de nossas vidas que sequer imaginamos.

3
Portanto, queremos nos juntar a você no esforço de superar suas eventuais dificuldades nos conceitos
essenciais da Matemática. Esse é um processo gradual, em que vamos avançando numa espiral pouco a
pouco: o que importa é termos clareza de quais são as lacunas que devem ser preenchidas e as metas
que devemos atingir. Dito isso, vamos explicar como o uso do caderno pode ajudar nesses percursos
formativos que iniciam com tarefas que retomam conhecimentos básicos e, progressivamente, avançam
para exercícios e problemas mais complexos. Nesses caminhos, você é convidado a trabalhar inclusive
com questões de vestibulares, do ENEM e das avaliações externas, como o SPAECE. O caderno é
organizado em quatro percursos. Vejamos do que trata cada um deles.

Primeiro percurso

Neste percurso, trabalharemos a localização de números racionais na reta numérica. Para isso,
precisamos saber quando duas frações são equivalentes e como representar um número racional como
fração ou como número decimal. Em seguida, estudaremos relações de proporcionalidade entre duas
variáveis. Veremos como representar essas relações usando retas no plano cartesiano. Essas retas são
gráficos de funções afins. Finalizamos o percurso, mostrando as relações de proporcionalidade entre
lados e perímetros de triângulos e quadriláteros semelhantes.

Segundo percurso

Agora, retomamos o estudo das funções afins, identificando os gráficos (linhas retas) que as representam
e interpretando geometricamente os coeficientes de suas expressões algébricas. Veremos, em seguida,
como relacionar funções afins e equações lineares. Finalizamos estudando funções quadráticas
resolvendo problemas a respeito dos seus zeros e valores máximos/mínimos.

Terceiro percurso

Retornamos ao estudo da proporcionalidade, explorando, desta vez, problemas que envolvem razões,
proporções e porcentagens, em diferentes contextos, inclusive relacionados à Matemática Financeira.
Na sequência, estudaremos o comportamento de funções (crescimento, decrescimento, zeros) analisando
seus gráficos. Finalizamos o percurso com problemas envolvendo funções exponenciais e crescimento
geométrico.

Quarto percurso

O caderno finaliza com exercícios e problemas sobre tratamento da informação, ou seja, a análise de
tabelas, de gráficos de barras, de linhas ou de setores. Além disso, trabalharemos questões envolvendo
probabilidade em vários contextos.

Dicas sobre o uso do caderno

Os percursos são independentes, mas recomendamos que você siga a sequência do primeiro ao quarto.
Em cada um, siga a ordem dos exercícios, pois esses aumentam gradualmente em complexidade. Caso
sinta dificuldades em alguns deles, registre-as e converse com seu professor a respeito. Não se sinta
desencorajado se não conseguir resolver uma ou outra questão: o mais importante é comunicar suas
dificuldades para que o professor possa ajudá-lo com mais precisão. Além de recorrer ao professor,
recomendamos que consulte outros materiais de apoio. Mencionamos, por exemplo, os seguintes recursos:
• os cadernos do material estruturado do Foco na Aprendizagem: https://www.ced.seduc.ce.gov.br/
foco-na-aprendizagem-matematica-2/
• Os módulos do Portal da Matemática da OBMEP: https://portaldaobmep.impa.br/index.php/site/
index?a=1
• Khan Academy: https://pt.khanacademy.org/math/em-mat-algebra

4
Seção 1. Primeiro percurso: coordenadas, semelhança, perímetros e áreas

As tarefas a seguir envolvem conhecimentos prévios fundamentais para desenvolver as habilidades nos
seguintes descritores da Matriz de Referência do SAEB (terceira série do Ensino Médio):

• D14 - Identificar a localização de números na reta numérica.


• D15 - Resolver problema que envolva variações proporcionais, diretas ou inversas, entre grandezas.
• D6 - Identificar a localização de pontos no plano cartesiano.
• D7 - Interpretar geometricamente os coeficientes da equação de uma reta.
• D1 - Identificar figuras semelhantes mediante o reconhecimento de relações de proporcionalidade.
• D11 - Resolver problema envolvendo o cálculo de perímetro de figuras planas.
• D12 - Resolver problema envolvendo o cálculo da área de figuras planas.

Questão 1 Observe o segmento da reta numérica representado na seguinte figura.

A B C D E
0 P Q 1 R S T 2

Agora, faça as seguintes atividades.

i) Determine os pontos que correspondem aos números


1 2 2 4 14 9
0,6 0,9 1,2
10 10 5 5 10 5
ii) Determine as frações que correspondem aos pontos A, B, C, D e E.
iii) Determine os números decimais que correspondem aos pontos P, Q, R, S e T .
iv) Determine a localização (mesmo que aproximada) dos números racionais
1 1 2 4 8 4 8 9
0,25 0,625 1,25 1,75
6 3 3 6 9 3 6 6

Solução. i) A reta numérica está graduada em intervalos cujo comprimento é igual a 1/10 da
unidade de medida. Sendo assim, temos
2 0,6 4 0,9 1,2 1,4 9
5 5 5

0 1 2 4 6 8 9 1 12 14 18 2
10 10 10 10 10 10 10 10 10

ii) Na reta numérica seguinte, indicamos os números racionais correspondentes aos pontos A, B, C, D e
E:
A B C D E
0 3 4 7 1 15 17 2
10 10 10 10 10

iii) Na reta numérica seguinte, indicamos os números racionais correspondentes aos pontos P, Q, R, S e
T . Para isso, percebemos que esses são pontos médios dos intervalos; logo, são múltiplos de 1/20. Temos
5 15 25 35 39
20 20 20 20 20

0 P Q 1 R S T 2

5
iv) Quase todos os pontos na lista são múltiplos ou submúltiplos de 13 . Portanto, o primeiro passo deve
ser determinar a localização, exata ou aproximada, do número racional 31 . Para isso note que

1 3 3
= >
3 9 10
e
1 4 4
= < ·
3 12 10
1 3 4
Com isso, mostramos que 3 está entre 10 e 10 . Logo,
1
3

0 3 4 1 2
10 10

Agora, podemos determinar outros pontos da lista. Por exemplo, temos


2 1 4 4:2 2
=2· e = =
3 3 6 6:2 3
e, da mesma forma,
4 1 8 8:2 4
=4· e = = ·
3 3 6 6:2 3
Sendo assim, marcamos esses pontos na reta numérica como múltiplos de 13 :
1 2 4 5
3 3 3 3

0 3 4 4 1 8 2
10 10 6 6

1
Agora, é preciso localizar a fração 6 na reta numérica. Para isso, podemos considerar que

1 1 1
= · ,
6 2 3
1
ou seja, observar que 6 corresponde ao ponto médio entre 0 e 13 . Sendo assim, temos:

0 1 1 2 1 4 5 2
6 3 3 3 3

Note que
9 1
=9· ·
6 6
Assim,
4 8 9 10 5
= < < = .
3 6 6 6 3
9 8 4 10
De fato, 6 é o ponto médio entre 6 = 3 e 6 = 35 . Assim, obtemos:

0 1 1 2 1 4 9 5 2
6 3 3 3 6 3

1
O próximo passo é localizar a fração 9 na reta numérica. Para isso, devemos ver que

1 1 1
= ·
9 3 3
o que é representado na reta numérica como segue:

6
1 2 4 5
3 3 3 3

0 1 2 3 6 8 1 12 15 2
9 9 9 9 9 9 9

Note que, na reta numérica, temos


2 6 8 9
= < < = 1.
3 9 9 9
Finalmente, observamos que
2 3
1 0,2 + 0,3 10 + 10
0,25 = · 0,5 = = ,
2 2 2
2 3
o que significa que 0,25 é o ponto médio entre 0.2 = 10 e 0,3 = 10 . Além disso, temos

1,25 = 1 + 0,25 e 1,75 = 2 − 0,25.

Escrevendo esses números nessas formas, podemos localizá-los na reta numérica como segue:
0,2 0,3 0,6 0,7 1,2 1,3 1,7 1,8

0 0,25 0,625 1 1,25 1,75 2

Concluímos, observando que


2 3
0,02 + 0,03 100 + 100
0,625 = 0,6 + 0,025 = 0,6 + = 0,6 + .
2 2


Questão 2 Observe o segmento da reta numérica representado na seguinte figura.

A B CD E
0 1 2

Assinale a alternativa em que o ponto corresponde corretamente ao número racional.

7 B) B = 0,75 5 D) D = 0,57 E) E = 0,075


A) A = 5 C) C = 7

Solução. A reta numérica no suporte da questão está graduada em intervalos com medida igual a
1/10 da unidade. Sendo assim,
14
E= = 1,4.
10
Já o ponto D é o ponto médio entre 7/10 e 8/10, ou seja,
7 8
10 + 10 15
D= = ·
2 20
Observe, portanto, que
3
D= = 0,75.
4
Logo, as alternativas D) e E) estão erradas.
5 6
Quanto ao ponto B, a figura indica que está entre 10 = 0,5 e 10 = 0,6. Logo,

B < 0,6 < 0,75.

7
1
Portanto, a alternativa B) também não é correta. De modo similar, vemos que A está entre 0 e 10 = 0,1.
Sendo assim, concluímos que
7
A < 0,1 < 1 < ,
5
o que mostra que a alternativa A) não é verdadeira.
Por fim, o ponto C está entre 7/10 = 0,7 e D = 0,75. Por outro lado,
5 1 50 1 1 1 10 1 3
   
= · = · 7+ = 0,7 + · = 0,7 + · 1+ = 0,71 + . . .
7 10 7 10 7 100 7 100 7
Deduzimos que a alternativa C) é correta. 

Questão 3 — SABE - Item M120905E4, adaptado. Observe a reta numérica abaixo. Ela está dividida
em segmentos de mesma medida.

S T U V W X
0 2 1 4 4 5 5 1
7 3 9 7 8 6

3
O número está localizado entre os pontos
4
A) S e T . B) T e U . C) U e V . D) V e W . E) W e X.
1
Solução. O segmento da reta entre 0 e 1 está dividido em 10 intervalos de comprimento 10 . Logo,
3
= 0,75
4
7 8 1
está entre os números 0,7 = 10 e 0,8 = 10 , ou seja, está no oitavo intervalo de comprimento 10 , da
esquerda para a direita. Portanto, está entre os pontos W e X. Comparando as frações, vê-se que, de
fato,
5 6 3
< =
8 8 4
e
3 9 10 5
= < = ·
4 12 12 6
A alternativa correta é a E). 

3
Observação 0.1 Apenas para completar a discussão, observe na reta numérica 27 < 10 (de fato
20 = 2 · 10 < 3 · 7 = 21) e, portanto,
2 3 3
< <
7 10 4
Da mesma forma, temos
4 6 3 3
< = < ,
7 10 5 4
e assim por diante.

Exercício 0.1 — PAEBES - Item M090258G5, adaptado. Observe a reta numérica abaixo. Ela está
dividida em segmentos de mesma medida.

M L K J
0 0,4 0,8 1,2 1,6 2 2,4 2,8 3,2 3,6 4 4,4 4,8 5,2 5,6 6

8
5
Qual é o ponto que melhor representa a localização do número nessa reta?
4
A) M . B) L. C) K. D) J.
2
Solução. A reta numérica está dividida em intervalos de comprimento 0,2 = 10 . Observe que

5 1
= 1 + = 1,25.
4 4
Logo,
5
1,2 < < 1,4
4
e a alternativa correta é B). 

Questão 4 — SPAECE - Item M090307H6. Observe abaixo a reta numérica dividida em segmentos de
mesma medida.

X
−6 −4 −2 0 2

O número racional representado pelo ponto X é

A) −6,4. B) −5,5. C) −4,5. D) −4,6.


5
Solução. A reta numérica está dividida em intervalos de comprimento 0,5 = 10 = 12 . Observe que
X é, portanto, dado por
−4 − 0,5 = −4,5
ou, equivalentemente, por
−6 + 3 · 0,5 = −6 + 1,5 = −4,5
Logo, a alternativa correta é C). 

Questão 5 — SAEPE - Item M110764E4, adaptado. Caroline está completando a reta numérica
representada abaixo, na qual as distâncias entre dois pontos consecutivos são todas iguais.

−32 −24 R −8 0 8

Para completar essa reta numérica, qual número Caroline deve escrever no lugar da letra R?

A) −25. B) −23. C) −16. D) −9. E) −7.

Solução. A reta numérica está dividida em intervalos de comprimento 8. Observe que R é, portanto,
dado por
−8 − 8 = −16
ou, equivalentemente, por
−24 + 8 = −16.
Logo, a alternativa correta é C). 

9
Questão 6 Nas retas numéricas a seguir, quais frações correspondem aos pontos A, B, C, D, E, F ,
G, H, I, J, K e L?

0 A 1

0 1/4 B C 1

0 1/3 D 1

0 1/6 2/6 E F 1

0 1/12 G H 1

0 2/12 I J 1

0 3/12 K L 1

Solução. Observe que os pontos A, B, E e H coincidem com o ponto que fica à mesma distância
de 0 e de 1. Logo, esse ponto divide o segmento de reta de 0 a 1 em duas partes iguais e, portanto,
1 2 3 6
A= =B= =E= =H= ·
2 4 6 12
Agora, veja que o ponto G divide o segmento de reta de 0 a H em duas partes iguais. Além disso,
observe que o segmento de reta que contém G está dividido em 12 partes com mesma medida. Logo,
1 1 1 1 3 1
G = metade de = · = e G= = ·
2 2 2 4 12 4
Podemos também ver que os pontos D, F, J e K coincidem, ou seja D = F = J = K, ou seja,
2 4 8
= = ·
3 6 12
Por fim, verificamos que
3 9 4 2 1
C= =L= e I= = = ·
4 12 12 6 3


Questão 7 Usando as retas numéricas na questão 6, encontre o valor da incógnita x ou da incógnita


y em cada uma das seguintes equivalências de frações.

10
1 y 1 2 y 4 8 4
i) = ii) = iii) = iv) =
2 4 x 6 3 6 12 x
Solução. Temos
1 1·2 2 2 2:2 1 4 4:2 2 8 8:2 4
i) = = ii) = = iii) = = iv) = =
2 2·4 4 6 6:2 3 6 6:2 3 12 12 : 2 6


Questão 8 Complete as seguintes frações com numeradores ou denominadores de modo que as


igualdades sejam verdadeiras.
3 6 15 18
= = = = = = =
4 12 16 28 32

Solução. Temos
3 3·2 6 3 3·3 9
= = e = = ·
4 4·2 8 4 4·3 12
Da mesma forma, temos
3 3·4 12 3 3·5 15
= = e = = ·
4 4·4 16 4 4·5 20
Finalmente,
3 3·6 18 3 3·7 21 3 3·8 24
= = e = = e = = ·
4 4·6 24 4 4·7 28 4 4·8 32


Questão 9 Complete a seguinte tabela com valores das variáveis x e y, considerando que são
diretamente proporcionais.

Valores de y 3 6 - - 15 18 - -
Valores de x 4 - 12 16 - - 28 32

Solução. Com base nos resultados na questão 8, temos

Valores de y 3 6 9 12 15 18 21 24
Valores de x 4 8 12 16 20 24 28 32

Sejam m, n, p e q são números naturais, com n e q diferentes de 0. Sendo assim, as frações


m p
e
n q
são equivalentes, ou seja, a igualdade
m p
=
n q
é verdadeira se, e somente se,
q · m = p · n.
De fato, basta multiplicarmos cada um dos lados da igualdade pelo produto q · n, obtendo
m p
q·n· =q·n· ,
n q

11
igualdade que pode ser reescrita como
1 1
q·m·n· =n·p·q· ,
n q
o que nos permite concluir que
q · m = n · p.
3 9
Por exemplo, as frações e (com m = 3, n = 4, p = 9 e q = 12) são equivalentes, pois
4 12
3 9
12 · 4 · = 4 · 12 · ,
4 12
1 1
ou seja, multiplicando 4 · 4 e 12 · 12 , temos

12 · 3 = 4 · 9,

uma igualdade verdadeira, pois resulta em 36 = 36.

Observação 0.2 A equivalência de frações pode ser entendida como uma relação de proporciona-
lidade: dizemos que
m p
=
n q
quando m está para n assim como p está para q, situação em que vale a igualdade do produto dos
meios e do produto dos extremos, ou seja,

q · m = p · n,

uma vez que


m p
q·Z
n· = n · qA ·
n
Z qA

Usando a definição acima de equivalência de frações, vamos, agora, apresentar critérios práticos, que
são consequências lógicas da definição, para verificar se duas frações são equivalentes. Se multiplicarmos
ou dividirmos o numerador e o denominador de uma fração por um mesmo número a natural
diferente de zero, obtemos uma fração equivalente. De fato,
m m·a
= ,
n n·a
visto que
m · n · a = n · m · a.
Da mesma forma,
m m:a
= ·
n n:a
Neste caso, a deve ser um divisor ou fator comum de m e n com m : a = p e n : a = q. Assim, temos
m p·a p m:a
= = = ,
n q·a q n:a
3 6 3·2 6
como queríamos demonstrar. Por exemplo, as frações 4 e 8 são equivalentes, pois 4·2 = 8 e, da mesma

12
9 12
forma, as frações 12 e 16 são também equivalentes, pois

9:3 3 3·4 12
= = = ,
12 : 3 4 4·4 16
o que comprova a equivalência dessas frações.

Questão 10 Represente os valores das variáveis x e y na tabela da questão 9 como coordenadas de


pontos no seguinte plano cartesiano. Veja o exemplo do ponto P = (4, 3) em que x = 4 e y = 3.
y
27
24
21
18
15
12
9
6
P
3

0 4 8 12 16 20 24 28 32 x

Agora, responda às seguintes perguntas.

1) Os pontos devem estar alinhados, ou seja, devem pertencer a uma mesma reta r. Você conseguiu
perceber isso em seu desenho?
2) Qual o valor de y para que o ponto (8, y) pertença à reta r?
3) Qual o valor de x para que o ponto (x, 9) pertença à reta r?
4) Qual o valor de y para que o ponto (32, y) pertença à reta r?
5) Qual o valor de x para que o ponto (x, 27) pertença à reta r?
6) Por qual razão os pontos que você representou no plano devem estar alinhados, ou seja, devem
pertencer todos a uma mesma reta?
7) Qual relação existe entre as coordenadas x e y para que o ponto (x, y) pertença à reta r?
8) O ponto (9, 6), em que x = 9 e y = 6, pertence à reta r? Justifique sua resposta.
9) O ponto (10, 15/2), em que x = 10 e y = 15/2, pertence à reta r? Justifique sua resposta.
10) Qual o valor de x para que o ponto (x, 15/4) pertença à reta r?
11) Qual o valor de y para que o ponto (3, y) pertença à reta r?
12) Existe algum ponto (x, y) na reta r tal que x + y = 14?

Solução. 1) Ao considerarmos os valores de x e os valores correspondentes de y na tabela da


questão 8 como coordenadas cartesianas, obtemos os seguintes pontos no plano cartesiano:
P = (4, 3), A = (8, 6), B = (12, 9), C = (16, 12), D = (20, 15), E = (24, 18), F = (28, 21), Q = (32, 24).
Observamos que as primeiras coordenadas (absicssas) em cada par aumentam 4 unidades de um ponto
para o imediatamente seguinte, da esquerda para a direita; da mesma forma, as segundas coordenadas
(ordenadas) em cada par aumentam 4 unidades nessa sequência de pontos. Portanto, a variação da
coordenada x (abscissa) de um ponto para o imediatamente seguinte é dada por
∆x = 4,

13
ao passo que a variação da coordenada y (ordenada) entre dois pontos consecutivos é

∆y = 3.

Logo, a razão entre essas variações é igual a

∆y 3
= ,
∆x 4
ou seja,
3
∆y = ∆x. (1)
4
Em resumo, constatamos, para esses pontos, que
a variação da coordenada y entre esses dois pontos da reta r é igual a 3/4 da variação correspondente
da coordenada x
Representando os pontos P, A, B.C, D, E, F e Q no plano cartesiano, obtemos uma figura como a
seguinte:
y
27
Q
24 r
F
21
E
18
D
15
C
12
B
9
A
6
P
3

0 4 8 12 16 20 24 28 32 x

Os pontos assinalados pertencem, todos, à reta r traçada na figura anterior.


2) Note que a variação da coordenada x do ponto (4, 3) para o ponto (8, y) é igual a

8 − 4 = 4.

Portanto, para que o ponto (8, y) pertença à reta r, a variação correspondente na coordenada y deve ser
3/4 da variação da coordenada x, ou seja,

3
y−8= · 4 = 3,
4
ou seja, y = 11. Portanto, o ponto (8, 11) pertence à reta r.
3) A variação da ordenada entre os pontos (4, 3) e (x, 9) é dada por

9 − 3 = 6.

Já a variação da abscissa entre esses pontos é

x − 3.

14
Usando (1), temos
3
(x − 3) = 6,
4
ou seja,
4
x−4= · 6.
3
Portanto
x − 4 = 4 · 2 = 8,
de onde segue que x = 12. Portanto, o ponto (12, 9) pertence à reta r.
4) A variação da abscissa do ponto (4, 3) ao ponto (32, y) é dada por

32 − 4 = 28.

Logo, a variação da ordenada é dada por


3
y−3= · 28 = 3 · 7 = 21.
4
Portanto, y = 24. Concluímos que o ponto (32, 24) pertence a r.
5) Raciocinando de modo ligeiramente diferente, observe que, do ponto (4, 3) para o ponto (x, 27), a
segunda coordenada (isto é, a ordenada) teve 8 aumentos sucessivos de 3 unidades, uma vez que

27 = 3 + 8 · 3.

Logo, a primeira coordenada (ou seja, a abscissa) deve ter 8 aumentos sucessivos de 4, a partir do valor
inicial 4, ou seja,
x = 4 + 8 · 4 = 4 + 32 = 36.
Portanto, o ponto (36, 27) pertence a r.

Em todas essa situações, perceba que, dado um valor da variável x, existe um único valor da
variável y tal que o par (x,y) pertence a r. Isso significa que y é função de x e que r é o gráfico
dessa função

6) e 7) Um ponto com coordenadas (x, y) pertence à reta r se, e somente se, dado outro ponto (x0 , y0 )
dessa reta, a variação das ordenadas de um ponto a outro é 3/4 da variação das abscissas entre esses
pontos, ou seja
y − y0 3
=
x − x0 4
Como a origem (4, 3) pertence à reta r, podemos fixar x0 = 4 e y0 = 3, obtendo
y−3 3
= ,
x−4 4
ou seja,
3
y − 3 = (x − 4)
4
e, portanto,
3
y = x. (2)
4
Essa é a expressão algébrica de y como função de x.
8) Observe que
3
6 6= · 9,
4

15
pois 6 · 4 6= 3 · 9. Portanto, em vista do critério dado pela expressão (2), o ponto (9, 6), em que x = 9 e
y = 6, não pertence a r.
9) Dada a condição (2), verificamos que o ponto (10, 15/2) pertence a r, pois
15 3
= · 10,
2 4
uma vez que 15 · 4 = 2 · 3 · 10.
10) Do ponto (10, 15/2) para o ponto (x, 15/4), a ordenada foi dividida por 2. Tendo em conta a relação
de proporcionalidade (2) entre ordenadas (valores da variável y) e abscissas (valores da variável x),
devemos ter
10
x= − 5.
2
Assim, o ponto (5, 15/4) pertence a r. De fato,
15 3
= · 5.
4 4
11) Se x = 3, o valor correspondente de y é dado por
3 9
y = ·3=
4 4
12) Dado um ponto (x, y) em r, devemos ter
3
y = x.
4
Assim, caso as coordenadas desse ponto também satisfaçam a equação x + y = 14, temos
3
x + x = 14.
4
Logo,
4x + 3x
= 14
4
e, sendo assim,
14 · 4
x= = 2 · 4 = 8.
7
Portanto,
3
y = ·8=6
4
(ou, equivalentemente, y = 14 − x = 14 − 8 = 6). Portanto, o ponto (8, 6) pertence, simultaneamente, à
reta r e à reta definida pela equação x + y = 14, conforme ilustrado na seguinte figura:
y
27
Q
24 r

21
18
15
12
9
A
6
P
3
s
0 4 8 12 16 20 24 28 32 x

16
Neste ponto, cabe discutir que a expressão x + y = 14 define, de fato, uma reta. Para tanto, devemos
observar que os pontos I = (14, 0) e J = (0, 14) satisfazem, ambos, a equação x + y = 14. Agora,
dado um ponto U = (x, y) qualquer, satisfazendo essa equação, a razão entre a variação da ordenada
e a variação da abscissa entre os pontos U e J é igual a
y − 14 14 − x − 14 −x
= = = −1.
x−0 x x
Portanto, a variação da ordenada é igual a −1 vezes a variação da abscissa: isso significa que, a
medida em que a variação das abscissas e a variação das ordenadas são proporcionais uma a outra.
De fato, observe na figura que, aumentando em 1 unidade o valor da variável x, diminuímos o valor
da variável y em 1 unidade, sempre nessa mesma proporção.

Observação 0.3 Caro(a) professor(a), até este ponto, avançamos gradualmente da noção de equi-
valência de frações ao conceito de proporcionalidade. Em seguida, representamos geométrica e
graficamente a ideia de proporcionalidade em termos de alinhamento de pontos no plano cartesiano,
cujas coordenadas mantêm uma relação de proporcionalidade.
A partir daqui, vamos tratar de dois temas:

• nesta seção, continuamos o percurso com critérios de semelhança de figuras planas em termos de
relações de proporcionalidade entre medidas de lados correspondentes; as noções de perímetro
e área de figuras planas; e as relações entre perímetro e área de figuras planas semelhantes.
• Na seção seguinte, estudamos as representações algébrica e gráfica de funções afins e de equações
lineares, interpretando geometricamente os coeficientes angular e linear em suas expressões
algébricas.

Observação 0.4 Os valores de x e y na tabela da questão 9 satisfazem a seguinte relação de


proporcionalidade:
y 3
= , (3)
x 4
sempre que x 6= 0. Por exemplo, os valores x = 8 e y = 6 podem estar na tabela, pois
6 3
= ,
8 4
frações que são equivalentes, uma vez que 6 · 4 = 8 · 3. Já os valores x = 9 e y = 6 não podem estar
na tabela, dado que as frações
6 3
e
9 4
não são equivalentes, visto que
6 · 4 6= 9 · 3.
Voltando ao caso geral, observe que a relação de equivalência (3) pode ser escrita na forma
3
y = x. (4)
4
Essa expressão mostra que a variável y é uma função afim da variável x. Se, por exemplo, tomamos
x = 8, temos
3 8
y = · 8 = 3 · = 3 · 2 = 6,
4 4

17
ou seja, o valor da variável y correspondente ao valor x = 8 é y = 6. Por essa razão, o ponto com
coordenadas (8, 6) está alinhado aos demais pontos cujas coordenadas completam a tabela na questão
9, a saber,
(12, 9), (16, 12), (20, 15), (24, 18), (28, 21), (32, 24).
Lembre-se que você representou esses pontos no plano cartesiano na questão 10 e, assim, verificou
que pertencem a uma mesma reta, que denotamos por r. Essa reta r, representada na seguinte figura,
é o gráfico da função 4.
y

Q
24
F

E
18
D

C
12
B

A
6
P

0 4 8 12 16 20 24 28 32 x

Note que o alinhamento dos pontos em uma mesma reta no plano significa que, aumentando 4
unidades no valor da variável x, avançamos 3 = 34 · 4 unidades no valor da variável y.
Note que, na função afim em (4), o coeficiente angular é dado por a = 3/4. Já o coeficiente
linear é dado por b = 0, uma vez que o gráfico dessa função afim intersecta o eixo y no ponto (0,0),
ou seja, na origem: de fato, se x = 0, então y = 43 · 0 = 0.

Questão 11 Considere o gráfico da função afim

3
y= x (5)
4
representado na figura exposta na observação 0.4. Responda às seguintes questões:

1) Quais as coordenadas (x, y) dos pontos P, A, B, C, D, E, F e Q?


2) Essas coordenadas satisfazem a relação (5)?
3) Essas coordenadas satisfazem a equação linear 3x − 4y = 0?
4) O ponto (6, 4) pertence ao gráfico da função afim?
5) O ponto (14, 9/2) pertence ao gráfico da função afim?
6) As coordenadas do ponto (14, 10) satisfazem a equação linear 3x − 4y = 0?
7) As coordenadas do ponto (6, 9/2) satisfazem a equação linear 3x − 4y = 0?
8) Qual a taxa de variação da função (5) entre os pontos P e A?
9) Qual a taxa de variação da função (5) entre os pontos P e Q?
10) Qual a taxa da variação da função (5) entre o ponto P e um ponto (x, y) qualquer do gráfico?
11) As coordenadas x dos pontos P, A, B, C, D, E, F e Q estão em progressão aritmética. Qual sua
razão?
12) As coordenadas y dos pontos P, A, B, C, D, E, F e Q estão em progressão aritmética. Qual sua
razão?
13) Dividindo essas razões, obtemos a taxa de variação da função afim. Verdadeiro ou falso?

18
Solução. 1) e 2) Na resolução da questão (10), vimos que

P = (4, 3), A = (8, 6), B = (12, 9), C = (16, 12), D = (20, 15), E = (24, 18), F = (28, 21), Q = (32, 24).

O padrão formado pelas coordenadas desses pontos foi, então, descrito da seguinte forma:

A variação da coordenada y entre dois desses pontos é proporcional à variação da coordenada x entre
eles: de um ponto para o seguinte, da esquerda para a direita, a coordenada x aumenta 4 unidades ao
passo que a coordenada y aumenta 3 unidades.

Esses pontos estão alinhados e, portanto, pertencem a uma mesma reta, que denotamos por r. Essa
reta contém a origem O = (0,0). Sendo assim, dado um ponto (x, y) qualquer em r, temos
3
y − 0 = (x − 0),
4
ou seja,
3
y = x.
4

3) Observamos que a relação de proporcionalidade


3
y= x
4
pode ser escrita como
4y = 3x,
o que significa que o aumento da coordenada y em 3 unidades equivale ao aumento da coordenada x em
4 unidades. Note que todos os pontos na lista acima satisfazem essa equação, De fato, temos

4 · 3 = 3 · 4, 4 · 6 = 3 · 8, 4 · 9 = 3 · 12, 4 · 12 = 3 · 16, 4 · 15 = 3 · 20,

e assim por diante. Concluímos que as coordenadas (x, y) desses pontos satisfazem a equação linear

3x − 4y = 0.

4) O ponto (6, 4) tem coordenadas x = 6 e y = 4 satisfaz

3x − 4y = 3 · 6 − 4 · 4 = 18 − 16 = 2 6= 0.

Logo, esse ponto não pertence à reta r.


5) O ponto (14, 9/2) tem coordenadas x = 6 e y = 4 satisfaz
9
3x − 4y = 3 · 14 − 4 · = 42 − 18 = 24 6= 0.
2
Logo, esse ponto não pertence à reta r.
6) O ponto (14, 10) tem coordenadas x = 6 e y = 4 satisfaz

3x − 4y = 3 · 14 − 4 · 10 = 42 − 40 = 2 6= 0.

Logo, esse ponto não pertence à reta r.


7) O ponto (6, 9/2) tem coordenadas x = 6 e y = 4 satisfaz
9
3x − 4y = 3 · 6 − 4 · = 18 − 18 = 0.
2

19
Logo, esse ponto pertence à reta r.
8) a 10) A relação de proporcionalidade
3
y = x. (6)
4
entre as coordenadas (x, y) de pontos na reta r significa que a razão entre as variações da coordenada x
e da coordenada y são proporcionais. Como O = (0,0) pertence à reta r, temos
variação de y y−0 y 3
= = = ·
variação de x x−0 x 4
Logo, a taxa de variação de y como função de x é dada por 34 . Em resumo, a expressão (6) significa
que y é uma função linear de x, com taxa de variação igual a 34 . A reta r é o gráfico dessa função.
Os pontos P = (4, 3) e A = (8, 6) pertencem à reta r. A taxa da variação da função entre esses dois
pontos é igual a
6−3 3
=
8−4 4
e, da mesma forma, a taxa da variação da função entre os pontos P = (4,3) e Q = (32, 24) é igual a
24 − 3 21 21 : 7 3
= = = ·
32 − 4 28 28 : 7 4
A taxa de variação entre dois pontos quaisquer do gráfico é sempre igual a 34 . Essa taxa de variação é a
razão entre as variações da coordenada y e da coordenada x entre pontos da reta.
11) a 13) Como mencionado no início, as abscissas dos pontos estão em uma progressão aritmética com
razão 4. Isso significa que
8 = 4 + 4,
12 = 8 + 4,
16 = 12 + 4,
20 = 16 + 4,
e assim por diante. De modo similar, as ordenadas desses pontos estão em uma progressão aritmética
com razão 3, ou seja,
6 = 3 + 3,
9 = 6 + 3,
12 = 9 + 3,
15 = 12 + 3,
e assim sucessivamente. Dividindo as razões dessas progressões aritmética, temos, exatamente, a taxa
de variação da função, isto é, 34 .


Observação 0.5 Veja que a variável y é função afim da variável x segundo a expressão

3
y= x
4
se, e somente se, 4y = 3x, isto é, se, e somente se, as coordenadas (x, y) satisfazem a equação linear

3x − 4y = 0. (7)

Portanto, a reta r que contém os pontos P e Q, representada na figura da observação 0.4, é o lugar
geométrico dos pontos (x, y) cujas coordenadas são as soluções da equação linear (7). Por exemplo, o

20
ponto (8, 6), com x = 8 e y = 6 pertence a essa reta, visto que

3 · 8 − 4 · 6 = 24 − 24 = 0.

Já o ponto (9, 6), com x = 9 e y = 6, não pertence à reta r, posto que

3 · 9 − 4 · 6 = 27 − 24 = 3 6= 0.

De fato, o ponto com x = 9 que pertence a essa reta é o ponto (9, 27/4), uma vez que
27 4
3·9−4· = 27 − 27 · = 27 − 27 = 0.
4 4

Considere, novamente, o gráfico da função afim y = 34 x representado na seguinte figura.


y

Q
24
F β

E
18
D

C
12
B

A
6
P

α γ
x
O P0 A0 B0 C0 D0 E0 F0 Q0

Nessa figura, destacamos os triângulos retângulos OP P 0 , OAA0 , OBB 0 , OCC 0 , ODD0 , OEE 0 ,
OF F 0 e OQQ0 . Um dos vértices desses triângulos é a origem O, outro é um ponto sobre o gráfico da
função (por exemplo, P ) e o terceiro é a projeção desse ponto do gráfico sobre o eixo x (por exemplo,
P 0 ). Na figura, destacamos um desses triângulos, a saber, o triângulo OP P 0 .
Observe que esses triângulos têm o mesmo ângulo α no vértice O. Além disso, os ângulos nos
pontos correspondentes desses triângulos têm a mesma medida, ou seja,
.
∠P = ∠A = ∠B = ∠C = ∠D = ∠E = ∠F = ∠Q = β

e
.
∠P 0 = ∠A0 = ∠B 0 = ∠C 0 = ∠D0 = ∠E 0 = ∠F 0 = ∠Q0 = γ.
Note que γ é um ângulo reto, ou seja, sua medida é igual a 90◦ (lê-se “noventa graus”).

Questão 12 Dadas as informações no gráfico anterior, resolva os seguintes problemas.

1) Qual a soma das medidas dos ângulos α + β + γ?


2) Qual a soma das medidas dos ângulos α + β?
3) Mostre a seguinte igualdade entre as razões:

PP0 AA0 BB 0 CC 0 DD0 EE 0 FF0 QQ0 3


0
= 0
= 0
= 0
= 0
= 0
= 0
= 0
= (8)
OP OA OB OC OD OE OF OQ 4

21
Solução. 1) Os ângulos α, β e γ são os ângulos interno do triângulo OQQ0 . Portanto, a soma das
medidas desses ângulos é igual a 180◦ ou π radianos. Assim,

α + β + γ = 180◦ .

2) Uma vez que os segmentos OQ0 e QQ0 são perpendiculares um ao outro. Portanto, γ é um ângulo
reto, ou seja, cuja medida é igual a 90◦ ou π/2 radianos. Logo, a soma das medidas dos ângulos α e
beta é igual a
α + β = 180◦ − γ = 180◦ − 90◦ = 90◦ .

3) Temos
PP0 3 AA0 6 3 BB 0 9 3
0
= , 0
= = , 0
= =
OP 4 OA 8 4 OB 12 4
e
CC 0 12 3 DD0 15 3 EE 0 18 3
0
= = , 0
= = , 0
= =
OC 16 4 OD 20 4 OE 24 4
Finalmente,
FF0 21 3 QQ0 24 3
0
= = , 0
= =
OF 28 4 OQ 32 4
O fato de que as razões entre as medidas de lados correspondentes são iguais comprovam que os
triângulos retângulos OP P 0 , OAA0 , OBB 0 , OCC 0 , ODD0 , OEE 0 , OF F 0 , OQQ0 são semelhantes. Além
disso, todos esses triângulos têm ângulo α no vértice O; os ângulos nos vértices P, A, B, C, D, E, F, Q
têm mesma medida, igual a β; finalmente, os ângulos nos vértices P 0 , A0 , B 0 , C 0 , D0 , E 0 , F 0 , Q0 têm mesma
medida, igual a β = 90◦ .


Os triângulos OAA0 e OQQ0 acima são semelhantes, isto é, as medidas dos lados correspondentes
nesses dois triângulos são proporcionais. Para verificar isso, note que os pares de lados correspondentes
nesses triângulos são

OA e OQ OA0 e OQ0 AA0 e QQ0 .

Observe que a medida de AA0 está para a medida de QQ0 assim como a medida de OA0 está para a
medida de OQ0 , ou seja,
AA0 QQ0
0
= ·
OA OQ0
Essas razões são iguais à tangente tg α do ângulo α.
Q

α γ

O A0 Q0

22
Observe que esse é um ângulo interno de ambos os triângulos no vértice O. Da mesma forma, a
medida de OA0 está para a medida de OQ0 assim como a medida de OA está para a medida de OQ,
ou seja,
OA0 OQ0
= ·
OA OQ
Essa razão comum aos dois triângulos é chamada do cosseno do ângulo α e denotada por cos α.
Finalmente, temos a igualdade entre as razões

AA0 QQ0
= ·
OA OQ
Essa razão define o seno sen α do ângulo α. Em nosso exemplo, temos
3 4
sen α = e cos α = ·
5 5
Logo,
32 42 9 + 16
sen2 α + cos2 α =
2
+ 2 = = 1.
5 5 25
Esse resultado é um caso particular do Teorema de Pitágoras, segundo o qual as medidas dos
segmentos OA0 e AA0 (catetos) no triângulo retângulo OAA0 estão relacionados à medida do
segmento OA (hipotenusa) pela seguinte expressão

OA02 + AA02 = OA2 . (9)

Outro critério de semelhança que permite verificar que, de fato, os triângulos OAA0 e OQQ0
são semelhantes é o seguinte: os ângulos nos vértices correspondentes têm medidas iguais, ou seja,

• os ângulos de OAA0 e OQQ0 no vértice O têm medida α;


• os ângulos de OAA0 em A e de OQQ0 em Q têm medida β;
• os ângulos de OAA0 em A0 e de OQQ0 em Q0 têm medida γ.

Observação 0.6 As razões em (8) são todas iguais a 3/4, o coeficiente angular da função afim
(5). Vimos que elas definem a chamada tangente do ângulo α. Logo,
3
tg α = ,
4
ou seja, o coeficiente angular da função afim é igual a tangente do ângulo entre o eixo x e o gráfico
da função. Note que, quanto maior o ângulo α (medido entre 0 e 90◦ ) entre o eixo x e o gráfico da
função (ou seja, quanto mais inclinado o gráfico, em relação à direção horizontal), maior o coeficiente
angular da função afim. Esse coeficiente angular é a tangente do ângulo α.
Veja, na seguinte figura, o exemplo das funções afins
1 3 7
y = x, y= x e y = x,
3 4 5
cujos coeficientes angulares estão na seguinte ordem
1 3 7
< < ·
3 4 5

23
y S = (20, 28)
28

Q
24

20

16

12
R

O S 0 = (20,0) R0 x

A reta que contém os pontos O e R é o gráfico menos inclinado em relação ao eixo x dentre os três
gráficos na figura. Observe que o coeficiente angular da função afim y = 13 x é igual a a = 1/3: essa
é a taxa de variação dessa função. De fato, dados os pontos O = (0,0) e R = (x, y) no gráfico da
função, essa taxa de variação é dada por
y−0 y 1
= = ·
x−0 x 3
Note que essa taxa de variação é a tangente do ângulo β no vértice O do triângulo ORR0 , pois

RR0 y
tg β = 0
= ,
OR x
onde (x, y) são as coordenadas do ponto R. Da mesma forma, a reta que contém os pontos O e S é o
gráfico de maior inclinação dentre os três gráficos na figura. Se as coordenadas do ponto S são dadas
por (20, 28), essa inclinação é dada pela tangente do ângulo γ no vértice O do triângulo OSS 0 , ou
seja, por
SS 0 28 28 : 4 7
tg γ = = = = ,
OS 0 20 20 : 4 5
ou seja, a inclinação da reta é dada pela taxa de variação da função
7
y = x.
5
Concluímos que a inclinação da reta é igual a taxa de variação da função afim que, por sua vez, é
dada pelo seu coeficiente angular.

Questão 13 A seguinte figura representa triângulos semelhantes OAA0 e P QQ0 , sendo que

∠O = ∠P = α, ∠A = ∠Q = β, ∠A0 = ∠Q0 = γ.

Suponha, além disso, que


AA0 QQ0 3
0
= 0
= ·
OA PQ 4

24
Q

A
β

α γ α γ
O A0 P Q0

Responda às seguintes questões.


OA0 P Q0
1) É verdade que OA = PQ ? Em caso afirmativo, qual a razão de semelhança?
AA0 QQ0
2) É verdade que OA = PQ ?Em caso afirmativo, qual a razão de semelhança?
3) Se OA0 = 4 centímetros e P Q0 = 16 centímetros, quais as medidas de AA0 e de QQ0 ?
4) Nessas condições, quais as medidas de OA e de P Q?
5) Nessas condições, qual a razão entre os perímetros dos dois triângulos?
6) Nessas condições, qual a razão entre as áreas dos dois triângulos?

Solução. 1) e 2) O enunciado da questão informa que os dois triângulos têm ângulos com mesma
medida em vértices correspondentes, ou seja,

∠O = ∠P = α, ∠A = ∠Q = β, ∠A0 = ∠Q0 = γ.

Isso implica que os dois triângulos são semelhantes e, portanto, as medidas de lados correspondentes
são proporcionais, ou seja, existe uma razão de semelhança entre essas medidas. De acordo com o
enunciado, temos
AA0 QQ0 3
0
= = · (10)
OA P Q0 4
Como os triângulos OAA0 são triângulos retângulos, com ângulo reto nos vértices A0 e Q0 , respectivamente,
o Teorema de Pitágoras implica que

OA2 = OA02 + AA02

e
P Q2 = P Q02 + QQ02 .
Usando a relação de proporcionalidade (10), deduzimos que
2
3 9 9 25
  
OA2 = OA02 + OA0 = OA02 + OA02 = 1 + OA02 = OA02
4 16 16 16

e, de modo similar,
2
3 9 9 25
  
02
2
PQ = PQ + P Q0 = P Q + P Q02 = 1 +
02
P Q02 = P Q02
4 16 16 16

Calculando as raízes quadradas dos termos nessas expressões, obtemos


5 5
OA = OA0 e P Q = P Q0 .
4 4
Sendo assim, concluímos que
OA0 P Q0 4
= = ·
OA PQ 5

25
Por fim, temos
AA0 AA0 OA0 3 4 3
= 0
= · =
OA OA OA 4 5 5
e
QQ0 QQ0 P Q0 3 4 3
= 0
= · = ·
PQ PQ PQ 4 5 4

3) e 4) De acordo com as relações de proporcionalidade (10), temos


3 3
AA0 = OA0 = · 4 = 3 centímetros
4 4
e
3 3
QQ0 = P Q0 = · 16 = 3 · 4 = 12 centímetros.
4 4
Da mesma forma, a relação de proporcionalidade
OA0 P Q0 4
= =
OA PQ 5
implica que
5 5
OA = OA0 = · 4 = 5 centímetros
4 4
e
5 5
P Q = P Q0 = · 16 = 5 · 4 = 20 centímetros.
4 4
5) Por definição, o perímetro do triângulo retângulo OAA0 é a soma das medidas de seus lados, isto é,

OA0 + AA0 + OA = 4 + 3 + 5 = 12 centímetros.

Analogamente, o perímetro do triângulo retângulo P QQ0 é a soma das medidas de seus lados, ou seja,

P Q0 + QQ0 + P Q = 16 + 12 + 20 = 48 centímetros.

De modo mais simples, observe que as medidas de lados correspondentes nos triângulos retângulos
semelhantes OAA0 e P QQ0 estão em uma relação de proporcionalidade:
P Q0 QQ0 PQ
0
= 0
= = 4.
OA AA OA
Com essa observação, concluímos que o perímetro de P QQ0 é 4 vezes o perímetro de OAA0 , ou seja,

P Q0 + QQ0 + P Q = 4 · (OA0 + AA0 + OA) = 4 · 12 = 48 centímetros.

6) Vimos, na questão 5) anterior, que a razão entre os perímetros dos dois triângulos semelhantes é
igual a razão entre as medidas dos lados correspondentes nesses triângulos, ou seja,
P Q0 + QQ0 + P Q P Q0 QQ0 PQ
0 0
= 0
= 0
= = 4.
OA + AA + OA OA AA OA
Já a razão entre as áreas desses triângulos não é igual a razão entre as medidas dos lados correspondentes.
De fato, a área do triângulo OAA0 é igual a
1 4·3
OA0 · AA0 = = 6 cm2 ,
2 2
enquanto a área do triângulo P QQ0 é dada por
1 16 · 12
P Q0 · QQ0 = = 16 · 6 = 96 cm2 .
2 2

26
Observamos, assim, que a razão entre as área dos dois triângulos é o quadrado da razão entre as
medidas dos lados correspondentes, uma vez que

16 = 42 .

Isso pode ser visto também com os seguintes cálculos:

1 (4 · OA0 ) · (4 · AA0 ) OA0 · AA0


área de P QQ0 = P Q0 · QQ0 = =4·4· = 16 · área de OAA0 .
2 2 2


Questão 14 As seguintes figuras representam cinco triângulos retângulos (I, II, III, IV e V) e as
respectivas medidas de alguns de seus lados ou ângulos.
D

B S E0
α
C
A
6 cm 6cm
4 cm
3 cm
α θ θ
O 4 cm A0 P 8 cm B0 Q 4 cm C 0 R 8 cm D0 E
I II III IV V

Quais desses triângulos são semelhantes?

A) I e III B) I, II e V C) I e IV D) III e V E) II, III e IV

Solução. Os triângulos OAA0 , P BB 0 , QCC 0 , RDD0 e SEE 0 são triângulos retângulos com ângulos
retos nos vértices A0 , B 0 , C 0 , D0 e E 0 , respectivamente. Além disso, de acordo com a figura, as medidas
dos catetos dos triângulos OAA0 e P BB 0 estão em uma relação de proporcionalidade. De fato, temos

BB 0 6
= = 2.
AA0 3
e
P B0 8
0
= = 2.
OA 4
As medidas das hipotenusas desses triângulos podem ser determinadas utilizando-se o Teorema de
Pitágoras, segundo o qual

OA2 = OA02 + AA02 = 42 + 32 = 16 + 9 = 25 = 52

e
P B 2 = P B 02 + BB 02 = 82 + 62 = 64 + 36 = 100 = 102 .
Portanto, as medidas das hipotenusas, dadas por OA = 5 cm e P B = 10 cm, também satisfazem a
mesma relação de proporcionalidade, ou seja,

PB 10
= = 2.
OA 5
Concluímos que a razão de semelhança entre os triângulos P BB 0 e OAA0 é igual a 2.
Dado que os triângulos OAA0 e P BB 0 são semelhantes, as medidas dos ângulos correspondentes são
iguais, ou seja,
∠O = ∠P = α, ∠A0 = ∠B 0 = 90◦ , ∠A = ∠B = 90◦ − α.

27
Na terceira sequência de igualdades, usamos o fato que a soma dos ângulos internos de cada um dos
triângulos é igual a 180◦ e, portanto,

∠O + ∠A0 + ∠A = 180◦ =⇒ ∠A = 180◦ − ∠A0 − ∠O = 180◦ − 90◦ − α = 90◦ − α.

Agora, observamos que o triângulo SEE 0 tem ângulos com medidas α e 90◦ nos vértices S e E 0 ,
respectivamente. Assim, o ângulo em E tem medida 90◦ − α. Logo, os ângulos nos triângulos OAA0 e
em SEE 0 , nos vértices correspondentes, têm mesmas medidas, ou seja,

∠O = ∠S = α, ∠A0 = ∠E 0 = 90◦ , ∠A = ∠E = 90◦ − α.

Concluímos que os triângulos OAA0 e SEE 0 são semelhantes. Por isso, as medidas de lados correspon-
dentes nesses triângulos são proporcionais: visto que

EE 0 6
0
= = 2,
AA 3
a razão de semelhança entre os triângulos semelhantes SEE 0 e OAA0 é igual a 2. Logo,

SE 0 SE
=2 e =2
OA0 OA
e, portanto,
SE 0 = 2 · 4 = 8 cm e SE = 2 · 5 = 10 cm.
O mesmo argumento mostra que os triângulos QCC 0 e RDD0 são semelhantes, com

∠Q = ∠R = θ, ∠A0 = ∠E 0 = 90◦ , ∠A = ∠E = 90◦ − θ.

Neste caso, a razão de semelhança entre os triângulos RDD0 e QCC 0 é também igual a 2, visto que

RD0 8
0
= = 2.
QC 4

Da mesma forma,
DD0 RD
=2 e = 2.
CC 0 QC
Portanto,
DD0 = 2 · 4 = 8 cm
e
RD = 2 · QC.
Por outro lado, pelo Teorema de Pitágoras, temos

QC 2 = QC 02 + CC 02 = 42 + 42 = 2 · 42 .

Com isso, concluímos que QC = 4 2 cm e
√ √
RD = 2 · 4 2 = 8 2 cm.

Em resumo, com a discussão anterior, concluímos que

• os triângulos OAA0 , P BB 0 e SEE” são semelhantes;


• os triângulos QCC 0 e RDD0 são semelhantes.

28
Todavia, os triângulos OAA0 e QCC 0 não são semelhantes. De fato, as medidas de seus lados não estão
em uma relação de proporcionalidade. Por exemplo, temos
OA0
= 1,
QC 0
e, no entanto,
AA0 3
0
=
CC 4
Decorre disso que os ângulos em O e em Q não têm a mesma medida. De fato, temos

θ > α.

Da mesma forma, temos


OA0 3 AA0
= 1 > =
CC 0 4 QC 0
e
∠C = 90◦ − θ > ∠A = 90◦ − α.
Concluímos que a alternativa correta é B). 

Questão 15 Calcule as áreas e as tangentes dos ângulos α e θ nos triângulos I a V na questão 14


anterior.

Solução. Vimos que os catetos do triângulo retângulo OAA0 tem medidas iguais a

OA0 = 4 cm e AA0 = 3 cm.

Logo, tomando OA0 como base e AA0 como a altura correspondente a essa base, a área de OAA0 é dada
por
1 4·3
área de OAA0 = OA0 · AA0 = = 6 cm2 .
2 2
Para calcular a área do triângulo P BB 0 , lembramos, da questão anterior, que esse triângulo é semelhante
ao triângulo P BB 0 , com razão de semelhança igual a 2. Logo, a área de P BB 0 deve ser 22 = 4 vezes
a área de OAA0 , ou seja, é igual a 4 · 6 = 24 cm2 . Para verificar esse valor, podemos calcular diretamente
essa área, considerando como base do triângulo o cateto P B 0 e como a altura correspondente o cateto
BB 0 . Sendo assim, temos
1 8·6
área de P BB 0 = OA0 · AA0 = = 24 cm2 .
2 2
O mesmo raciocínio se aplica ao triângulo SEE 0 , visto que esse triângulo é também semelhante ao
triângulo OAA0 com a mesma razão de semelhança, a saber, 2. Isso significa que os triângulos P BB 0 e
SEE 0 são congruentes, ou seja, semelhantes, mas com razão de semelhança igual a 1. Portanto, não
apenas as medidas dos ângulos correspondentes nos triângulos P BB 0 e SEE 0 são iguais: as medidas de
lados correspondentes são também iguais (e não simplesmente proporcionais). Concluímos que

área de SEE 0 = área de P BB 0 = 24 cm2 .

Com relação a tangentes, observamos que, por definição, a tangente de α (medida do ângulo em O
no triângulo OAA0 ) é dada por
AA0 3
tg α = = ·
OA0 4
Da mesma forma, a tangente de θ (medida do ângulo em Q no triângulo QCC 0 ) é dada por
QC 0 4
tg θ = 0
= = 1·
CC 4

29
Aproveitamos para observar que o triângulo QCC 0 é um triângulo isósceles uma vez que os lados
QC 0 e CC 0 têm medidas iguais (de fato, medem 4 cm, de acordo com a figura). Sendo assim, os ângulos
em P e C também têm mesma medida, ou seja,
∠P = ∠C = θ.
Uma vez que
∠P + ∠C + ∠C 0 = 180◦
e ∠C 0 = 90◦ , deduzimos que
2θ = 180◦ − 90◦ = 90◦ ,
ou seja, que θ = 45◦ .


Questão 16 Use o Teorema de Pitágoras para calcular os perímetros dos triângulos I a V na questão
14.

Solução. Na resolução da questão 14, determinamos as medidas dos lados dos triângulos OAA0 e
QCC 0 . De fato, temos
OA0 = 4 cm, AA0 = 3 cm, OA = 5 cm
e √
QC 0 = 4 cm, CC 0 = 4 cm, QC = 4 2 cm
Sendo assim, calculamos
perímetro de OAA0 = OA0 + AA0 + OA = 4 + 3 + 5 = 12 cm
e √ √
perímetro de QCC 0 = QC 0 + CC 0 + QC = 4 + 4 + 4 2 = 4(2 + 2) cm.
Para calcular os perímetros dos demais triângulos, não é necessário determinar explicitamente as
medidas de seus lados: basta, na verdade, usarmos as razões de semelhança entre os triângulos. Por
exemplo, a razão de semelhança entre os triângulos P BB 0 e OAA0 é igual a 2; logo, a razão entre os
perímetros desses triângulos é também igual a 2. Temos, portanto,
perímetro de P BB 0 = 2 · perímetro de OAA0 = 2 · 12 = 24 cm.
Como os triângulos SEE 0 e P BB 0 são congruentes, têm o mesmo perímetro, ou seja,
perímetro de SEE 0 = perímetro de P BB 0 = 24 cm.
Por fim, a razão de semelhança entre os triângulos RDD0 e QCC 0 é igual a 2. Logo,
√ √
perímetro de RDD0 = 2 · perímetro de QCC 0 = 2 · 4(2 + 2) = 8(2 + 2) cm.


Questão 17 As seguintes figuras representam cinco quadriláteros (I, II, III, IV, V e VI). Cada um
dos lados dos quadriláteros I, II e III mede 5 centímetros.

IV V
VI
I II III

30
Assinale a alternativa em que temos um par de quadriláteros semelhantes.

A) I e II B) I e III C) II e IV D) II e VI E) IV e V

Solução. Como enunciado, os lados dos quadriláteros I e II têm medidas iguais (5 cm cada um
deles). No entanto, os quatro ângulos internos no quadrilátero têm a mesma medida, ou seja, cada
um deles mede 90◦ . Com isso, podemos afirmar que o quadrilátero II é um quadrado. Quanto ao
quadrilátero I, observamos que os pares de ângulos em vértices opostos têm mesma medida, sendo que
a medida de cada ângulo em um desses pares é maior que 90◦ (ângulos obtusos) enquanto a medida
de cada ângulo no outro par é menor que 90◦ (ângulos agudos). Desse modo, o quadrilátero II é um
losango, mas não é um retângulo. Portanto, não é um quadrado.
Quanto ao quadrilátero VI, observamos que cada um de seus quatro ângulos internos mede 90◦
(são ângulos retos). Além disso, seus lados tem medidas iguais (na escala da figura, essas medidas são
iguais a 10 cm). Portanto, esse quadrilátero é um quadrado semelhante ao quadrado I, com razão de
semelhança igual a
10
= 2.
5

Também de acordo com o enunciado, os lados do quadrilátero III medem, cada um, 5 cm. Logo, esse
quadrilátero é também um losango. Novamente, observe que as medidas de ângulos opostos são iguais,
havendo dois ângulos com mesma medida, maior que 90◦ ; e outros dois ângulos com mesma mesma
medida, mas menor que 90◦ . Em particular, II e III não são semelhantes e VI e III não são semelhantes.
Afirmamos que os losangos I e III não são semelhantes: de fato, no losango I, o segmento que liga
os vértices opostos com ângulos obtusos mede 8 cm. Portanto, esse segmento “divide” o losango I em
dois triângulos que não são equiláteros. Em particular, a medida do ângulo agudo é maior que 60◦ .
Quanto ao losango III, o segmento que liga os vértices opostos com ângulos obtusos divide o losango em
dois triângulos equiláteros e, portanto, cada ângulo agudo em III mede 60◦ . Portanto, fica demonstrada
nossa afirmação. A figura seguinte esclarece esse argumento:

IV V
VI
II

Sobre o quadrilátero IV, percebemos, com a ajuda da seguinte figura, que suas diagonais medem 12 cm
e 9 cm. Essas diagonais intersectam-se em seu ponto médio. Logo, usando o Teorema de Pitágoras,
concluímos que os lados de IV têm medidas iguais e que, portanto, esse quadrilátero é um losango. Por
outro lado, as diagonais do losango I medem, de acordo com a figura, 8 cm e 6 cm. Sendo assim, as
medidas das diagonais nos losangos I e IV não são proporcionais, pois

6 8
6= ·
9 12

Portanto, I e IV não são semelhantes. Além disso, no losango IV, o segmento que liga os vértices opostos
com ângulos obtusos “divide” o losango em dois triângulos que não são equiláteros. Portanto, seguindo
o mesmo raciocínio que antes, deduzimos que os losangos III e IV não são semelhantes.

31
V
VI
II

Finalmente, observe que, no quadrilátero V, temos dois lados opostos paralelos, com medidas iguais a
8 cm. Com relação ao outro par, o Teorema de Pitágoras nos permite concluir que cada um dos lados
mede p √ √
72 + 42 = 49 + 16 = 65 > 8 cm.
Portanto, V não é um losango. Logo, não pode ser semelhante a nenhuma das outras cinco figuras.
Concluímos que apenas os quadriláteros II e VI (dois quadrados) são semelhantes. 

Questão 18 A respeito dos quadriláteros de I a V I na questão 17, calcule valores exatos ou estime
valores aproximados

1) das áreas dos quadriláteros.


2) das razões entre as áreas de pares de quadriláteros.
3) das medidas das tangentes dos ângulos em cada um dos quadriláteros.
4) dos perímetros dos quadriláteros.
5) das razões entre os perímetros de pares de quadriláteros.

Solução. 1) e 2) O quadrado II tem lados com medida igual a 5 cm. Logo, sua área é igual a
52 = 25 cm2 . Da mesma forma, o quadrado VI tem lados com o dobro da medida da medida dos
lados de II: portanto, sua área é 22 = 4 vezes a área de II. Assim, a área do quadrado VI é igual a
4 · 25 = 100 cm2 .
Com a ajuda das figuras na resolução da questão 17, observamos que o losango I é “decomposto” em
quatro triângulos retângulos com catetos medindo 4 cm e 3 cm. Assim, a área de I é dada por
4·3
4· = 2 · 4 · 3 = 24 cm2 .
2
Da mesma forma, o losango IV é “decomposto” em quatro triângulos retângulos com catetos medindo
6 cm e 4,5 cm. Assim, a área de IV é dada por
6 · 4,5
4· = 2 · 6 · 4,5 = 54 cm2 .
2
A figura seguinte permite visualizar melhor essas decomposições:

V
VI
II

Quanto a área do paralelogramo V, a figura anterior sugere a seguinte estratégia: considere o retângulo
cujos lados tem medidas 12 cm e 7 cm que pode ser decomposto em I e em dois triângulos retângulos
com catetos medindo 7 cm e 4 cm. Portanto, a área de V é dada por
7·4
 
12 · 7 − 2 · = 12 · 7 − 4 · 7 = 8 · 7 = 56 cm2 .
2

32
Já o losango III é decomposto em dois triângulos equiláteros cujos lados medem 5 cm. A altura de cada
um desses triângulos equiláteros é igual a

3
q 2 q q
2 2 2
5 − 5/2 = 5 − 5 /4 = 3 · 5 /4 = 2 · 5 cm.
2

5 h

A
5
2

Portanto, a área de III é igual a duas vezes a área do triângulo equilátero, ou seja,
√ √
3
5· ·5 3
2· 2
= 25 · cm2 .
2 2

4) e 5) O perímetro do quadrado I é igual a

5 + 5 + 5 + 5 = 4 · 5 = 20 cm,

ao passo que o perímetro do quadrado VI é dado por

2 · 20 = 40 cm,

onde usamos o fato de que a medida de cada um dos lados de VI é 2 vezes a medida de cada um dos
lados de I. As medidas de cada um dos lados dos losangos II e III é igual a 5 cm. Portanto, os perímetros
desses dois losangos são também iguais a 20 cm, cada um.
Pelo Teorema de Pitágoras, cada um dos lados do losango IV mede
q q q 15
(9/2)2 + 62 = (81 + 144)/4 = 225/4 = = 7,5 cm.
2
Assim, o perímetro do losango IV é igual a

4 · 7,5 = 30 cm.

Finalmente,
√ vimos, na resolução da questão anterior, que o paralelogramo V tem lados com medidas
8 cm e 65 cm. Assim, o perímetro de V é dado por

16 + 2 65 cm.

3) No losango III, destacado na figura anterior, os ângulos nos vértices A e B medem, cada um, 60◦ .
Observamos que a tangente é dada por

◦ h 3 · 5/2 √
tg 60 = = = 3.
5/2 5/2

Deixamos os demais caso como exercício para o leitor (requer o uso de expressões para a tangente do
arco duplo e outros expedientes técnicos). 

33
Observação 0.7 Os quadrados representados nas figuras II e IV são semelhantes: de fato, dois
quadrados são sempre semelhantes (por quê?) Os lados do quadrado II medem 5 centímetros e os
lados do quadrado IV medem 10 centímetros. Logo, a razão de proporcionalidade entre os lados (e
os perímetros) do quadrado IV e do quadrado II é igual a 2. No entanto, observe que a razão entre a
área do quadrado IV e a área do quadrado II é igual a 4, ou seja, a 22 : de fato, temos 102 /52 = 4.

Questão 19 Na figura seguinte, os quadrados que formam a malha quadriculada têm lados medindo
1 cm cada um.
A B C D E

P Q

Responda às seguintes questões a respeito dos triângulos P AQ, P BQ, P CQ, P DQ e P EQ destacados
na figura.

1) Os triângulos P AQ e P EQ são semelhantes?


2) Os triângulos P BQ e P DQ são semelhantes?
3) Os triângulos P AQ e P CQ são semelhantes?
4) Os triângulos P AQ e P BQ são semelhantes?
5) Os triângulos P BQ e P CQ são semelhantes?
6) Qual a altura desses triângulos com respeito a base P Q, comum a todos eles?
7) As áreas desses triângulos são iguais. Por qual razão?
8) Qual(is) desse(s) triângulo(s) tem (têm) o maior perímetro?
9) Qual(is) desse(s) triângulo(s) tem (têm) o menor perímetro?
10) Quais os senos, cossenos e tangentes dos ângulos internos do triângulo P CQ?

Solução. 1) e 2) Os triângulos P AQ e P EQ são congruentes: observe, pela simetria da figura,


que as medidas de AP e EQ são iguais assim como as medidas de AQ e EP . Além disso, esses dois
triângulos têm o lado P Q em comum. Assim, os triângulos P AQ e P EQ têm lados correspondentes
com mesma medida. São, portanto, congruentes.
Da mesma forma, os triângulos P BQ e P DQ são congruentes, visto que têm o lado P Q em comum;
além disso, os lados BP e DQ têm mesma medida assim como BQ e DP . Portanto, esses triângulos
tem pares de lados com medidas iguais. Logo, são congruentes.
Lembre que dois triângulos são congruentes quando são semelhantes com razão de semelhança
igual a a 1.

Os movimentos rígidos ou isometrias no plano são as translações, rotações (fixando um dado


ponto) e as reflexões (fixando um dado eixo). Dois triângulos T e T 0 (ou, mais geralmente, duas
figuras planas) são congruentes se um deles pode ser obtido a partir do outro por uma sequência de
uma ou mais isometrias.

34
T T0

Temos os seguintes critérios de congruência de triângulos: dois triângulos T e T 0 são congruentes


se, e somente se, uma das seguintes condições for verdadeira:

• podemos definir uma correspondência entre os lados de T e os lados de T 0 de modo que lados
correspondentes têm a mesma medida (critério LLL): isso significa que, se é possível construir
um triângulo com três segmentos de reta dados, esse triângulo é único a menos de isometrias;
• podemos definir uma correspondência entre dois lados de T e dois lados de T 0 de modo que
lados correspondentes têm a mesma medida e os ângulos determinados por esses pares de lados
têm mesma medida (critério LAL): isso quer dizer que, se é possível construir um triângulo,
dados dois segmentos de reta e o ângulo definido por eles (não-nulo e não-raso), esse triângulo é
único a menos de isometrias;
• podemos definir uma correspondência entre dois ângulos de T e dois ângulos de T 0 de modo que
ângulos correspondentes têm a mesma medida e os lados comuns a esses ângulos têm mesma
medida (critério ALA): isso quer dizer que, se é possível construir um triângulo, dados um lado
e os ângulos que contêm esse lado, esse triângulo é único a menos de isometrias.

Recomendamos, a respeito, a leitura do seguinte material da OBMEP: http://clubes.obmep.


org.br/blog/sala-para-leitura_024-um-pouco-sobre-congruencia-de-triangulos/ e https:
//cdnportaldaobmep.impa.br/portaldaobmep/uploads/material_teorico/hiphzxsnhjk88.pdf

3) a 5) Os triângulos P AQ e P CQ não são semelhantes. Se houvesse uma relação de semelhança entre


P AQ e P CQ, o lado P Q em P AQ não poderia corresponder a CP ou a CQ no triângulo P CQ, pois,
do contrário, a razão de semelhança seria
CP CQ
ou
PQ PQ
e, em um caso ou no outro, seria maior que 1. Mas, sendo assim, um dos lados AP ou AQ em P AQ
deveria corresponder ao lado P Q em P CQ. Teríamos, então, razão de semelhança
PQ PQ
ou ,
AP AQ
um número menor que 1. Logo, o lado P Q em P AQ deveria corresponder, em uma relação de semelhança,
ao lado P Q em P CQ. Mas essa possibilidade também não ocorre, pois analisando a figura, constatamos
que o ângulo de P AQ em P tem medida maior que o ângulo de P CQ em P ; por outro lado, o ângulo
de P AQ em Q tem medida menor que o ângulo de P CQ em Q.
Argumentos similares mostram que os triângulos P AQ e P BQ não são semelhantes; da mesma
forma, demonstramos que os triângulos P BQ e P CQ não são semelhantes.
6) e 7) Todos os triângulos destacados na figura seguinte têm altura com medida igual a 8 cm: os
segmentos AA0 , BB 0 , CC 0 , DD0 e EE 0 são perpendiculares à reta suporte da base P Q dos triângulos
P AQ, P BQ, P CQ, P DQ e P EQ. Todos esses segmentos têm medida igual a 8 cm. Como P Q mede
4 cm, a área de cada um desses triângulos é igual a
4·8
= 16 cm2 .
2

35
A B C D E

A0 B0 P C0 Q D0 E0

8) e 9) Usando o Teorema de Pitágoras, temos


p √ √
AP = DP = BQ = EQ = 62 + 82 =
36 + 64 = 100 = 10 cm,
p √ √ √
BP = CP = CQ = DQ = 22 + 82 = 4 + 64 = 4 · 17 = 2 17 cm,
p √ √ √
EP = AQ = 102 + 82 = 100 + 64 = 4 · 41 = 2 41 cm.

Portanto,
√ √
perímetro de P AQ = perímetro de P EQ = 4 + 10 + 2 41 = 2(7 + 41) cm,
√ √
perímetro de P BQ = perímetro de P DQ = 4 + 10 + 2 17 = 2(7 + 17) cm,
√ √ √
perímetro de P CQ = 4 + 2 17 + 2 17 = 4(1 + 17) cm.

10) O ângulo no vértice P do triângulo P CQ tem tangente dada por

CC 0 8
tg(∠P ) = 0
= = 4.
PC 2
Da mesma forma
CC 0 8
tg(∠Q) = = = 4.
QC 0 2
Agora, calculemos o seno desses ângulos: temos

CC 0 8 4
sen(∠P ) = = √ =√
PC 2 17 17
e
CC 0 8 4
sen(∠Q) = = √ =√ ·
QC 2 17 17
Finalmente, o cosseno desses ângulos é dado por

P C0 2 1 QC 0 2 1
cos(∠P ) = = √ =√ e cos(∠Q) = = √ =√
PC 2 17 17 QC 2 17 17
Note que
sen(∠P )
tg(∠P ) = e sen2 (∠P ) + cos2 (∠P ) = 1.
cos(∠P )
O cálculo das razões trigonométricas no vértice C requer o uso de algumas expressões trigonométricas e
pode ser dispensado numa primeira leitura desse material. Temos
1 8 8
sen(∠C) = 2 sen(∠C/2) cos(∠C/2) = 2 · √ · √ =
17 2 17 17

36
e
16 1 15
cos(∠C) = cos2 (∠C/2) − sen2 (∠C/2) = − = ·
17 17 17
Logo,
sen(∠C) 8
tg(∠C) = = ·
cos(∠C) 15


Temos os seguintes critérios de semelhança de triângulos: dois triângulos T e T 0 são semelhantes


se, e somente se, uma das seguintes condições for verdadeira:

• podemos definir uma correspondência entre os lados de T e os lados de T 0 de modo que lados
correspondentes têm medidas proporcionais (critério LLL): a constante de proporcionalidade
entre essas medidas é a razão de semelhança entre os triângulos;
• podemos definir uma correspondência entre dois lados de T e dois lados de T 0 de modo que
lados correspondentes têm medidas proporcionais e os ângulos determinados por esses pares de
lados têm mesma medida (critério LAL);
• podemos definir uma correspondência entre os ângulos internos de T e os ângulos internos de T 0
de modo que ângulos correspondentes têm a mesma medida (critério AAA).

Recomendamos, a respeito, a leitura do seguinte material da OBMEP: https://cdnportaldaobmep.


impa.br/portaldaobmep/uploads/material_teorico/c72gbsow17sow.pdf

Questão 20 — SAEPE - Item M120392ES. No desenho abaixo estão representados os triângulos I, II,
III e IV e suas medidas em centímetros.

O par de triângulos semelhantes nesse desenho é

A) I e II. B) I e III. C) I e IV. D) II e IV. E) III e IV.

Solução. Podemos colocar os lados com medidas 6 cm e 8 cm no triângulo I em correspondência


com os lados do triângulo III cujas medidas são 12 cm e 16 cm. Esses pares de lados determinam, tanto
no triângulo I quando no triângulo III, um ângulo reto. Portanto, pelo critério LAL, os triângulos I e
III são semelhantes com razão de semelhança igual a 2.
Note que é possível também usar o critério LLL, apenas observando que o lado com medida 10 cm
no triângulo I é posto em correspondência com o lado no triângulo III cuja medida é 20 cm.
Completando a resolução, observamos, ainda, que nenhum dos triângulos II e IV é retângulo, ou seja,
nenhum dos dois tem um ângulo reto. Logo, não podem ser semelhantes a I e III. Além disso, não há
como definir uma correspondência entre os lados de II e IV de modo que as respectivas medidas sejam
proporcionais. De fato, comparando os respectivos lados, daquele de maior medida para o de menor
medida, temos
12 10
6= ·
24 12
Portanto, a alternativa correta é B). 

37
Questão 21 — SAEPE - Item M110375E4. Observe os triângulos abaixo.

Qual desses triângulos são semelhantes?

A) I, II e III. B) I, II e V. C) I e III. D) II e IV. E) III e V.

Solução. Usando o critério AAA de semelhança de triângulos, constatamos que os triângulos I e II


são semelhantes. Observamos, além disso, que, no triângulo V, a medida do ângulo não informada na
figura é igual a
180◦ − 40◦ − 105◦ = 35◦ .

Logo, os ângulos internos em V são iguais aos ângulos internos correspondentes em I. Logo, usando
uma vez mais o critério AAA, concluímos que os triângulos I e V são também semelhantes. Finalmente,
podemos verificar que os ângulos internos nos triângulos III e IV não são todos iguais aos ângulos
internos em I. Além disso, não há igualdade entre os ângulos internos de III e os ângulos internos de
IV. Portanto, não são semelhantes um ou outro e não são semelhantes a I, III e V. Concluímos que a
alternativa correta é B) 

Observação 0.8 Com a ajuda de seu professor(a), faça desenhos geométricos, usando uma malha
quadriculada, para “comprovar” que dois triângulos OAB e O0 A0 B 0 são semelhantes se uma das
seguintes condições ocorre: a) têm ângulos (digamos, em O e O0 ) com mesma medida e as razões
entre os lados OA/O0 A0 e OB/O0 B 0 são iguais; ou b) ângulos correspondentes têm a mesma medida,
ou seja, ∠O = ∠O0 , ∠A = ∠A0 , ∠B = ∠B 0 . Nesse caso, “comprove”, em seus desenhos, que a razão
entre os perímetros dos triângulos é igual a razão entre seus lados e que a razão entre suas áreas é
igual ao quadrado da razão entre os lados.

Observação 0.9 No percurso, até este ponto, trabalhamos problemas envolvendo semelhança de
figuras planas enfatizando as conexões desse tópico com a ideia de proporcionalidade. Aproveitamos
o contexto geométrico das questões com triângulos e quadriláteros para, lateralmente, revisitarmos
o Teorema de Pitágoras e, então, utilizá-lo tanto no cálculo de perímetros quanto de razões trigo-
nométricas. Destacamos, dentre as razões trigonométricas, a tangente, dado que essa razão surge,
naturalmente, no estudo da declividade de retas no plano cartesiano e, portanto, na interpretação
geométrica do coeficiente angular na expressão algébrica de funções afins.
Alcançado esse objetivo, aprofundamos, nesse final de percurso, o estudo de áreas e perímetros
em conexão com os descritores D11 e D12 da Matriz de Referência do SAEB. Na seção seguinte,
retomamos os conhecimentos sobre coordenadas no plano, proporcionalidade, semelhança e declividade
de retas ao estudarmos as funções afins e as equações lineares.

38
Questão 22 Na figura seguinte, os quadrados que formam a malha quadriculada têm lados medindo
2 cm cada um.

Sendo assim, qual o perímetro da figura em forma de cruz destacada na figura seguinte?

A) 21 cm B) 28 cm C) 32 cm D) 56 cm E) 64 cm

Solução. Uma forma direta, mas trabalhosa e sujeita a erros, é contar o número de segmentos de 2
cm que compõem a figura. Em cada braço da cruz, temos 6 + 2 desses segmentos. Logo, multiplicando
esse número por 2 e tendo em conta que a cruz tem 4 braços, obtemos
2 · 4 · (6 + 2) = 64 cm.
Uma abordagem alternativa é transladar os segmentos que compõem a cruz e, com isso, constatar que
obtemos o perímetro externo do quadrado:

De um modo ou de outro, obtemos a resposta 64 cm, ou seja, a alternativa correta é E).




Questão 23 — SPAECE - Item M110782E4. O desenho abaixo apresenta as dimensões da laje da casa
de Isadora. Ela irá colocar um muro de proteção nessa laje e, para calcular a quantidade de material
a ser comprado, precisou medir o seu contorno.

39
Qual é o perímetro da laje dessa casa?

(a) 58 m (b) 116 m (c) 232 m (d) 360 m (e) 720 m

Solução. Para calcular o perímetro do retângulo, devemos somar

40 + 18 + 40 + 18 = 80 + 36 = 116 m.

Logo, a resposta correta é a alternativa B). 

Questão 24 — SAEPE - Item M120195H6. Paulo comprou um terreno retangular de 120 000 m2 . Esse
terreno possui 200 m de largura.
Quanto mede o comprimento desse terreno?

A) 200 m B) 300 m C) 400 m D) 600 m E) 800 m

Solução. A área do retângulo é o produto de suas dimensões, ou seja,

comprimento · largura = 120 000.

Como a largura é igual a 200 m, deduzimos que


120 000 1 200
comprimento = = = 600 m.
200 2
A resposta correta é a alternativa D). 

Questão 25 — SAEPE - Item M120195H6. Observe no desenho abaixo o projeto da quadra de basquete
que será construída no condomínio em que Bruno mora.

A medida da área dessa quadra de basquete, em metros quadrados, será

A) 480. B) 240. C) 92. D) 46. E) 14.

Solução. A área da quadra retangular é dada pelo produto de suas medidas lineares, ou seja, por
30 · 16 = 480 m2 . A resposta correta é a alternativa A). 

Questão 26 Dê exemplos de retângulos com mesma área e perímetros diferentes, e vice-versa.

Solução. A figura a seguir mostra vários exemplos de retângulos com mesma área (16 cm2 ) e
perímetros diferentes. Pense um pouco para decidir qual deles têm menor perímetro. Considere que os
quadrados que formam a malha quadriculada têm lados medindo 1 cm cada um.

40
Para reforçar o entendimento a respeito da distinção entre perímetro e área, vejamos alguns exemplos
de retângulos com mesmo perímetro e áreas diferentes. Desta vez, analise qual desses retângulos tem
maior área.

Questão 27 Observe as letras S e T desenhadas geometricamente na seguinte figura:

Podemos afirmar que

A) a área da letra S é menor do que a área da letra T.


B) a área da letra S é igual a área da letra T.
C) o perímetro da letra S é igual ao perímetro da letra T.
D) o perímetro da letra S é maior do que o perímetro da letra T.

Solução. Seja a a medida do lado de cada um dos quadrados que compõem a malha quadriculada.

41
Então
perímetro de S = 18a e perímetro de T = 16a
ao passo que
área de S = 8a2 e perímetro de T = 7a2 .
Logo, a alternativa correta é D).


Questão 28 — SAEPE - Item M100270E4. Marta comprou um terreno na forma de trapézio cujas
medidas estão representadas no desenho abaixo. Para construir um muro em torno desse terreno, ela
precisa calcular o seu perímetro.

Qual é o perímetro desse terreno?

A) 12 m B) 20 m C) 24 m D) 32 m E) 40 m

Solução. O perímetro é dado por 4 + 4 + 6 + 10 = 24 m, o que corresponde à alternativa C). 

Questão 29 Qual a área do terreno de Marta?


√ √
A) 20 m2 B) 24 m2 C) 32 m2 D) 8 12 m2 E) 10 12 m2

Solução. O cálculo da área requer determinarmos a altura h do trapézio, ou seja, a distância entre
os segmentos que são paralelos, chamados bases. Veja que a diferença entre as medidas das bases é
igual a 10 − 6 = 4 metros. Pela simetria da figura, deduzimos que

22 + h2 = 42 ,

ou seja, h2 = 16 − 4 = 12. Portanto, h = 2 3 metros. Com essa medida, podemos dividir o trapézio em
dois triângulos sem sobreposição, um dos quais tem base 10 m e a altura do trapézio; e outro, que tem
base 6 m e altura do trapézio. Sendo assim, a área do trapézio e a soma das áreas desses triângulos, ou
seja, √ √ √
10 · 2 3 6 · 2 3 (10 + 6) · 2 3 √ √ √ √
+ = = 16 · 3 = 8 · 2 · 3 = 8 · 4 · 3 = 8 · 12 m2 .
2 2 2


Questão 30 Marcos tem um terreno na forma de um quadrado. A seguinte figura representa esse
terreno, visto de cima, dividido em quadrados de lados iguais a 10 m.

42
R

Se Marcos deseja construir um muro separando as duas partes do terreno destacadas na figura,
quantos metros teria esse muro?

A) 48 m B) 200 m C) 240 m D) 400 m E) 480 m

Solução. Observe que, deslocando os segmentos horizontais e verticais que formam o muro, preen-
chemos o perímetro inteiro do terreno, sem sobreposições. Portanto, o perímetro do muro é igual ao
perímetro do terreno, ou seja, igual a 4 · 120 = 480m. 

Questão 31 A área da região R na figura corresponde a que percentual da área total do terreno?

A) 48% B) 50% C) 72% D) 75%

Solução. Deslocando quatro dos quadrados que formam a parte murada, obtemos duas partes do
terreno que são exatamente simétricas. Concluímos que a parte murada corresponde à metade, ou seja,
50% da área do terreno total. Logo, a resposta correta é B). 

Questão 32 Na seguinte figura, cada um dos 64 quadrados que formam o quadrado maior tem lados
medindo 1 unidade de comprimento.

8
7
6
5
4
3
2
1
O 1 2 3 4 5 6 7 8

Qual a razão entre a área destacada e área total do quadrado?

Solução. Para determinarmos essa razão, basta reconhecermos que a figura pode ser decomposta
em 8 triângulos cuja base mede 3 unidades de comprimento e cuja altura, relativa a essa base, mede 2
unidades de comprimento, conforme a seguinte figura:

43
8
7
6
5
4
3
2
1
O 1 2 3 4 5 6 7 8

Portanto, a área da figura corresponde a


3·2
8· = 24 unidades de área
2
ao passo que o quadrado como um todo tem 64 unidades de área. Logo, a razão entre a área da figura e
a área do quadrado é igual a
24 3
= ·
64 8


Questão 33 Na figura abaixo, estão representados os trapézios ABCD e M N OP , os quais são


semelhantes.

N
B
x

O
C

64 m y 32 m 80 m

D
P
25,6 m
x
A
M

De acordo com a figura, faça as seguintes atividades.

A) Identifique os pares de ângulos internos nos trapézios que são congruentes.


B) Determine a razão (de proporcionalidade) entre as medidas dos lados AB e M N .
C) Determine as medidas x e y.
D) Determine os perímetros dos dois trapézios e, em seguida, calcule a razão entre esses perímetros.
E) Determine a área do trapézio M N OP .
F) Determine a razão entre as áreas dos dois trapézios e a utilize para calcular a área do trapézio
ABCD.

Solução. A) A relação de semelhança implica igualdade das medidas de pares de ângulos internos
correspondentes. Temos:

∠A = ∠M, ∠B = ∠N, ∠C = ∠O, ∠D = ∠P.

44
B) A razão de semelhança entre as medidas de lados correspondentes nos trapézios ABCD e M N OP
é dada pela razão, por exemplo, entre as medidas dos segmentos correspondentes AB e M N , ou seja,
por
64 64 : 16 4
= = ·
80 80 : 16 5

C) Usando essa razão de semelhança e considerando que os lados AD e M P são correspondentes, temos

25,6 4
= ,
x 5
ou seja,
5 24,0 + 1,6
x= · 25,6 = 5 · = 5 · (6 + 0,4) = 30 + 2 = 32 m.
4 4
Da mesma forma, como os lados CD e OP são correspondentes, temos

y 4
= ,
32 5
de onde segue que
4 64
y= · 32 = 4 · = 4 · 6,4 = 25,6 m.
5 10

D) O perímetro de ABCD é dado por

64 + 25, 6 + 25, 6 + y = 64 + 3 · 25,6 = 64 + 75 + 1,8 = 140,8 m

ao passo que o perímetro de M N OP é igual a

80 + x + x + 32 = 80 + 3 · 32 = 80 + 96 = 176 m

Não é preciso, de fato, calcular explicitamente os dois perímetros, pois, pela relação de semelhança, é
necessário que o perímetro de ABCD seja igual a 45 do perímetro de M N OP . De fato, verificamos essa
razão de semelhança dividindo
140,8 8,8 4
= = 0,8 = ·
176 11 5

E) Para calcular a área de M N OP , determinamos, inicialmente, a altura desse trapézio relativa à base
M N . Usamos, para tanto, o Teorema de Pitágoras conforme a seguinte figura.

32 m 80 m

32 m
24 m

45
Note que, na figura, um dos catetos do triângulo retângulo mede 24 m, visto que 80 − 32 = 48 m é
a diferença das medidas dos lados M N e OP . Dividindo essa diferença por 2, obtemos a medida do
cateto. Sendo assim, temos
h2 + 242 = 322 ,
ou seja,
h2 = 82 · (42 − 32 ) = 82 · 7.
Portanto, √
h = 8 7 m.
Agora, observamos que o trapézio M N OP pode ser decomposto em dois triângulos, com altura h e
bases M N e OP , conforme a seguinte figura.

h
P

Então, a área de M N OP é dada por



1 1 7 √
OP · h + M N · h = · (80 + 32) = 56 7 m2 .
2 2 2
F) A área de ABCD pode ser calculada diretamente, No entanto, é bem mais simples. usar o fato de
que a razão entre as área dos trapézios semelhantes ABCD e M N OP é igual ao quadrado da razão
de semelhança entre essas figuras. Uma vez que conhecemos a área de M N OP , o cálculo da área de
ABCD leva em conta, portanto, que
 2
área de ABCD 4
= ,
área de M N OP 5
e, portanto,

16 16 · 56 7 √
área de ABCD = · área de M N OP = = 35,84 · 7 m2 .
25 25

Seção 2. Segundo percurso: coordenadas, proporcionalidade e funções

As tarefas a seguir envolvem conhecimentos prévios fundamentais para desenvolver as habilidades nos
seguintes descritores da Matriz de Referência do SAEB (terceira série do Ensino Médio):

• D6 - Identificar a localização de pontos no plano cartesiano.


• D7 - Interpretar geometricamente os coeficientes da equação de uma reta.
• D15 - Resolver problema que envolva variações proporcionais, diretas ou inversas, entre grandezas.
• D19 - Resolver situação problema envolvendo uma função de primeiro grau.

46
• D22 - Resolver problema envolvendo PA/PG, dada a fórmula do termo geral.
• D20 - Analisar crescimento/decrescimento, zeros de funções reais apresentadas em gráficos.
• D25 - Resolver problemas que envolvam os pontos de máximos ou de mínimo no gráfico de uma
função do segundo grau.

Questão 34 Em seu caderno, desenha uma malha quadriculada representando o plano cartesiano, de
acordo com os seguintes passos: a) trace duas retas r e s perpendiculares uma a outra; b) denote
o ponto de intersecção de r e s por O; c) trace retas paralelas a r, de modo que a distância entre
duas retas consecutivas seja sempre a mesma (digamos, 1 centímetro); d) trace retas paralelas a s,
de modo que a distância entre duas retas consecutivas seja sempre a mesma da letra c (digamos, 1
centímetro). Observe que, com esse procedimento, você “divide” o plano em quadrados de lados
com medidas iguais a 1 centímetro. Quando finalizar seu desenho, marque, no seu modelo de plano
cartesiano, os pontos (2, 0), (0, 2), (2, 2), (2, 1/2), (1/2, 3/2) e (4/10, 6/10).

Solução. Na figura a seguir, representamos o primeiro quadrante do plano cartesiano, cujos


pontos são associados a coordenadas cartesianas não-negativas. As linhas mais espessas estão a distância
de 1 unidade de medida uma da outra; as linhas menos espessas estão distantes uma da outra por 1/10
dessa unidade de medida.
y
20/10
B C

18/10

16/10
E

14/10

12/10

10/10

8/10

F
6/10
D

4/10

2/10

A
x
2/10 4/10 6/10 8/10 10/10 12/10 14/10 16/10 18/10 20/10

Note que

A = (2,0) = (20/10, 0), B = (0, 2) = (0, 20/10), C = (2, 2) = (20/10, 20/10),


D = (2, 1/2) = (2; 0,5) = (20/10, 5/10), E = (1/2, 3/2) = (0,5; 1,5) = (5/10, 15/10),
F = (4/10, 6/10).

Observe que os pontos A, E e B estão alinhados e que a soma das coordenadas de cada um desses pontos
é igual a 2. 

47
Questão 35 a) Marque, no plano cartesiano abaixo, os pontos com coordenadas

A = (−4, 3) B = (−4, −3) C = (3, 4) D = (−3, 4) E = (4, −3) F = (4, 3)

5
4
3
2
1

−5 −4 −3 −2 −1 1 2 3 4 5
−1
−2
−3
−4
−5

b) Determine para quais desses pontos as coordenadas (x, y) satisfazem a equação y = x + 1. Em


seguida, trace uma reta contendo esses pontos.
c) Faça o mesmo com relação à equação y = −x + 1.

Solução. a) Temos
5
D C y =x+1
4
A F
3

−5 −4 −3 −2 −1 1 2 3 4 5
−1

−2
B E
−3
y = −x + 1
−4

−5

b) As coordenadas de B são x = −4 e y = −3. Logo, satisfazem a equação y = x + 1, pois

y = −4 = −3 + 1 = x + 1.

Da mesma forma, as coordenadas de C são x = 3 e y = −4. Portanto, satisfazem

y = 4 = 3 + 1 = x + 1.

Note que a razão entre a variação da coordenada y e a variação da coordenada x de B a C é dada por
∆y 4 − (−3) 7
= = = 1.
∆x 3 − (−4) 7
Logo, a cada aumento de 1 unidade na variável x, há um aumento de 1 unidade na variável y. Isso
significa que a declividade da reta contendo esses pontos é 1. Dito de outro modo, um ponto (x, y)

48
pertence a essa reta se, e somente se,
y − (−3)
= 1,
x − (−4)
ou seja, se e somente se,
y + 3 = x + 4,
equação que pode ser escrita como
y = x + 1.
Concluímos que, de fato, um ponto (x, y) pertence à reta que contém B e C se, e somente se, y = x + 1.
Isso significa que essa reta é o gráfico da função

y = x + 1.

Nessa expressão, y é uma função afim de x. Note que o gráfico intersecta o eixo vertical (eixo das
ordenadas) quando x = 0 e y = 1.
c) As coordenadas de D são x = −3 e y = 4. Logo, satisfazem a equação y = −x + 1, pois

y = 4 = −(−3) + 1 = −x + 1.

Da mesma forma, as coordenadas de E são x = 4 e y = −3. Portanto, satisfazem

y = −3 = −4 + 1 = −x + 1.

A reta (pontilhada na figura anterior) que contém D e E é gráfico da função afim

y = −x + 1.

O coeficiente angular dessa função é −1: isso significa que o aumento de 1 unidade na variável x
implica a diminuição de 1 unidade na variável 1, ou seja,

∆y
= −1.
∆x
Por exemplo, a taxa de variação entre a variação da coordenada y e a variação da coordenada x do
ponto D para o ponto E é dada por
−3 − 4 −7
= = −1.
4 − (−3) 7

Logo, um ponto (x, y) pertence à reta contendo D e E se, e somente se,


y−4
= −1,
x − (−3)

isto é, se e somente se,


y − 4 = −(x + 3),
ou seja,
y = −x − 3 + 4 = −x + 1.
Note que o coeficiente linear dessa função afim é igual a 1: isso significa que, quando x = 0, temos
y = 1, ou seja, o ponto (0,1) é a intersecção do gráfico da função com o eixo vertical.


49
Caro(a) professor(a), aproveite a discussão desta questão para revisar os chamados “jogos de sinais”
na adição de números inteiros; outro tópico que pode ser revisado é a expressão decimal de frações
(usada para localizar, na reta numérica, os pontos correspondentes a 1/2, 3/2, e assim por diante).

Questão 36 — PAEBES - Item M120205G5, adaptado. Observe o plano cartesiano abaixo.

5
4
L R
3
H S
2
F
1

−5 −4 −3 −2 −1 1 2 3 4 5
−1
O −2

Os pontos que têm coordenadas (−2, −2) e (1, 3) são, respectivamente,

A) O e R. B) S e R. C) H e R. D) O e L. E) O e F.

Solução. Temos a seguinte correspondência entre os pontos destacados na figura e suas coordenadas
cartesianas:

F = (3, 1), H = (−2, 2), L = (−1, 3), R = (1, 3), S = (2, 2), O = (−2, −2).

A alternativa correta é A). 

Questão 37 — SAEPE - Item M120701H6, adaptado. Observe os pontos P, Q, R, S e T representados


no plano cartesiano abaixo.

S R

T Q
P

Em qual desses pontos a abscissa é −3 e a ordenada é −2?

A) P B) Q C) R D) S E) T

Solução. Temos a seguinte correspondência entre os pontos destacados na figura e suas coordenadas
cartesianas:

P = (−2, −3), Q = (3, −2), R = (3, 2), S = (−3, 2), T = (−3, −2).

A alternativa correta é E). 

A sequência das questões 38 a 44 faz referência à figura no enunciado da questão 38.

50
Questão 38 Observe os seguintes gráficos no plano cartesiano, em que alguns pontos estão
destacados.
variável y

22 000 P
20 000
(10, y0 )
18 000
16 000 (20, y1 )

14 000
Q
12 000
10 000 R
(x0 , 8 000)
8 000 (x1 , 7 000)
S
6 000
4 000
2 000

10 x1 30 variável x

Determine os seguintes dados:

1) As coordenadas x0 e x1 .
2) As coordenadas y0 e y1 .
3) As coordenadas dos pontos P e Q.
4) As coordenadas dos pontos R e S.

Solução. 1) A coordenada x0 pode ser lida diretamente na figura: veja que é abscissa do ponto
(x0 , 8 000), ou seja, x0 = 10. Já a coordenada x1 não está explicitamente informada na figura, embora
seja razoável considerar x1 = 20. De fato, x1 é o ponto médio, no eixo horizontal, entre os pontos 10 e
30.
2) A coordenada y0 pode ser lida diretamente na figura: veja que é abscissa do ponto (10, y0 ), ou seja,
y0 = 18 000. Já a coordenada y1 não está explicitamente informada na figura, embora seja razoável
considerar y1 = 15 000. De fato, y1 é o ponto médio, no eixo vertical, entre os pontos 14 000 e 16 000.
3) Os pontos P e Q têm coordenadas dadas, respectivamente, por (0, 21 000) e (30, 12 000).
4) Os pontos R e S têm coordenadas dadas, respectivamente, por (0, 9 000) e (30, 6 000).


Caro(a) professor(a), para comprovar todas essas afirmações e aproveitar a questão no estudo de
funções afins (e de suas representações gráficas e algébricas), observe que é possível determinar as
funções cujos gráficos são as retas destacadas no plano cartesiano da figura. No caso da reta pontilhada,
que contém os pontos (x0 , 8 000) = (10, 8 000) e S = (30, 6 000), a taxa de variação entre as variação
da ordenada e a variação da abscissa entre esses pontos é dada por
6 000 − 8 000 −2 000
= = −100.
30 − 10 20
Portanto, o coeficiente angular da função deve ser a = −100. Essa informação já nos permite
determinar precisamente as coordenadas do ponto R = (0, b). De fato, a taxa de variação entre a
variação da ordenada e a variação da abscissa do ponto R ao ponto S é também igual a −100 e,

51
portanto,
6 000 − b
= −100.
30 − 0
Logo,
6 0000 − b = −3 000,
ou seja,
b = 9 000,
como já havíamos intuído, na resolução da questão, a partir da análise da figura. Concluímos que
o coeficiente linear da função é dado por b = 9 000. Logo, a função afim cujo gráfico é a reta
pontilhada contendo R e S é dada por

y = −100
| {z } {z } .
x + 9| 000
=a =b

No que diz respeito à reta contínua, que contém os pontos (10, y0 ) = (10, 18 000) e Q = (30, 12 000),
a taxa de variação entre as variação da ordenada e a variação da abscissa entre esses pontos é dada
por
12 000 − 18 000 −6 000
= = −300.
30 − 10 20
Portanto, o coeficiente angular da função deve ser a = −300. Essa informação já nos permite
determinar precisamente as coordenadas do ponto P = (0, b). De fato, a taxa de variação entre a
variação da ordenada e a variação da abscissa do ponto P ao ponto Q é também igual a −100 e,
portanto,
12 000 − b
= −300.
30 − 0
Logo,
12 0000 − b = −9 000,
ou seja,
b = 21 000,
como já havíamos intuído, na resolução da questão, a partir da análise da figura. Concluímos que o
coeficiente linear da função cujo gráfico contém os pontos P e Q é dado por b = 21 000. Logo, essa
função afim é dada por
y = −300
| {z }
x + 21 000} .
| {z
=a =b

Questão 39 No plano cartesiano representado na figura da questão 38, marque os pontos cujas
coordenadas são

1) (0, 6 000) 2) (10, 12 000) 3) (15, 0) 4) (20, 11 000) 5) (25, 6 500)

Solução. Temos
variável y
14 000
(10, 12 000)
12 000 (20, 11 000)
10 000
8 000 (25, 6 500)
6 000 (0, 6 000)
4 000
2 000
(15, 0)
10 20 30 variável x

52


Questão 40 Observe, na figura da questão 38, o gráfico representado pela linha contínua que contém
os pontos P e Q. Qual dos seguintes pontos pertence a esse gráfico?

1) (0, 20 000) 2) (10, 20 000) 3) (15, 16 500) 4) (15, 15 000) 5) (25, 15 000)

Solução. Denotando
A = (0, 20 000), B = (10, 20 000), C = (15, 16 500), D = (15, 15 000), E = (25, 15 000),
constatamos que apenas C = (15, 16 500) pertence à reta contendo P e Q. Isso pode ser confirmado
usando a expressão da função afim cujo gráfico é essa reta. De fato, temos
−300 · 15 + 21 000 = −4 500 + 21 000 = 16 500.
variável y

22 000 P
B
20 000 A
(10, y0 )
18 000
C
16 000 (20, y1 ) E

14 000 D
Q
12 000

10 000

8 000

6 000

4 000

2 000

10 20 30 variável x

Questão 41 Observe, na figura da questão 38, o gráfico representado pela linha pontilhada que
contém os pontos R e S. Qual dos seguintes pontos pertence a esse gráfico?

1) (0, 10 000) 2) (5, 9 000) 3) (15, 8 000) 4) (15, 7 500) 5) (25, 6 000)

Solução. Denotando
A = (0, 10 000), B = (5, 9 000), C = (15, 8 000), D = (15, 7 500), E = (25, 6 000),
temos
variável y

10 000 A B
(x0 , 8 000) C
8 000 R (x1 , 7 000)

D S
6 000
E
4 000

2 000

10 20 30 variável x

53
Observamos que apenas D = (15, 7 500) pertence à reta contendo R e S. Isso pode ser confirmado
usando a expressão da função afim cujo gráfico é essa reta. De fato, temos

−100 · 15 + 9 000 = −1 500 + 9 000 = 7 500.

Questão 42 Trace, na figura da questão 38, o gráfico representado por uma linha reta que contenha
os pontos (0, 16 000) e (30, 11 000).

Solução. Denote A = (0, 16 000) e B = (30, 11 000). A reta desejada, contendo os pontos A e B,
está indicada na figura seguinte.
variável y

22 000 P
20 000
(10, 18 000)
18 000
16 000 (20, 15 000)
A
14 000
Q
12 000
10 000 R B
(10, 8 000)
8 000 (20, 7 000)
S
6 000
4 000
2 000

10 20 30 variável x

Cabe observar que a declividade desta reta é dada pela taxa de variação entre a variação da variável y e
a variação da variável x do ponto A para o ponto B, ou seja,
11 000 − 16 000 −5 000 500
= =− ·
30 − 0 30 3
Além disso, a reta intersecta o eixo vertical em A = (0, 16 000). Logo, a reta é gráfico da função afim
500
y=− x + 16 000,
3
em que o coeficiente angular é a = −500/3 e o coeficiente linear b = 16 000. 

Questão 43 Trace, na figura da questão 38, a linha reta que corresponde ao gráfico da função afim
y = 1 000x + 4 000.

Solução. Tomando x = 0 na expressão da função, temos

y = 1 000 · 0 + 4 000 = 4 000.

Logo, o gráfico dessa função intersecta o eixo vertical no ponto A = (0, 4 000). Por outro lado, o valor
da função quando x = 20 é dado por

y = 1 000 · 20 + 4 000 = 24 000.

Portanto, o gráfico da função é a reta que contém os pontos A = (0, 4 000) e B = (20, 24 000).

54
variável y

24 000
B
22 000

20 000

18 000

16 000

14 000

12 000

10 000

8 000

6 000

4 000 A

2 000

10 20 30 variável x

Questão 44 Trace, na figura da questão 38, a linha reta que corresponde ao gráfico da função afim
y = −4 000x + 20 000.

Solução. Tomando x = 0 na expressão da função, temos

y = −4 000 · 0 + 20 000 = 20 000.

Logo, o gráfico dessa função intersecta o eixo vertical no ponto A = (0, 4 000). Por outro lado, o valor
da função quando x = 5 é dado por

y = −4 000 · 5 + 20 000 = 0.
Portanto, o gráfico da função é a reta que contém os pontos A = (0, 4 000) e B = (5, 0).
variável y

4 000 A

2 000

B
5 10 variável x

Observação 0.10 A linha contínua que contém os pontos P e Q na figura da questão 38 é parte do
gráfico que representa a função afim
y = ax + b, (11)
cujos coeficientes são dados por a = −300 e b = 21 000. Logo, a função afim representada por esse
gráfico é dada por
y = −300x + 21 000.

55
Vejamos como determinar esses coeficientes. Para tanto, observe, inicialmente, que o ponto P tem
coordenadas (0, 21 000) e o ponto Q tem coordenadas (30, 12 000). Sendo assim, a variação da variável
x do ponto P para o ponto Q é dada por
30 − 0
e a variação correspondente da variável y é dada por

12 000 − 21 000 = −9 000.

Logo, a taxa de variação de y com relação a x é dada pela razão


12 000 − 21 000 −9 000
= = −300.
30 − 0 30
Concluímos que o coeficiente a em (11) é igual a taxa de variação, ou seja, a = −300.
Quanto ao coeficiente b em (11), observe que esse é o valor da variável y quando x = 0, pois

y(0) = a · 0 + b = b.

No nosso exemplo, temos


y(0) = −300 · 0 + b = 21 000.
Observe que (0, 21 000) são exatamente as coordenadas do ponto P em que o gráfico intersecta o eixo
y.

Em resumo, dada uma função afim y = ax + b,

1) o coeficiente a, denominado coeficiente angular, é dado pela taxa de variação da função:


dados dois pontos P = (x0 , y0 ) e Q = (x1 , y1 ) no gráfico da função, temos
y1 − y0
a= ,
x1 − x0
onde

y0 = ax0 + b,
y1 = ax1 + b.

2) Já o coeficiente linear b é dado pela coordenada (vertical) do ponto em que o gráfico da


função intersecta o eixo y, ou seja, b é o valor da função y quando x = 0. Logo, o ponto (0, b)
pertence ao gráfico da função.

Questão 45 a) Use o plano cartesiano representado na figura da questão 38 para traçar retas que
contenham os seguintes pares de pontos:

i) (0, 2 000) e (30, 20 000)


ii) (10, 0) e (30, 20 000)
iii) (10, 2 000) e (30, 20 000)
iv) (5, 2 000) e (20, 17 000)
v) (5, 5 000) e (25, 15 000)

b) Em seguida, determine os coeficientes a e b das funções y = ax + b cujos gráficos são dados pelas
retas que você traçou.
c) Por fim, determine qual dessas retas contém o ponto (20, 10 000).

56
Solução. a) A seguinte figura mostra segmentos das retas r, s e t contendo, respectivamente, os
pontos
A = (0, 2 000) e P = (30, 20 000), B = (10, 0) e P = (30, 20 000);
e
C = (10, 2 000) e P = (30, 20 000).
variável y
24 000
22 000
20 000
18 000
16 000
14 000
12 000 r t
10 000
s
8 000
6 000
4 000
2 000

10 20 30 variável x

Agora, a seguinte figura mostra segmentos das retas m e n contendo, respectivamente, os pontos
C = (5, 2 000) e Q = (20, 17 000), D = (5, 5 000) e R = (25, 15 000).
variável y
24 000
22 000
20 000
18 000
16 000
14 000 m
12 000
10 000
8 000 n
6 000
4 000
2 000

10 20 30 variável x

b) As declividades das retas r, s e t são, respectivamente, dadas pelas taxas de variação entre a variação
das ordenadas e a variação das abscissas entre os pontos A e P . B e P ; e C e P , respectivamente.
Portanto, as declividades das retas r, s e t são, respectivamente,
20 000 − 2 000 18 000
= = 600,
30 − 0 30

57
20 000 − 0 20 000
= = 1 000
30 − 10 20
e
20 000 − 2 000 18 000
= = 900.
30 − 10 20
Além disso, a reta r intersecta o eixo vertical (eixo das ordenadas) no ponto A = (0, 2 000). Concluímos,
assim, que a reta r é o gráfico da função afim
y = 600x + 2 000.
Por sua vez, a reta s é gráfico de função afim da forma
y = 1 000x + b,
em que o coeficiente linear b é a ordenada do ponto em que s intersecta o eixo vertical. A taxa de
variação da função deste ponto ao ponto B é igual a 1 000 e, portanto,
0−b
= 1 000.
10 − 0
Assim, b = −10 000. Concluímos que a reta s é o gráfico da função afim
y = 1 000x − 10 000.
A reta s, por sua vez, intersecta o eixo vertical em um ponto (0, b), onde b satisfaz
2 000 − b
= 900.
10 − 0
Logo, b = −7 000. Deduzimos, assim, que s é o gráfico da função afim
y = 900x − 7 000.
De modo similar, deduzimos que os coeficientes angular e linear da função afim cujo gráfico é m são
dados por
17 000 − 2 000 15 000
a= = = 1 000
20 − 5 15
e
2 000 − b
= 1 000,
5−0
ou seja, b = −3 000. Assim, a reta m é gráfico da função afim
y = 1 000x − 3 000.
Da mesma forma, deduzimos que os coeficientes angular e linear da função afim cujo gráfico é n são
dados por
15 000 − 5 000 10 000
a= = = 500
25 − 5 20
e
5 000 − b
= 500,
5−0
ou seja, b = 2 500. Assim, a reta n é gráfico da função afim
y = 500x + 2 500.

c) Pela análise dos gráficos nas figuras anteriores. o ponto (20, 10 000) pertence à reta s, ou seja, ao gráfico
da função y = 1 000x − 10 000. De fato, se x = 20, então, neste caso, y = 1 000 · 20 − 10 000 = 10 000.


58
Questão 46 Observe os seguintes gráficos no plano cartesiano, em que alguns pontos estão
destacados.
variável y

22 000 P
A
20 000
B
18 000
C
16 000 D
E
14 000
Q
12 000

10 000 R
A0
B0
C0
8 000 D0
E0 S
6 000

4 000

2 000

5 10 15 20 25 30 variável x

Determine os seguintes dados:

1) A variação da variável x entre os pontos P e B.


2) A variação da variável y entre os pontos P e B.
3) A taxa de variação de y em relação a x entre os pontos P e B.
4) A variação da variável x entre os pontos P e Q.
5) A variação da variável y entre os pontos P e Q.
6) A taxa de variação de y em relação a x entre os pontos P e Q.
7) O coeficiente angular (a) da função cujo gráfico contém os pontos P e Q.
8) O coeficiente linear (b) da função cujo gráfico contém os pontos P e Q.
9) O valor da variável y quando x = 7,5.

Solução. 1) Temos ∆x = 10 do ponto P = (0, 21 000) ao ponto B = (10, 18 000).


2) Temos ∆y = 18 000 − 21 000 = −3 000 do ponto P = (0, 21 000) ao ponto B = (10, 18 000).
3) A taxa de variação entre a variação de y e a variação de x ponto P = (0, 21 000) ao ponto
B = (10, 18 000) é igual a
∆y −3 000
= = −300.
∆x 10

4) Temos ∆x = 30 do ponto P = (0, 21 000) ao ponto Q = (30, 12 000).


5) Temos ∆y = 12 000 − 21 000 = −9 000 do ponto P = (0, 21 000) ao ponto Q = (10, 12 000).
6) A taxa de variação entre a variação de y e a variação de x ponto P = (0, 21 000) ao ponto
Q = (30, 12 000) é igual a
∆y −9 000
= = −300.
∆x 30

7) O coeficiente angular é dada pela taxa de variação de y como função de x entre dois pontos quaisquer
do gráfico. Nas etapas 1) a 6) anteriores, calculamos essa taxa de variação entre P e B e entre P e Q.
Como esperado, observamos que a taxa de variação é a mesma nesses dois pares de pontos. Concluímos
que o coeficiente angular da função afim é a = −300.

59
8) O coeficiente linear é, por definição, a ordenada b do ponto de intersecção (0, b) do gráfico da função
afim com o eixo vertical. Pela análise do gráfico, esse ponto é exatamente o ponto P = (0, 21 000). Logo,
b = 21 000 e, portanto, a expressão algébrica da função afim é

y = −300
| {z }
x + 21 000} .
| {z
=a =b

9) Dada a expressão acima da função afim, tomando x = 7,5, temos

y = −300 · 7,5 + 21 000 = −2 250 + 21 000 = 18 750.

Esse valor pode ser calculado diretamente a partir da taxa de variação. De fato, temos

y − 21 000
= −300.
7,5 − 0

Questão 47 Seja y = ax + b a função afim cujo gráfico é dado pela linha que contém os pontos R e S.

1) Qual a variação da variável x entre os seguintes pares de pontos: R e S; R e B 0 ; B 0 e S; C 0 e


D0 ; D0 e E 0 ; E 0 e S?
2) Qual a variação da variável y entre esses pares de pontos?
3) Qual o valor da variável y para os seguintes valores de x: x = 0, x = 2,5, x = 7,5 e x = 12,5?
4) A função cresce ou decresce à medida que x varia de x = 0 para x = 30?
5) Qual a taxa de variação de y em relação a x entre os pontos R e S?
6) Se tomássemos outro par de pontos (por exemplo, R e B 0 ou B 0 e S), a taxa de variação seria
a mesma ou mudaria? Justifique sua resposta.
7) Qual o coeficiente angular (a) na expressão da função y = ax+b cujo gráfico é a linha pontilhada
que contém R e S?
8) Qual o coeficiente linear (b) na expressão dessa função?
9) Para essa função afim, para qual valor da variável x temos y = 0?
10) Dada essa função afim, para qual valor de x temos y = 7 500?

Solução. 1) Temos

• Variação de x entre R e S: ∆x = 30 − 0 = 30.


• Variação de x entre R e B 0 : ∆x = 10 − 0 = 10.
• Variação de x entre B 0 e S: ∆x = 30 − 10 = 20.
• Variação de x entre C 0 e D0 : ∆x = 20 − 15 = 5.
• Variação de x entre D0 e E 0 : ∆x = 25 − 20 = 5.
• Variação de x entre E 0 e S: ∆x = 30 − 25 = 5.

2) Temos

• Variação de y entre R e S: ∆y = 6 000 − 9 000 = −3 000.


• Variação de y entre R e B 0 : ∆y = 8 000 − 9 000 = −1 000.
• Variação de y entre B 0 e S: ∆y = 6 000 − 8 000 = −2 000.
• Variação de y entre C 0 e D0 : ∆y = 7 000 − 7 500 = −500.
• Variação de y entre D0 e E 0 : ∆y = 6 500 − 7 000 = −500.
• Variação de y entre E 0 e S: ∆y = 6 000 − 6 500 = −500.

60
3) Observamos que um aumento de 10 unidades na variável x, isto é, ∆x = 10 implica diminuição de
1 000 unidades na variável y, ou seja, ∆y = −1 000. Proporcionalmente, temos

∆x = 5 =⇒ ∆y = −500.

Portanto, uma variação de 2,5 unidades em x implica uma variação de −250 em y, ou seja,

∆x = 2,5 =⇒ ∆y = −250.

Portanto, se x = 2,5 temos ∆x = 2,5 com relação a x = 0 e

∆y = y − 9 000 = −250,

ou seja,
y = 9 000 − 250 = 8 750
quando x = 2,5. Agora, se x = 7,5, temos
∆x = 5
em relação a x = 2,5 e, portanto,
∆y = y − 8 750 = −500.
Logo, se x = 7,5, então y = 8 250. Finalmente, se x = 12,5 temos ∆x = 5 com relação a x = 7,5 e

∆y = y − 8 250 = −500,

ou seja,
y = 8 250 − 500 = 7 750
quando x = 12,5.
4) a 6) Note que um aumento da variável x, isto é, uma variação positiva ∆x > 0 implica diminuição da
variável y, ou seja, uma variação negativa ∆y < 0, proporcional a ∆x. De fato, temos

∆y
= −100 ou ∆y = −100∆x
∆x
Em resumo, a taxa de variação da função afim é constante, igual a −100, independentemente do par
de pontos que consideramos no gráfico da função.
7) e 8) Determinamos, anteriormente, os coeficientes angular (a = −100) e linear (b = 9 000) da função
afim cujo gráfico é a reta que contém os pontos R e S. Temos

y = −100x + 9 000.

Observe que essa expressão algébrica nos permite confirmar os valores da função quando x = 2,5, x = 7,5
e x = 12,5, calculados anteriormente. De fato, temos

• x = 2,5 =⇒ y = −100 · 2,5 + 9 000 = −250 + 9 000 = 8 750.


• x = 7,5 =⇒ y = −100 · 7,5 + 9 000 = −750 + 9 000 = 8 250.
• x = 12,5 =⇒ y = −100 · 12,5 + 9 000 = −1 250 + 9 000 = 7 750.

9) e 10) De acordo com a expressão algébrica da função afim, temos y = 0 se, e somente se,

−100x + 9 000 = 0,

ou seja, se e somente se,


−9 000
x= = 90.
−100

61
Portanto, a função y = −100x + 9 000 tem um zero em x = 90.
De modo similar, para que y = 7 500 devemos ter

−100x + 9 000 = 7 500.

Portanto, o valor da variável x deve ser, neste caso,


7 500 − 9 000 −1 500
x= = = 15.
−100 −100


Observação 0.11 Note que um ponto A com coordenadas (x, y) pertence ao gráfico dado pela linha
inclinada que contém P e Q se, e somente se, a coordenada y é dada por

y = ax + b,

onde
a = −300 e b = 21 000.
Por exemplo, se x = 15, devemos ter

y(15) = −300 · 15 + 21 000 = −4 500 + 21 000 = 16 500.

Assim, o ponto (15, 16 500) pertence ao gráfico. Por outro lado, o ponto (20, 16 000) não pertence ao
gráfico, uma vez que o valor de y correspondente a x = 20 deve ser dado por

y(20) = −300 · 20 + 21 000 = −6 000 + 21 000 = 15 000.

Logo, o ponto do gráfico com coordenada x = 20 deve ser (20, 15 000) e não (20, 16 000).
Assim, dado um ponto A = (x, y) no gráfico que contém os pontos P e Q, temos y = ax + b e a
taxa de variação de y relativa a x entre os pontos P = (0,b) e A = (x, y) é dada por

y−b ax
= = a.
x−0 x

Em resumo, a linha contínua que contém os pontos P e Q é parte do gráfico da função afim

y = −300x + 21 000, (12)

onde a = −300 e b = 21 000. Já a linha pontilhada que contém os pontos R e S é parte do gráfico da
função afim
y = −100x + 9 000. (13)
onde a = −100 e b = 9 000. Observando a figura, note que o gráfico da função (12) é mais inclinado
do que o gráfico da função (13). Isso que dizer que, com a mesma variação da variável x, a variação
da função (12) é, em valores absolutos, maior do que a variação da função (13).
Por exemplo, se x varia de x = 10 para x = 20, a função (12) varia de y = 18 000 para y = 15 000
(veja os pontos B e D no gráfico contínuo) ao passo que a função (13) varia de y = 8 000 para y = 7 000
(veja os pontos B 0 e D0 no gráfico pontilhado). Logo, com a mesma variação de x (de x = 10 para
x = 20), temos as seguintes taxas de variação da função (12)
15 000 − 18 000 −3 000
= = −300
20 − 10 10

62
e da função (13)
7 000 − 8 000 −1 000
= = −100
20 − 10 10
Logo, a maior taxa de variação (em valores absolutos) é a da função (13), cujo gráfico é o mais
inclinado com relação ao eixo x.

Questão 48 Considere os gráficos na figura exposta na questão 46.

1) Estendendo as funções f (x) = −300x + 21 000 e g(x) = −100x + 9 000 para valores de x maiores
que 30, os seus gráficos se intersectam em algum ponto?
2) Qual o valor da coordenada x desse ponto de intersecção, caso exista?
3) Qual o valor da coordenada y desse ponto de intersecção, caso exista?

Solução. Do ponto de vista geométrico (ou gráfico), podemos resolver os problemas observando
como os gráficos são prolongados para valores de x maiores que 30:

22 000 P
20 000
18 000
16 000
14 000
12 000
10 000 R
8 000
6 000
4 000 Z
2 000

10 20 30 40 50 60

Constatamos que, a cada incremento de 10 unidades na variável x, a separação vertical entre os


gráficos diminui 2 000 unidades. Como iniciamos com uma separação vertical, quando x = 0, de 12 000
unidades, ou seja, 6 · 2 000 unidades, os gráficos se intersectam após um incremento de 6 · 10 = 60
unidades.
Verifiquemos essa constatação algebricamente: as funções cujos gráficos são representados pelas
linhas de P a Z e de R a Z são dadas, respectivamente, por

f (x) = −300x + 21 000

e
g(x) = −100x + 9 000.
No ponto Z as duas funções têm o mesmo valor, ou seja, a coordenada x do ponto Z é tal que

−300x + 21 000 = −100x + 9 000.

Somando 300x a cada um dos lados da igualdade, temos

21 000 = 300x − 100x + 9 000.

Subtraindo 9 000 de cada um dos lados dessa nova igualdade, obtemos

21 000 − 9 000 = 200x.

63
Assim, chegamos a uma equação bem mais simples:

200x = 12 000.

Dividindo cada um dos lados dessa equação por 200, deduzimos que

x = 60,

ou seja, a coordenada x do ponto Z é x = 60 ao passo que sua coordenada y é dada por

y = −300 · 60 + 21 000 = −18 000 + 21 000 = 3 000.

Portanto, Z = (60, 3 000).




Questão 49 Para qual valor da variável x a diferença entre os valores das funções

f (x) = −100x + 9 000

e
g(x) = −300x + 21 000,
nessa ordem, é igual a 2 000 unidades?

A) x = 0 B) x = 7 C) x = 50 D) x = 70 E) x = 2 000

Solução. Devemos ter


f (x) − g(x) = 2 000,
ou seja,
−100x + 9 000 − (−300x + 21 000) = 2 000,
o que equivale a
200x − 12 000 = 2 000,
equação que é, por sua vez, equivalente a

200x = 2 000 + 12 000 = 14 000.

Portanto, o valor adequado da variável x é


14 000
x= = 70,
200
o que corresponde à alternativa D). 

Observação 0.12 Nos próximos exercícios, discutiremos equações lineares em duas variáveis (x e
y, digamos) da forma

Ax + By = C, (14)

onde A, B e C são números dados, isto é, são coeficientes constantes. Observe que, se B 6= 0, podemos
reescrever a equação acima pondo y em função de x:
A C
y=− x+ ,
B B
A
uma função afim com coeficiente angular a = − B . Se B = 0 (e A 6= 0) na equação (14), essa

64
C
equação reduz-se a x = A. Nesse caso, temos uma reta vertical que intersecta o eixo horizontal no
ponto (C/A, 0).

Questão 50 — SPAECE - Item M120935E4, adaptado. Observe abaixo o gráfico de uma função poli-
nomial do 1◦ grau.
y
4

−2 −1 0 1 2 3 4 x
−1

−2

A lei de formação dessa função f é

A) y = −3x + 3 B) y = −x + 4 C) y = −x + 3 D) y = 2x + 1 E) y = 3x + 3

Solução. Observe que o gráfico está negativamente inclinado, isto é, a declividade do gráfico é
negativa. Além disso, à medida que os valores da coordenada x dos pontos sobre a reta aumentam, os
respectivos valores da coordenada diminuem. Em resumo, dada uma variação positiva da variável x, a
variação correspondente da variável y é negativa. Portanto, o coeficiente angular da função é negativo
e as alternativas D) e E) são falsas. Observamos, ainda, que o gráfico intersecta o eixo vertical no ponto
(0,3). Logo, o coeficiente linear b da função afim é igual a 3. Portanto, a alternativa B) não é correta.
Para decidirmos entre as alternativas A) e C), devemos calcular, explicitamente, o valor do coeficiente
angular. Veja que, quando a variável x varia de x = 0 para x = 3, a variável y varia de y = 3 para y = 0.
Assim, o coeficiente angular a é dado por
variação de y 0−3
a= = = −1,
variação de x 3−0

Concluímos que a função afim é expressa por y = −x + 3. Portanto, a alternativa correta é C). 

Observação 0.13 O gráfico da função afim y = −x + 4 é uma reta s paralela ao gráfico da função
y = −x + 3 (que indicamos por r na figura seguinte). De fato, as duas funções têm a mesma
declividade, ou seja, o mesmo coeficiente angular, a saber, a = −1. O que distingue uma função da
outra são seus coeficientes lineares: o coeficiente linear da função y = −x + 3 é b = 3, o que significa
que seu gráfico intersecta o eixo vertical no ponto (0, 3); por outro lado, o coeficiente linear da função
y = −x + 4 é b = 4, o que significa que seu gráfico intersecta o eixo vertical no ponto (0, 4).
Em contraste, as funções y = −3x + 3 e y = 3x + 3 têm o mesmo coeficiente linear (b = 3):
portanto, os gráficos das duas funções, as retas t e t0 na figura seguinte, respectivamente, intersectam
o eixo vertical no mesmo ponto, a saber, o ponto (0,3); no entanto, t tem declividade −3 enquanto t0
tem declividade 3. Isso significa que a função y = −3x + 3 é decrescente (com taxa de variação igual
a −3) ao passo que a função y = 3x + 3 é crescente, com taxa de variação igual a 3.

65
Questão 51 Trace, no plano cartesiano representada na questão 50, as retas dadas pelas funções nas
alternativas incorretas.

Solução. Os fatos descritos na observação anterior estão representados nas seguintes figuras do
plano cartesiano.
y
r s
4

−2 −1 0 1 2 3 4 x
−1

−2

y
9

t 7 t0

6 m

−2 −1 0 1 2 3 x
−1

−2

−3

−4

−5

−6

Observação 0.14 Na segunda figura, a reta indicada por m é o gráfico da função afim y = 2x + 1,
uma vez que contém os pontos (0,1) e (3, 5). Observe que as retas m e t0 intersectam-se no ponto
(−2, − 3). De fato, tomando x = −2 e y = −3, as equações
(
y = 2x + 1
y = 3x + 3

66
ou
(
−2x + y = 1
−3x + y = 3

são ambas satisfeitas, pois

−2 · (−2) + (−3) = 1 e − 3 · (−2) + (−3) = 3.

De modo similar, as retas m e t intersectam-se no ponto (2/5, 9/5). De fato, tomando x = 2/5 e
y = 9/5, as equações
(
y = 2x + 1
y = −3x + 3
ou
(
−2x + y = 1
3x + y = 3

são ambas satisfeitas, pois


2 9 5 2 9 15
−2 · + = =1 e 3· + = = 3.
5 5 5 5 5 5


Questão 52 — SPAECE - Item M120881E4. No plano cartesiano abaixo está representado o gráfico de
uma reta r de equação y = ax + b.

y
r

Os valores dos coeficientes angular e linear dessa reta, são, respectivamente,

A) a<0 e b < 0.
B) a<0 e b > 0.
C) a>0 e b > 0.
D) a>0 e b < 0.
E) a>0 e b = 0.

Solução. Observe que o gráfico está positivamente inclinado, isto é, dada uma variação positiva da
variável x, temos que a variação correspondente da variável y é também positiva. Portanto, a razão
entre as variações, ou seja, o coeficiente angular, é positiva. Assim, a > 0.
Observamos, ainda, que o gráfico intersecta o eixo vertical em um ponto (0, b) com b < 0, ou seja, o
coeficiente linear b da função afim é negativo.
Portanto, a alternativa correta é D). 

67
Questão 53 Represente, no plano cartesiano, retas que representam os gráficos das seguintes funções:

1) y = 2x + 1
2) y = 2x − 1
3) y = −2x + 1
4) y = −2x − 1
5) y = 12 x − 2
6) y = x + 21
7) y = − 12 x + 2

Solução. Indicando as retas pelos números no enunciado, temos:


5
(1) (2)

−5 −4 −3 −2 −1 1 2 3 4 5

−1

−2

−3

−4

(4) (3)
−5

(7)
4

−5 −4 −3 −2 −1 1 2 3 4 5

−1

−2

(6) −3

−4
(5)

−5

68
Questão 54 No plano cartesiano abaixo está representado o gráfico de uma reta r de equação
y = ax + b.
y
r

2 x

Os valores dos coeficientes angular e linear dessa reta, são, respectivamente, iguais a

A) a = 2 e b = 1. B) a = 1 e b = 2. C) a = −1/2 e b = 1. D) a = 1/2 e b = 1.

Solução. A reta r na figura contém os pontos (0,1) e (2, 0). Com essa informação, já deduzimos
que o coeficiente linear da função afim cujo gráfico é a reta r é dado por b = 1. Quanto ao coeficiente
angular ou declividade da reta, calculamos a taxa de variação entre as variações das coordenadas y e
x do ponto (0, 1) para o ponto (2, 0), obtendo
0−1 1
a= =− ·
2−0 2

Portanto, a reta r é o gráfico da função afim y = − 12 x + 1, ou seja, é o conjunto de soluções da equação


linear x + 2y = 2. A alternativa correta é C). 

Questão 55 a) Trace, no plano cartesiano representado na questão 54, as retas que correspondem às
funções nas alternativas erradas.
b) O ponto (1, 1/2) pertence a qual dessas retas?
c) O ponto (−1, 1/2) pertence a qual dessas retas?

Solução. a) Indicando as retas correspondentes às funções pelos números no enunciado da questão,


temos:
y

(D) x
−2 −0.5 2

(B) (A)

b) Reta r (com a = −1/2 e b = 1, gráfico de y = − 12 x + 1). c) Reta (D) (com a = 1/2 e b = 1, gráfico
de y = 21 x + 1). 

69
Observação 0.15 Vejamos exemplos com as diferentes possibilidades de sinais dos coeficientes angular
a e linear b em uma função afim da forma

y = ax + b.

Nos casos em que o coeficiente angular é positivo, isto é, quando a > 0, temos inclinação positiva da
reta: ou seja, variações positivas de x correspondem a variações positivas de y. Dito de outro modo,
se a variável x cresce, a variável y também cresce (proporcionalmente, além disso). No exemplo
seguinte, temos a > 0 e b > 0, visto que, além de inclinado positivamente, o gráfico intersecta o eixo
vertical no ponto (0, b) com ordenada positiva.

1 2 3 4 5

Neste exemplo, temos


4
y = x + 1,
5
4
ou seja, a = 5 e b = 1.

Observação 0.16 No exemplo seguinte, temos a > 0 e b < 0, ou seja, o gráfico é também positivamente
inclinado mas, desta vez, intersecta o eixo vertical em um ponto (0, b) com ordenada negativa:

1 2 3 4 5
−1

Neste exemplo, temos


4
y = x − 1,
5
4
ou seja, a = 5 e b = −1.

70
Observação 0.17 No próximo exemplo, o gráfico é negativamente inclinado, ou seja, o coeficiente
angular a é negativo. Isso significa que, dada uma variação positiva de x, a variação correspondente
de y é negativa. Em outros termos, se a variável x cresce, a variável y decresce proporcionalmente.
Vejamos, portanto, um exemplo em que a < 0 mas b > 0, indicando que o gráfico intersecta o eixo
vertical em um ponto acima do eixo horizontal:

1 2 3 4 5

Neste exemplo, temos a função afim dada por


4
y = − x + 4,
5
com a = − 45 e b = 4.

Observação 0.18 Finalmente, apresentamos um exemplo de função afim y = ax + b com a < 0


(gráfico negativamente inclinado) e b < 0 (intersecção do gráfico com o eixo vertical em um ponto
abaixo do eixo horizontal). Temos:

1 2 3 4 5
−1

−2

−3

−4

−5

Nesta figura, representamos o gráfico da função afim


4
y =− x−1
5
com a = − 45 e b = −1. Note que o gráfico é negativamente inclinado (a < 0) e intersecta o eixo
vertical em um ponto abaixo do eixo horizontal com ordenada negativa b < 0.

71
Questão 56 Indique quais das retas no plano cartesiano representado abaixo são gráficos das funções
5 10
i) y = −0,6x + 3 ii) y = 0,6x + 1 iii) y = 1,25x − 1,25 iv) y = −0,6x + 4 v) y = x−
3 3

1 2 3 4 5

Solução. Temos a seguinte correspondência entre retas (gráficos) e funções afins:


i) Reta contendo os pontos (0,3) e (5, 0): declividade (coeficiente angular) dada por
0−3 3
a= = − = −0,6.
5−0 5
Intersecção com o eixo vertical dada por (0, b) = (0,3). Logo, o coeficiente linear é b = 3.
ii) Reta contendo os pontos (0,1) e (5, 4): declividade (coeficiente angular) dada por
4−1 3
a= = = −0,6.
5−0 5
Intersecção com o eixo vertical dada por (0, b) = (0,1). Logo, o coeficiente linear é b = 1.
iii) Reta contendo os pontos (1,0) e (5, 5): declividade (coeficiente angular) dada por
5−0 5
a= = = 1,25.
5−1 4
Intersecção com o eixo vertical dada por (0, b), onde b é dado por
0−b
= 1,25,
1−0
ou seja, b = 1,25.
iii) Reta com mesma declividade (coeficiente) angular da reta em i). No entanto, o coeficiente linear
é 4 em vez de 3. Logo, a reta intersecta o eixo vertical no ponto (0, 4) e é paralela à reta em i).
Portanto, contém os pontos (0, 4) e (5, 1).
iii) Reta contendo os pontos (2,0) e (5, 5): declividade (coeficiente angular) dada por
5−0 5
a= = ·
5−2 3
Intersecção com o eixo vertical dada por (0, b), onde b é dado por
0−b 5
= ,
2−0 3
ou seja, b = − 10
3 .


72
Questão 57 — SPAECE - Item M120881E4. A reta t de equação y = jx + k está representada no gráfico
abaixo.
y

Os coeficientes angular j e linear k, em relação ao sinal, são, respectivamente,

A) negativo e negativo.
B) negativo e nulo.
C) positivo e negativo.
D) positivo e nulo.
E) positivo e positivo.

Solução. Observe que o gráfico está negativamente inclinado, isto é, dada uma variação positiva
da variável x, temos que a variação correspondente da variável y é negativa. Portanto, o coeficiente
angular j é negativo. Em suma, j < 0.
Observamos, ainda, que o gráfico intersecta o eixo vertical na origem, isto é, no ponto (0,0). Logo, o
coeficiente linear k da função afim y = jx + k é nulo, ou seja, k = 0. Portanto, a alternativa correta
é B). 

Questão 58 — SAEPE - Item M120460H6. Observe abaixo um esboço do gráfico da função polinomial
do 1◦ grau f : R → R:
y
1

−2 −1 0 1 2 3x
−1

−2

−3

−4

−5

A lei de formação dessa função f é

A) y = −4x − 4 B) y = −4x + 1 C) y = −4x + 4 D) y = 4x − 4 E) y = 4x + 1

Solução. Observe que o gráfico está positivamente inclinado, isto é, dada uma variação positiva
da variável x, temos que a variação correspondente da variável y é também positiva. Portanto, o
coeficiente angular da função é positivo. Observamos, ainda, que o gráfico intersecta o eixo vertical no
ponto (0, − 4). Logo, o coeficiente linear b da função afim é igual a −4. Para calcular explicitamente,

73
o valor do coeficiente angular, vemos que, quando a variável x varia de x = 0 para x = 1, a variável y
varia de y = −4 para y = 0. Assim, o coeficiente angular a é dado por

variação de y 0 − (−4)
a= = = 4,
variação de x 1−0

Concluímos que a função afim é expressa por

y = 4x − 4.

Portanto, a alternativa correta é D). 

Questão 59 — PAEBES - Item M120180G5, adaptado. Qual é a equação da reta que passa pelos
pontos (−1, 5) e (1,1)?

A) y = −2x − 3 B) y = −2x + 3 C) y = −2x + 9 D) y = − x2 + 3


2
E) y = 2x + 4

Solução. A equação da reta em questão é dada por

y−5 1−5
= ,
x − (−1) 1 − (−1)

ou seja,
4
y − 5 = − (x + 1).
2
Reagrupando os termos, escrevemos
y = −2x + 3.
Logo, y pode ser escrita como função afim de x do seguinte modo:

y = −2x + 3.

Portanto, a alternativa correta é D). 

Questão 60 — SAEPE - Item M120325H6, adaptado. Os coeficientes da equação reduzida de uma reta
r são ambos negativos. Qual das retas abaixo pode representar a reta r?

A) y B) y C) y

r r r

x x x

D) y E) y
r

x x

74
Solução. Nas alternativas A) e D), as retas passam pela origem (0,0) e, portanto, seus coeficientes
lineares são nulos. Nas alternativas C) e E), o gráfico intersecta o eixo vertical em um ponto com
ordenada positiva: logo o coeficiente de y na equação reduzida é positivo em ambos os casos. Na
alternativa B), o gráfico intersecta o eixo horizontal em um ponto com abscissa negativa e o eixo vertical
em um ponto com ordenada negativa: isso significa que os coeficientes das variáveis x e y são ambos
negativos na equação reduzida da reta. Em resumo, a alternativa correta é B). 

Observação 0.19 Recordemos que a equação de uma reta no plano tem a seguinte forma geral

Ax + By = C.

onde A, B e C são constantes e as coordenadas (x, y) especificam pontos dessa reta. No caso em
que a reta não passa pela origem (0,0), temos C 6= 0. Dividindo ambos os lados da equação por C,
obtemos a chamada equação reduzida da reta:
x y
+ = 1,
a b
onde a = C/A e b = C/B, nos casos em que A 6= 0 e B = 6 0 (quando a reta não é vertical e não é
horizontal). Observe que os coeficientes a e b têm a seguinte interpretação: os pontos (a, 0) e (0, b)
são os pontos em que a reta intersecta os eixos horizontal e vertical, respectivamente.
Por exemplo, dada a equação da reta 4x + 3y = 24, a equação reduzida é dada por
x y
+ = 1.
6 8
Logo, a reta descrita por essas equações intersecta o eixo horizontal no ponto (6, 0) e o eixo vertical
no ponto (0, 8). Vejamos o gráfico:

0 1 2 3 4 5 6 7 8

Questão 61 — SAEPE - Item M120184H6. A reta k passa pelos pontos de coordenadas (0, 1) e (3, 2).
Qual é a equação reduzida dessa reta?
x
A) y = −3x − 1 B) y = 3 +1 C) y = x + 3 D) y = 3x + 1 E) y = 3x + 2

Solução. A equação da reta em questão é dada por

y−1 2−1
= ,
x−0 3−0

75
ou seja,
1
y − 1 = x.
3
Reagrupando os termos, escrevemos y como função afim de x do seguinte modo:
1
y = x + 1.
3
A seguinte figura ilustra o gráfico dessa função:
3

−3 −2 −1 1 2 3
−1

Portanto, a alternativa correta é B). 

Questão 62 — SAEPE - Item M120414H6. A reta t passa pelos pontos (4, 8) e (2, −6). Qual é a equação
dessa reta t?

A) y = 7x − 20 B) y = −7x−52 C) y = 6x + 2 D) y = 4x + 8 E) y = 2x − 6

Solução. A equação da reta em questão é dada por

y−4 −6 − 8 −14
= = = 7,
x−8 2−4 −2
ou seja,
y − 8 = 7(x − 4).
Reagrupando os termos, escrevemos y como função afim de x do seguinte modo:

y = 7x − 20.

Portanto, a alternativa correta é A). 

Questão 63 — SAEGO - Item M120496A9. A reta s passa pelos pontos (3, 0) e (0, −6). A equação
dessa reta é

A) y = −6x + 3 B) y = −2x − 6 C) y = 2x − 6 D) y = 3x − 6 E) y = 6x + 3

Solução. A reta é gráfico de uma função afim da forma

y = ax + b,

onde os coeficientes a e b são constantes que devemos determinar a partir da informação de que os
pontos (4,1) e (2, 3) pertencem à reta. Assim, fixando x = 4 e y = 1, temos a seguinte equação linear
em a e b:
4a + b = 1.

76
Da mesma forma, tomando x = 2 e y = 3, temos uma segunda equação linear em a e b:
2a + b = 3.
Desse modo, obtemos o sistema de equações lineares:
(
4a + b = 1
2a + b = 3
Subtraindo os lados esquerdos e os lados direitos dessas duas equações, temos
2a = −2,
ou seja, a = −1. Substituindo esse valor de a na primeira equação, por exemplo, temos
−4 + b = 1,
do que concluímos que
b = 5.
Logo, a equação da reta em questão é dada por
y = −x + 5.
A seguinte figura ilustra o gráfico dessa função:
5

1 2 3 4 5

Portanto, a alternativa correta é A). 

Questão 64 — SPAECE - Item M120283H6. Dois pontos pertencentes à reta r são (−2, 3) e (1, 6). Qual
é a equação dessa reta?

A) y = −3x − 3 B) y = −2x + 3 C) y = x + 5 D) y = x + 6 E) y = 2x + 2

Solução. A equação da reta que desejamos encontrar é dada por


y−3 6−3
= ,
x − (−2) 1 − (−2)
ou seja,
y − 3 = x + 2.
Reagrupando os termos, escrevemos y como função afim de x do seguinte modo:
y = x + 5.
Portanto, a alternativa correta é C). 

77
Questão 65 — SPAECE - Item M120703H6. Considere uma reta r que passa pelos pontos de coordena-
das (−2, 3) e (4, − 2). Uma equação dessa reta r está representada em

A) x + 2y + 16 = 0
B) x + 6y − 16 = 0
C) 2x − 3y + 6 = 0
D) 5x + 6y − 8 = 0
E) 3x + 5y − 3 = 0

Solução. A equação da reta que desejamos encontrar tem a seguinte forma

Ax + By = C,

onde A, B e C são constantes e (x, y) são as coordenadas de um ponto arbitrário da reta. Fixando
x = −2 e y = 3, obtemos a equação linear em A e B:

−2A + 3B = C.

Agora, tomando x = 4 e y = −2, temos uma segunda equação linear em A e B:

4A − 2B = C.

Concluímos que
−2A + 3B = 4A − 2B,
ou seja, que
5B = 6A.
Portanto
6
B = A.
5
Substituindo essa expressão para B na primeira equação linear, por exemplo, temos
18
−2A + A = C.
5
Portanto
8
C = A.
5
Assim, a equação linear da reta tem a forma
6 8
Ax + Ay = A.
5 5
Dividindo ambos os lados da equação por A e multiplicando-os por 5, obtemos

5x + 6y = 8,

Portanto, a alternativa correta é D). 

Observação 0.20 A inspeção direta das alternativas pode ajudar a descobrir a resposta correta. Por
exemplo, substituindo os dados x = 4 e y = −2 nas equações escritas nas alternativas A), B) e C),
temos, respectivamente,
4 + 2 · (−2) + 16 = 16 6= 0,
4 − 6 · (−2) + 16 = 32 6= 0
e
2 · 4 − 3 · (−2) + 6 = 20 6= 0.

78
Da mesma forma, substituindo esses valores na equação escrita na alternativa E), obtemos

3 · 4 + 5 · (−2) − 3 = −1 6= 0.

A partir daqui, estudamos funções quadráticas.


Questão 66 Considerando a figura da questão 46, determine as áreas das seguintes figuras:

1) A área do trapézio cujos vértices são os pontos P , A, (5,0) e (0,0).


2) A área do trapézio cujos vértices são os pontos P , B, (10,0) e (0,0).
3) A área do trapézio cujos vértices são os pontos P , Q, (30,0) e (0,0).
4) A área do trapézio cujos vértices são os pontos R, S, (30,0) e (0,0).
5) Se (x, y) é um ponto do gráfico da função f (x) = −300x + 21 000, a área do trapézio cujos
vértices são os pontos P , (x, y), (x, 0) e (0,0).
6) Se (x, y) é um ponto do gráfico da função g(x) = −100x + 9 000, a área do trapézio cujos
vértices são os pontos R, (x, y), (x, 0) e (0,0).
7) A área do quadrilátero cujos vértices são os pontos P, Q, S e R.
8) A área do triângulo formado pelos pontos P , R e pelo ponto de intersecção dos dois gráficos.

Note que, se U = (x, y) é um ponto do gráfico da função y = f (x) = −300x + 21 000, o trapézio
definido pelos pontos P = (0, 21 000), U = (x, y), V = (x, 0) e O = (0, 0) pode ser representado na
seguinte figura.

P
21 000

U = (x,y)
W = (0,y)

T’
T
O = (0,0) V = (x,0)

A linha pontilhada divide o trapézio em duas partes disjuntas, isto é, sem pontos em comum além da
própria linha. Uma das partes (T ) é um triângulo com base U V e altura OV . A base U V mede y
unidades de comprimento e a altura OV mede x unidades de comprimento. Logo, a área de T é dada
por
x·y
,
2
ao passo que T 0 é um triângulo com base OP (medindo 21 000 unidades de comprimento) e altura
U W (medindo x unidades de comprimento). Assim, a área de T 0 é igual a
x · 21 000
.
2
Somando as áreas de T e T 0 , deduzimos que a área do trapézio é igual a
x · y x · 21 000 y + 21 000
+ =x· ,
2 2 2

79
ou seja, é dada pelo produto da medida da altura OV (isto é, x) pela média aritmética das medidas
das bases U V e OP (respectivamente, y e 21 000). Por fim, observe que U = (x, y) é um ponto do
gráfico da função f . Portanto, y = −300x + 21 000. Assim, a área do trapézio é dada pela expressão
−300x + 21 000 + 21 000
x· = −150x2 + 21 000x.
2
Concluímos que essa área define a função quadrática

A(x) = −150x2 + 21 000x,

ou, fatorando-se a expressão algébrica do lado direito,

A(x) = 150 · x · (140 − x).

Solução. 1) O ponto A corresponde a x = 5. Logo, a área do trapézio, neste caso, é

A(5) = 150 · 5 · 135 = 101 250.

2) O ponto B corresponde a x = 10. Logo, a área do trapézio, neste caso, é

A(5) = 150 · 10 · 130 = 195 000.

3) O ponto Q corresponde a x = 30. Logo, a área do trapézio, neste caso, é

A(5) = 150 · 30 · 110 = 495 000.

4) O trapézio determinado pelos pontos R = (0, 9 000), S = (30, 6 000), (30, 0) e (0, 0) pode ser decomposto
em dois triângulos, um com base medindo 9 000 unidades de comprimento e altura, perpendicular e
essa base, medindo 30 unidades de comprimento; o outro triângulo tem base medindo 6 000 unidades de
comprimento e altura de 30 unidades de comprimento. Assim, a área do trapézio é igual a
1 1
· 9 000 · 30 + · 6 000 · 30 = 15 000 · 15 = 225 000.
2 2

5) O trapézio determinado pelos pontos P = (0, 21 000), U = (x, f (x)), V = (x, 0) e (0, 0) pode
ser decomposto em dois triângulos, um com base medindo 21 000 unidades de comprimento e altura,
perpendicular e essa base, medindo x > 0 unidades de comprimento; o outro triângulo tem base medindo
f (x) unidades de comprimento e altura de x unidades de comprimento. Assim, a área do trapézio é
igual a
1 1 21 000 + f (x)
· 21 000 · x + · f (x) · x = x · ·
2 2 2
Como f (x) = −300x + 21 000, concluímos que a área desse trapézio, para um dado valor de x (com
0 < x < 70), é dada pelo valor da função quadrática
21 000 + 21 000 − 300x 
A(x) = x · = x · 21 000 − 150x .
2

6) O trapézio determinado pelos pontos R = (0, 9 000), U = (x, g(x)), V = (x, 0) e (0, 0) é dada por
1 1 9 000 + g(x)
· 9 000 · x + · g(x) · x = x · ·
2 2 2
Como g(x) = −100x + 9 000, concluímos que a área desse trapézio, para um dado valor de x (com
0 < x < 90), é dada pelo valor da função quadrática
9 000 + 9 000 − 100x 
A(x)
e =x· = x · 9 000 − 50x .
2

80
7) A área do quadrilátero determinado pelos pontos P = (0, 21 000), Q = (30, 12 000), R = (0, 9 000)
e S = (30, 6 000) é dada pela diferença das áreas dos trapézios calculadas nas questões 5) e 6) acima.
Assim, temos
.
P (x) = A(x) − A(x)
e
 
= x · 21 000 − 150x − x · 9 000 − 50x

= x · 12 000 − 100x .

8) De acordo com a resolução da questão (48), o triângulo tem base dada por 21 000 − 9 000 = 12 000
unidades a altura com medida igual a 60 unidades. Assim, a área desse triângulo é igual a
12 000 · 60
= 6 000 · 60 = 360 000
2
unidades de área.


Questão 67 A seguinte figura mostra o gráfico da função quadrática A(x) = −150x2 + 21 000x, onde
x varia de 0 a 140.
V
M

O 70 P

O ponto P = (x, 0) é um zero da função A. Qual o valor da coordenada x?

A) 0 B) 35 C) 140 D) 150

Solução. Sabemos que o máximo ou mínimo de uma função quadrática ocorre no ponto médio
entre os zeros ou raízes da função. Observamos que o valor máximo da função ocorre quando x = 70.
Como um dos zeros da função ocorre quando x = 0, o outro zero ocorre quando x = 2 · 70 = 140. A
alternativa correta é C).

Uma abordagem mais algébrica usa diretamente a expressão da função quadrática

A(x) = −150x2 + 21 000x


= 150 · x · (140 − x).

Observamos, usando essa expressão, que A(x) = 0 se x = 0 ou se x = 140. Portanto, os zeros da


função A são x = 0 e x = 140.

Questão 68 Considere o gráfico da função A(x) = −150x2 + 21 000x na figura da questão 67. O valor
dessa função quando x = 20 é igual a

A) 18 000. B) 36 000. C) 180 000. D) 360 000.

Solução. Temos A(20) = 150 · 20 · (140 − 20) = 150 · 20 · 120 = 3 000 · 120 = 360 000. A alternativa
correta é D). 

81
Questão 69 Considere o gráfico da função A(x) = −150x2 + 21 000x na figura da questão 67. Essa
função atinge valor máximo M quando x = 70. Qual o valor de M ?

A) 10 500 B) 21 000 C) 140 000 D) 735 000

Solução. Temos A(70) = 150 · 70 · (140 − 70) = 150 · 70 · 70 = 150 · 4 900 = 735 000. A alternativa
correta é D). 

Questão 70 Considere o gráfico da função A(x) = −150x2 + 21 000x na figura da questão 67. Com
respeito a essa função, é correto afirmar que

A) é crescente no intervalo [0, 140].


B) é crescente no intervalo [0,70].
C) é decrescente no intervalo [0,70].
D) é decrescente no intervalo [0,140].

Solução. A função A é crescente de x = 0 (onde tem um zero) a x = 70 (onde atinge valor máximo).
Em seguida, decresce no intervalo de x = 70 a x = 140 (onde ocorre o segundo zero da função). Logo, a
alternativa correta é B). 

Questão 71 — SAEPE - Item M100070C2. O volume (V ) em um reservatório de água varia em função


do tempo (t) em horas, conforme representado no gráfico da função quadrática abaixo.

De acordo com esse gráfico, em quantas horas esse reservatório atinge seu volume máximo?

A) 6 B) 12 C) 24 D) 144 E) 288

Solução. O valor máximo da função volume ocorre no ponto médio entre os zeros da função, que
são x = 0 e x = 24, ou seja, em
0 + 24
= 12 horas.
2
Portanto, a alternativa correta é B). 

Questão 72 Considere a função, definida para x ∈ [0, 24], cujo gráfico está representado na questão
71. Responda, então, as seguintes perguntas: a) Quais os zeros da função? b) Em que intervalo essa
função é crescente? c) Se o volume máximo é igual a 14 400 litros, qual a expressão algébrica da
função?

Solução. a) O gráfico intersecta o eixo horizontal quando x = 0 e x = 24. Isso significa que a
função se anula para esses valores da variável x. b) A função atinge um valor máximo quando
0 + 24
x= = 12,
2

82
a média aritmética dos zeros da função. Logo, a função é crescente no intervalo [0, 12] e é decrescente
no intervalo [12, 24]. c) Dados os zeros da função, podemos escrevê-la como f (x) = ax(x − 24). Sabendo
que f (12) = 14 400, temos −a · 12 · 12 = 14 400, ou seja, a = −100. 

Questão 73 Na seguinte figura, representamos uma parábola, gráfico da função quadrática f (x) =
ax2 + bx + c, definida no intervalo [−3, 7].
3

−3 P −1 1 2 3 4 5 Q 7
−1

−2

−3

−4
V

Responda às seguintes questões.

1) Para quais valores da variável x a função f se anula, ou seja, quais são os zeros de f ?
2) Quais as coordenadas dos pontos P e Q?
3) Em que intervalo a função f é crescente?
4) Em que intervalo a função f é decrescente?
5) Qual o valor mínimo da função f ?
6) Quais as coordenadas do ponto V (o vértice da parábola)?
7) Quais são os coeficientes a, b e c na expressão algébrica da função f ?
8) Para quais valores da variável x temos f (x) = 2?
9) Escrevendo-se a função f na forma fatorada f (x) = A(x − r)(x − s), quais são os valores de
A, r e s?

Solução. 1) e 2) Os zeros (ou raízes) da função f ocorrem quando x = −2 e x = 6. Portanto,


temos P = (−2, 0) e Q = (6, 0).
3) A função f é crescente no intervalo de x = 2 a x = 7, ou seja, [2, 7].
4) A função f é decrescente no intervalo de x = −3 a x = 2, ou seja, [−3, 2].
5) A função f atinge valor mínimo quando x = 2. Esse valor mínimo, de acordo com o gráfico, é y = −4.
6) O ponto V é o vértice da parábola: neste caso, é o ponto em que a ordenada y atinge o valor mínimo.
Logo, V = (2, −4).
7) Propomos duas abordagens para resolver o problema: na primeira, dada a forma geral
f (x) = ax2 + bx + c
de uma função quadrática, determinamos um sistema linear a ser satisfeito pelos coeficientes a, b e c ao
substituirmos as informações que temos sobre zeros e valor mínimo da função, ou seja, ao considerarmos
que
f (−2) = 0, f (2) = −4, f (6) = 0.
Temos



 4a − 2b + c = 0
4a + 2b + c = −4


 36a + 6b + c = 0.

83
Assim, a partir da primeira e segunda equações, obtemos

8a + 2c = −4,

ou seja,
c = −4a − 2.
Substituindo esse dado na terceira equação, temos

0 = 36a + 6b − 4a − 2 = 32a + 6b − 2.

Deduzimos que
1 16
b= − a.
3 3
Por fim, substituindo essas expressões para b e c na primeira equação do sistema, por exemplo, obtemos
2 32 32 8
0 = 4a − + a − 4a − 2 = a − ·
3 3 3 3
Com isso, concluímos que
1
a=
4
e, portanto,
1 4
b= − = −1 e c = −1 − 2 = −3.
3 3
Deduzimos, assim, que a função quadrática cujo gráfico é representado na figura tem a seguinte expressão
algébrica:
1
f (x) = x2 − x − 3.
4

Adotando um procedimento mais simples e conceitual, consideramos que, como −2 e 6 são os dois
zeros da função quadrática, os monômios x − (−2) = x + 2 e x − 6 são fatores do polinômio f (x), ou
seja, f (x) tem a forma
f (x) = a(x + 2)(x − 6).
Resta, então, determinar o valor do coeficiente a. Para tanto, usamos, agora, o fato de que f (2) = −4:
de fato, temos
−4 = f (2) = a · (2 + 2) · (2 − 6) = −16a.
Portanto,
−4 1
a= = ·
−16 4
Concluímos que
1
f (x) = (x + 2)(x − 6)
4
1 2
= (x − 6x + 2x − 12)
4
1 2
= x − x − 3.
4

8) Usando a expressão algébrica de f , temos f (x) = 2 se, e somente se,


1 2
x − x − 3 = 2,
4
ou seja, se e somente se,
x2 − 4x − 14 = 0.

84
Completando quadrados na equação, temos

x2 − 4x + 4 − 18 = 0,

ou seja,
(x − 2)2 = 18.
Temos, assim, √ √
x=2− 18 ou x=2+ 18.
Observamos, pelo gráfico na figura, que equações quadráticas da forma f (x) = d têm (duas) soluções
(ou raízes) se, e somente se, d > −4. Quando d = −4, há a única solução x = 2 e, caso d < −4, não
há soluções reais da equação. Isso pode ser explicado algebricamente, utlizando-se completamento de
quadrados: observamos que f (x) = d se, e somente se,
1 1 1 1
d = x2 − x − 3 = (x2 − 4x − 12) = (x2 − 4x + 4 − 16) = (x − 2)2 − 4,
4 4 4 4
ou seja, se e somente se,
1
(x − 2)2 = 4 + d ≥ 0.
4
9) Vimos, anteriormente, que a expressão algébrica de f pode ser escrita na forma fatorada f (x) =
1
4 (x + 2)(x − 6). Logo, temos a = 1/4, r = −2 e s = 6.


Na questão 73, lidamos com alguns fatos importantes sobre funções quadráticas ou funções
polinomiais de segundo grau. Os zeros ou raízes de uma função quadrática f (x) = ax2 + bx + c são
os valores da variável x para os quais f (x) = 0. No caso em que f tem dois zeros, digamos, x = r e
x = s, temos

f (x) = a(x − r)(x − s) = a(x2 − rx − sx + rs)


= ax2 − a(r + s)x + ars.

Portanto r + s = −b/a e rs = c/a. Além disso, a função atinge valor máximo (se a < 0) ou valor
mínimo (se a > 0) quando x = xV , onde
r+s
xV = ,
2
ou seja, quando x é igual a média aritmética dos zeros da função. Escrevendo
 b b2  b2  b 2 ∆
ax2 + bx + c = a x2 + 2 x+ 2 − +c=a x+ − ,
2a 4a 4a 2a 4a
onde ∆ = b2 − 4ac, vemos que os zeros ou raízes de f são dados, no caso em que ∆ ≥ 0, por
√ √
−b ∆ −b ∆
r= − e s= + ·
2a 2a 2a 2a
Logo,
−b
xV =
2a
e o valor máximo (ou mínimo) da função f é dado quando


yV = f (xV ) = −
4a

85
Sendo assim, o vértice da parábola, gráfico da função f , tem coordenadas V = (−b/2a, −∆/4a). Além
disso, a reta vertical que passa por V é um eixo de simetria dessa parábola.

Questão 74 — SAEPI - Item M120258ES. O gráfico abaixo representa uma função de domínio [−3, 3].

As raízes dessa função são

A) −2, −4 e 2. B) −2 e −4. C) −2 e 2. D) −2, 0 e 2. E) −4 e 0.

Solução. Note que o gráfico intersecta o eixo horizontal (eixo das abscissas) nos pontos em que
x = −2 e x = 2. Logo, a alternativa correta é C). Observamos que, com as informações dadas no gráfico,
podemos deduzir que a expressão algébrica da função é
y = a(x − (−2))(x − 2),
para uma constante a a ser determinada. Usando o fato de que y = −4 se x = 0, temos
−4 = a(0 + 2)(0 − 2) = −4a.
Logo a = 1 e, portanto, a expressão da função é
y = (x + 2)(x − 2) = x2 − 4.


Questão 75 — PUC-RS, 2019. O goleiro de um time de futebol deu um chute, e a bola realizou uma
trajetória que pode ser modelada pela expressão S(t) = at2 + bt + c, sendo S a altura alcançada pela
bola e medida em metros (m) e t o tempo medido em segundos (s). Se S(3) = S(6), então a bola
atingiu sua altura máxima em

A) 6,0 s B) 5,5 s C) 4,5 s D) 4,0 s

Solução. Uma solução simples consiste em observar que, como S(3) = S(6) = d, para uma certa
constante d, a função modificada H(t) = S(t) − d = at2 + bt + c − d tem zeros em t = 3 e em t = 6.
Logo, o valor máximo dessa função (e, portanto, da função S) ocorre no ponto médio entre os zeros, ou
seja, ocorre quando
3+6
t= = 4,5
2
segundos, o que corresponde à alternativa C). 

86
Observação 0.21 Com os dados do enunciado, é possível deduzir mais informações a respeito da
função S. Usando o fato de que t = 3 e t = 6 são zeros da função auxiliar H(t) = S(t) − d, temos

H(t) = a(t − 3)(t − 6).

Expandindo essa forma fatorada da função H, temos a seguinte equivalência de polinômios:


b c d
t2 − 9t + 18 = t2 + t + − ·
a a a
Logo, obtemos as seguintes relações:
b c−d
= −9 e = 18.
a a
Se conhecêssemos o valor máximo da altura, por exemplo, se S(4,5) = 9, teríamos

a · 1,5 · 1,5 = 9.

Nesse caso, obteríamos


9
a= =4
2,25
e, portanto,
b = −36 e c − d = 72.
De fato, a função H seria, nesse caso,

H(t) = 4(t − 3)(t − 6).

Restaria, então, determinar o parâmetro d: sabendo o valor de S(3) (ou S(6)), teríamos esse valor e
a expressão completa de S.

Questão 76 — UNICHRISTUS 2018. Uma empresa que comercializa diversos tipos de chocolate fez um
levantamento e detectou que o valor diário arrecadado para vender suas barras de chocolate especial
(com uma textura mais cremosa) é dado pela função V (x) = 18x − 0,6x2 , em que V (x) é o valor
diário, em reais, arrecadado com a venda das barras de chocolate especial e x é o número de barras de
chocolate especial que foram vendidas em um dia. Para que o valor diário arrecadado seja máximo, o
número de barras de chocolate especial que devem ser vendidas em um dia deve ser igual a

A) 10. B) 12. C) 15. D) 18. E) 20.

Solução. Para determinarmos os zeros da função V , observamos que podemos escrever essa função
na seguinte forma fatorada
18
 
V (x) = 0,6x − x = 0,6x(30 − x).
0,6
Com isso, determinamos, imediatamente, que seus zeros são x = 0 e x = 30. Logo, o valor máximo dessa
função ocorre no ponto médio entre os zeros, ou seja, quando
0 + 30
x= = 15.
2
Logo, a resposta correta é a alternativa C). 

87
Questão 77 — ENEM 2016, Caderno Azul, Questão 157, adaptada. Para evitar uma epidemia, a
Secretaria de Saúde de uma cidade dedetizou todos os bairros, de modo a evitar a proliferação do
mosquito da dengue. Sabe-se que o número f de infectados é dado pela função f (x) = −2x2 + 120x
(em que x é expresso em dia e x = 0 é o dia anterior à primeira infecção) e que tal expressão é válida
para os 60 primeiros dias da epidemia. A Secretaria de Saúde decidiu que uma segunda dedetização
deveria ser feita no dia em que o número de infectados chegasse à marca de 1 600 pessoas, e uma
segunda dedetização precisou acontecer. A segunda dedetização começou no

A) 19o dia. B) 20o dia. C) 29o dia. D) 30o dia. E) 60o dia.

Solução. Devemos encontrar o dia x em que o número de infectados, ou seja, f (x), é igual a 1 600
pessoas. Isto significa que x é solução da equação quadrática
f (x) = 1 600
ou
−2x2 + 120x = 1 600.
Subtraindo 1 600 dos dois lados da equação, temos
−2x2 + 120x − 1 600 = 0.
Dividindo os dois lados desta equação por −2, temos
x2 − 60x + 800 = 0.
Ora, a expressão quadrática do lado esquerdo da equação pode ser fatorada, ou seja, escrita na forma de
um produto como
(x − 20)(x − 40) = 0.
Uma vez que o produto é igual a zero, pelo menos um de seus fatores deve ser também igual a zero.
Logo, ou
x − 20 = 0 ou x − 40 = 0,
o que implica que ou x = 20 ou x = 40. Isto significa que o número de infectados chega a 1 600 no
vigésimo (20o ) dia e volta a ser igual a 1 600 no quadragésimo (40o ) dia. A resposta correta é a da
alternativa B). 

A equação quadrática (ou de segundo grau)

x2 − 60x + 800 = 0

poderia ser também resolvida por completamento de quadrados: lembrando que 60 = 2 · 30, teríamos

x2 − 2 · 30x + 800 = 0.

Lembremos também da expressão da diferença de quadrados:

a2 − 2ab + b2 = (a − b)2 ,

onde a, b são números ou variáveis reais. Em nosso caso, a = x e b = 30. Assim

0 = x2 − 2 · 30x + 800 = x2 − 2 · 30x + 800 + 302 − 302


= x2 − 2 · 30x + 302 + 800 − 302
= (x − 30)2 + 800 − 900
= (x − 30)2 − 100.

88
Logo, a equação quadrática pode ser reescrita como

(x − 30)2 − 100 = 0,

uma equação bem mais simples, cuja solução segue da seguinte forma: somamos 100 dos dois lados da
equação, obtendo
(x − 30)2 = 100.
Calculamos as raízes quadradas dos dois lados, o que resulta em

x − 30 = −10 ou x − 30 = 10,

donde segue que


x = −10 + 30 = 20 ou x = 10 + 30 = 40,
como antes.

Na forma rotineira de resolver uma equação quadrática como

x2 − 60x + 800 = 0,

empregamos a chamada “fórmula de Bhaskara”, fixando a = 1, b = −60 e c = 800:


p
−(−60) ± (−60)2 − 4 · 1 · 800
x=
√ 2×1
60 ± 3 600 − 3 200
=
2
60 ± 20
= ·
2
Logo, r = 20 ou s = 40 são as soluções ou raízes da equação. Na verdade, esta fórmula é uma mera
consequência da técnica de completamento de quadrados que vimos acima. No entanto, é interessante
usar a seguinte abordagem alternativa, elaborada pelo Professor Po-Shen Loh: como a soma das raízes
é igual a
−b
= 60,
a
sua média aritmética é igual a 30. Ora, essa média é o ponto médio entre as raízes da equação. Assim,
consideramos a variável u que define a distância em relação a esse ponto médio. Assim,

r = 30 − u e s = 30 + u.

Logo, o produto das raízes é dado por


c
(30 − u) · (30 + u) = = 800,
a
ou seja,
900 − u2 = 800,
o que implica
u2 = 100.
Portanto, como u deve ser positivo, temos. u = 10. Assim, reobtemos as raízes da equação:

r = 30 − 10 = 20 e s = 30 + 10 = 40.

89
Seção 3. Terceiro percurso: porcentagens, proporções, progressões e gráficos

As tarefas a seguir envolvem conhecimentos prévios fundamentais para desenvolver as habilidades nos
seguintes descritores da Matriz de Referência do SAEB (terceira série do Ensino Médio):

• D16 - Resolver problema que envolva porcentagem.


• D15 - Resolver problema que envolva variações proporcionais, diretas ou inversas, entre grandezas.
• D34 - Resolver problema envolvendo informações apresentadas em tabelas e/ou gráficos.
• D6 - Identificar a localização de pontos no plano cartesiano.
• D7 - Interpretar geometricamente os coeficientes da equação de uma reta.
• D28 - Reconhecer a representação algébrica ou gráfica da função polinomial de 1◦ grau.
• D19 - Resolver situação problema envolvendo uma função de primeiro grau.
• D22 - Resolver problema envolvendo PA/PG, dada a fórmula do termo geral.
• D20 - Analisar crescimento/decrescimento, zeros de funções reais apresentadas em gráficos.

Questão 78 — PAEBES - Item M080552E4. Uma confecção produziu 400 biquínis. Desses biquínis, 50
estão com defeitos. Qual é a porcentagem do total de biquínis produzidos que apresentou defeitos?

A) 8% B) 12,5% C) 50% D) 87,5%

Solução. A razão entre o número de biquínis com defeito e o número total de biquínis é dada por
50 5 5 · 2,5 12,5
= = = = 12,5%.
400 40 40 · 2,5 100
Portanto, a alternativa correta é B). 

Questão 79 — SAEPE - Item M110097H6. Uma empresa aérea realizou uma pesquisa entre seus clientes
para saber quais eram os seus destinos preferidos. Dos 300 clientes entrevistados, 40% optaram pela
Irlanda. Quantos clientes entrevistados preferem a Irlanda como destino?

A) 12 B) 40 C) 120 D) 180 E) 260

Solução. A fração de clientes que optaram pela Irlanda é


40 300
40% · 300 = · 300 = 40 · = 40 · 3 = 120.
100 100
Portanto, a alternativa correta é C). 

Questão 80 — SAEPI - Item 120465ES. A prefeitura de uma cidade adotou a seguinte promoção para
incentivar a arrecadação de IPTU (Imposto Predial Territorial Urbano): “pague com 10% de desconto
até o dia 10 de maio; preço normal de 11 a 31 de maio ou acréscimo de 10% após o dia 1◦ de junho”.
Carla recebeu seu carnê antecipadamente com o preço normal de R$ 350,00 e pagou no dia 10 de
junho. Quanto Carla pagou de IPTU?

A) R$ 385,00 B) R$ 360,00 C) R$ 350,00 D) R$ 340,00 E) R$ 315,00

Solução. Pagamentos após o dia primeiro de junho (no caso de Carla, no dia 10 de junho) têm
acréscimo de 10% sobre o valor principal. Portanto, o valor pago por Carla foi
350 + 10% · 350 = 350 + 35 = 385 reais.
Portanto, a alternativa correta é A). 

90
Questão 81 — SAEPI - Item M090129C2. Em uma determinada cidade de 100 000 habitantes apenas
1
5da população recebeu a vacina contra a gripe A (H1N1) no ano de 2011. Esse número de habitantes
vacinados representa que percentual do total de habitantes dessa cidade?

A) 1% B) 1,5% C) 5% D) 20%

Solução. A fração de habitantes vacinadas é igual a


1 1 · 20 20
= = = 20%
5 5 · 20 100
do total de habitantes, ou seja, que
100 000
20% · 100 000 = 20 · = 20 · 1 000 = 20 000
100
habitantes, o que corresponde à alternativa D). 

Questão 82 — SAEPI - Item M090579ES. Em um teatro, o valor do ingresso é R$ 60,00. No dia


internacional das mulheres, o ingresso feminino nesse teatro teve um desconto de 15%. Qual foi o
valor do ingresso feminino desse teatro nesse dia?

A) R$ 9,00 B) R$ 15,00 C) R$ 45,00 D) R$ 51,00

Solução. O valor do ingresso, após o desconto de 15%, é dado por


15 6 000 − 15 · 60 6 000 − 900 5 100
60 − · 60 = = = = 51,00
100 100 100 100
reais, o que corresponde à alternativa D). 

Questão 83 — SAEPI - Item M120258A8. O salário de Ricardo era de R$ 400,00 e agora é de R$


460,00. Ele recebeu um aumento de

A) 60% B) 50% C) 15% D) 13% E) 10%

Solução. O aumento percentual ou relativo do salário de Ricardo é dado por


460 − 400 60 6 6 · 2,5 15
= = = = = 15%,
400 400 40 40 · 2,5 100

o que corresponde à alternativa C). 

Questão 84 — UECE 2022.1, Segunda Fase. Desejando pintar uma superfície retangular cujas dimen-
sões são 15 m de comprimento e 3,2 m de largura, ao comprar a tinta, verifiquei que uma lata da
tinta de minha escolha custa R$ 12,00 e que, com uma lata de tinta, posso pintar apenas 2,0 m2 da
superfície. Se disponho de apenas R$ 180,00 para comprar tinta, a porcentagem da superfície que
posso pintar é

A) 66,0% B) 58,0% C) 65,5% D) 62,5%

Solução. Essa é uma questão interessante por combinar cálculos de áreas e de porcentagens, Uma
vez que a superfície é retangular, sua área é dada pelo produto

15 · 3,2 = 60 · 0,8 = 48 m2 .

91
Para pintar essa área superficial, são necessárias, portanto,
48
= 24
2
latas de tinta, que representam um custo de
24 · 12 = 144 · 2 = 288
reais. Portanto, com 180 reais, é possível pintar, nessa mesma proporção de tinta por área,
180 180 : 36 5
= = ,
288 288 : 36 8
ou seja,
5 · 125 625
= = 62,5%
8 · 125 1 000
o que corresponde à alternativa D). 

Questão 85 — SAEPE - Item M120442H6. Uma lanchonete vende sucos em copos com capacidade para
500 mL pelo preço de R$ 5,00. Atendendo aos pedidos de clientes, essa lanchonete também passará a
vender seus sucos em copos que comportam 200 mL a mais do que o modelo atual, e o preço desse
novo copo de suco será proporcional ao preço do suco vendido no copo de 500 mL.
De acordo com essas informações, por qual valor essa lanchonete deve vender esse novo copo de suco?

A) R$ 2,00 B) R$ 3,57 C) R$ 5,00 D) R$ 6,25 E) R$ 7,00

Solução. De acordo com o enunciado, 500 mL de suco correspondem a 5 reais. Proporcionalmente,


100 mL correspondem a 1 real. Logo, 200 mL custam 2 reais. Portanto, o preço, mantida essa proporção,
de 500 + 200 mL correspondem a 5 + 2 reais, ou seja, 7 reais. Portanto, a alternativa correta é E). 

Questão 86 — SAEPI - Item M100276E4. Para retirar o entulho de um terreno, 8 máquinas iguais
retiram um total de 24 toneladas de entulho por dia. Para agilizar o trabalho, foram acrescentadas
mais 4 máquinas iguais às anteriores.
Com todas essas máquinas trabalhando ao mesmo tempo, quantas toneladas de entulho serão retiradas
por dia desse terreno?

A) 16 B) 28 C) 32 D) 36 E) 48

Solução. De acordo com o enunciado, 1 máquina com essa produtividade consegue retirar
24
=3
8
toneladas de entulho por dia. Logo 8 + 4 = 12 dessas máquinas conseguem retirar 12 · 3 = 36 toneladas
de entulho por dia. A alternativa correta é D). 

Observação 0.22 Trata-se, agora, de um exemplo de questões bem corriqueiras envolvendo razões
e proporções e, usualmente, associada ao uso de “regras de três”. Propomos que o professor evite
utilizar procedimentos, apenas, sem justificar as razões por que funcionam, É bastante difundido,
por exemplo, descrever relações de proporcionalidade direta ou inversa com o uso de “setas para
baixo” ou “setas para cima” e, em seguida, efetuar produtos diretos ou inversos para determinar-
se um termo desconhecido nessas relações. Esse é, claro, um expediente útil para descrever as
relações de proporcionalidade entre as duas ou mais grandezas envolvidas no problema e não há,
a rigor, nada errado com isso, do ponto de vista procedimental. No entanto, para desenvolver,
no aluno, a compreensão conceitual da noção de proporcionalidade, sugerimos abordar esse e

92
os demais problemas seguintes enfatizando que estamos determinando, nos cálculos, constantes de
proporcionalidade ou taxas de variação. Desse modo, a expectativa é de que os alunos deixem de
perceber os procedimentos como simples “receitas” a serem executadas, compreendam os conceitos de
razões e proporções em termos de equivalência de frações e taxas de variação e, por fim, tornem-se
aptos a mobilizar esses conhecimentos básicos quando se defrontarem com problemas envolvendo
funções afins, progressões aritméticas e cálculo de juros simples, para mencionar alguns dos temas
em que essas noções são basilares.
Ao abordar a questão 86, segundo essas orientações, o professor pode observar que, se 8 máquinas
com mesma produtividade retiram 24 toneladas de entulho, 1 máquina, apenas, retira
24
=3
8
toneladas de entulho. Essa é a taxa de variação associada ao problema. Dada essa taxa, deduzimos
que 4 máquinas a mais, isto é, 12 máquinas conseguem retirar, no mesmo prazo,

3 · 12 = 36

toneladas de entulho, o que corresponde à alternativa D).

Questão 87 Um reservatório tem capacidade para 2 400 litros. Quando está com 800 litros, uma
torneira é aberta e despeja na caixa 20 litros por minuto. O tempo que transcorre do momento em
que a torneira é aberta até o enchimento total da caixa é

A) 160 min. B) 120 min. C) 80 min. D) 60 min. E) 40 min.

Solução. Para encher totalmente a caixa, faltam 2 400 − 800 = 1 600 litros. A vazão da torneira é
de 20 litros por minuto. Com essa taxa, são necessários
1 600
= 80 litros por minuto.
20
A alternativa correta é C). 

Questão 88 — ENEM 2022, Caderno Amarelo, Questão 143. No período de 2005 a 2013, o valor de
venda dos imóveis em uma cidade apresentou alta, o que resultou no aumento dos aluguéis. Os
gráficos apresentam a evolução desses valores, para um mesmo imóvel, no mercado imobiliário dessa
cidade.

93
A rentabilidade do aluguel de um imóvel é calculada pela razão entre o valor mensal de aluguel e o
valor de mercado desse imóvel.
Com base nos dados fornecidos, em que ano a rentabilidade do aluguel foi maior?

A) 2005 B) 2007 C) 2009 D) 2011 E) 2013

Observação 0.23 Na questão anterior, as rentabilidades em 2005, 2007, 2009, 2011 e 2013 são dadas,
respectivamente, pelas seguintes porcentagens:

• Rentabilidade em 2007:
960 96 96 1 0,1
= = · =8· = 8 · 0,1% = 0,8%
120 000 12 000 12 1 000 100
• Rentabilidade em 2011:
1 800 18 18 1 2 1 4 1
= = · = · = · = 0,4%
450 000 4 500 45 100 5 100 10 100
• Rentabilidade em 2013:
3 240 324 324 1 0,1
= = · =6· = 6 · 0,1% = 0,6%
540 000 54 000 54 1 000 100
Sendo assim, a rentabilidade diminuiu de 0,8% em 2007 para 0,4% em 2011, ou seja, houve uma
redução de
0,4 − 0,8 0,4 1 50
= = = = 50%
0,8 0,8 2 100
na rentabilidade, nesse período. Já de 2011 para 2013, a rentabilidade voltou a aumentar, chegando
a 0,6% em 2013. Isso significa um aumento percentual igual a
0,6 − 0,4 0,2 50
= = = 50%.
0,4 0,4 100

Em resumo, a rentabilidade diminuiu 50% de 2007 a 2011 e aumentou 50% de 2011 a 2013.

Questão 89 — ENEM 2020, Caderno Amarelo, Questão 174. O quadro representa os gastos mensais,
em real, de uma família com internet, mensalidade escolar e mesada do filho.

Internet Mensalidade escolar Mesada do filho


120 700 400

No início do ano, a internet e a mensalidade escolar tiveram acréscimos, respectivamente, de 20% e


10%. Necessitando manter o valor da despesa mensal total com os itens citados, a família reduzirá a
mesada do filho. Qual será a porcentagem da redução da mesada?

A) 15,0 % B) 23,5 % C) 30,0 % D) 70,0 % E) 76,5 %

Solução. A despesa mensal total, antes dos aumentos, é

120 + 700 + 400 = 1 220.

Com os aumentos, a internet e a mensalidade escolar aumentaram, respectivamente,

20% · 120 = 24

94
reais e
10% · 700 = 70
reais. Logo, é preciso reduzir, 24 + 70 = 94 reais na mesada, para que seja mantido o gasto mensal. Essa
redução, em termos de porcentagens, é igual a
94 100
≈ = 25%.
400 400
A resposta exata é 23,5%, o que corresponde à alternativa B). 

Questão 90 — ENEM 2022, Caderno Amarelo, Questão 149. Em janeiro de 2013, foram declaradas
1 794 272 admissões e 1 765 372 desligamentos no Brasil, ou seja, foram criadas 28 900 vagas de
emprego, segundo dados do Cadastro Geral de Empregados e Desempregados (Caged), divulgados
pelo Ministério do Trabalho e Emprego (MTE). Segundo o Caged, o número de vagas criadas em
janeiro de 2013 sofreu uma queda de 75%, quando comparado com o mesmo período de 2012.

Disponível em: http://portal.mte.gov.br. Acesso em: 23 fev. 2013 (adaptado)

De acordo com as informações dadas, o número de vagas criadas em janeiro de 2012 foi

A) 16 514. B) 86 700. C) 115 600. D) 441 343. E) 448 568.

Solução. A queda de 75%, informada no enunciado, significa que o número de vagas criadas
diminuiu 75% de janeiro de 2012 para janeiro de 2013. Logo, o número de vagas criadas em janeiro de
2023 foi apenas 25% do número de vagas criadas em janeiro de 2012. Como 25% = 1/4, o número de
vagas criadas em janeiro de 2012 foi 4 vezes maior que o número de vagas criadas em janeiro de 2013,
ou seja, igual a

4 · 28 900 = 4 · 29 000 − 4 · 100 = 116 000 − 400 = 115 600 vagas.

Portanto, a alternativa correta é C). 

Questão 91 Uma pesquisa sobre jogos eletrônicos mostrou que, num grupo de 1 000 jovens de 13
a 19 anos, 80% jogam em plataformas na internet e, dentre esses, 45% jogam em grupos. Qual a
quantidade de jovens nesse grupo que jogam sozinhos?

A) 800 B) 550 C) 440 D) 360 E) 350

Solução. Dos 1 000 jovens, 80% jogam em plataformas, ou seja,

80% · 1 000 = 800

jovens, Desses, 45% jogam em grupos e, portanto, 55% jogam sozinhos, ou seja,

55% · 800 = 440

jovens, do grupo de 1 000, jogam sozinhos, o que equivale a 44% do total de jovens no grupo. Logo, a
alternativa correta é C). 

Questão 92 — UNICAMP 2023, Primeira Fase, questão 45. Um recipiente de 30 litros contém uma
solução de 14 partes de álcool e 1 parte de água. Quantos litros de água devem ser adicionados para
que se tenha uma solução com 70% de álcool?

A) 8 litros. B) 10 litros. C) 12 litros. D) 14 litros.

95
Solução. De acordo com o enunciado, em 14 + 1 = 15 partes da solução havia 14/15 de álcool e
1/15 de água. Em um total de 30 litros, essas razões equivalem a
14
· 30 = 14 · 2 = 28 litros
15
de álcool e
1
· 30 = 2 litros
15
de água. Acrescentando x litros de água a essa solução, a razão entre a quantidade de litros de álcool e
a quantidade total de litros passa a ser 70% se, e somente se
28 70
= ,
x + 30 100
ou seja, se e somente se,
10
x + 30 = · 28 = 40.
7
Portanto, devem ser acrescentados x = 40 − 30 = 10 litros de água à solução. Logo, a alternativa correta
é B). 

Questão 93 — SAEPI - Item M120046ES. Em uma editora, o custo de produção de um livro é composto
por uma parcela fixa de R$ 10,00, acrescido de R$ 0,15 por página. Qual é o custo de produção de
um livro de 200 páginas nessa editora?

A) R$ 30,00 B) R$ 40,00 C) R$ 300,00 D) R$ 2 000,00 E) R$ 2 030,00

Solução. O custo de produção de um livro de 200 páginas, conforme os dados no enunciado, é


15 · 200
10 + 0,15 · 200 = 10 + = 10 + 30 = 40 reais.
100
Logo, a alternativa correta é B). 

Questão 94 — UECE 2023.2, Segunda Fase, adaptado. Um equipamento eletrônico utilizado por
uma indústria tem seu valor monetário continuamente reduzido em função do uso e do surgimento de
novas tecnologias, dentre outros fatores. Se o valor monetário do equipamento decresce linearmente
com o tempo, sabendo-se que foi adquirido há três anos pelo valor de R$ 180 000,00 e que hoje está
avaliado em R$ 135 000,00, é correto afirmar que o valor monetário do equipamento daqui a dois
anos será

A) R$ 105 000,00. B) R$ 115 000,00. C) R$ 108 000,00. D) R$ 112 000,00.

Solução. Conforme o enunciado, o valor do equipamento decresce, ano após ano, a uma taxa
constante. Em 3 anos, o decréscimo foi igual a

180,000 − 135 000 = 45 000 reais.

Portanto, o decréscimo em cada ano é igual a


45 000
= 15 000 reais por ano.
3
Deduzimos, assim, que, daqui a 2 anos, o valor diminuirá

2 · 15 000 = 30 000 reais.

96
Portanto, o valor do equipamento será

135 000 − 30 000 = 105 000 reais,

resposta que corresponde à alternativa A). 

Observamos que o valor V do equipamento pode ser modelado por uma função afim do tempo t
(em anos) da forma
V (t) = 180 000 − 15 000t,
em que o coeficiente angular a = 15 000 é a taxa constante do decréscimo do valor por ano. Isso condiz
com o resultado obtido anteriormente, visto que, decorridos t = 3 + 2 = 5 anos desde o momento
inicial, em que o equipamento valia 180 000 reais, temos

V (5) = 180 000 − 15 000 · 5 = 180 000 − 75 000 = 105 000 reais.

Questão 95 — SAEPE - Item M120014ES. Uma empresa de arquitetura paga o salário de seus funcio-
nários de acordo com a função apresentada no quadro abaixo.

y = 2 230 + 1 100x

Nessa função, y representa o salário mensal pago pela empresa de arquitetura ao profissional e x
é o número de projetos desse funcionário que foram aprovados no mês. Qual foi o salário de um
profissional que teve 3 de seus projetos aprovados em um mês?

A) R$ 2 230,00 B) R$ 3 200,00 C) R$ 3 330,00 D) R$ 5 530,00 E) R$ 9 990,00

Solução. Basta calcularmos o valor da variável dependente y quando o valor da variável


independente x é igual a 3. Temos

y = 2 230 + 1 100 · 3 = 2 230 + 3 300 = 5 530 reais,

resposta que corresponde à alternativa D). 

Questão 96 — SAEPI - Item M120011EX. A Copa do Mundo de Futebol é um torneio realizado a cada
4 anos. A sequência abaixo relaciona os anos em que houve a Copa do Mundo desde a conquista do
primeiro título brasileiro em 1958.

(1958, 1962, 1966, 1970, . . . )

Quantos torneios foram realizados de 1958 até 2014?

A) 13 B) 14 C) 15 D) 56 E) 60

Solução. Sabemos que a Copa do Mundo ocorre a cada 4 anos. Logo, a sequência dos anos em que
ocorre é uma progressão aritmética cuja razão é 4. Observe que, de 1 958 a 2 014, decorreram

2 014 − 1 958 = 14 + 42 = 56

anos, ou seja,
56
= 14
4
intervalos de 4 anos. Isso significa que ocorreram 14 + 1 = 15 copas do mundo nesse intervalo, o que
corresponde à alternativa C). 

97
Empregando a notação usual no estudo de progressões aritméticas, podemos modelar o problema
em termos da determinação do número n de termos da progressão aritmética, considerando que o
primeiro termo desta progressão é a1 = 1 958, o último termo é an = 2 014 e sua razão, como vimos, é
igual a r = 4. Sendo assim, temos
an = a1 + (n − 1) · r,
ou seja,
2 014 = 1 958 + 4 · (n − 1).
Logo,
2 014 − 1 958 56
n−1= = = 14,
4 4
o que implica n = 15, como havíamos obtido anteriormente.

Questão 97 — ENEM 2011, Caderno 6, Questão 159, adaptada. O número mensal de passagens de
uma determinada empresa aérea aumentou no ano passado nas seguintes condições: em janeiro foram
vendidas 33 000 passagens; em fevereiro, 34 500; em março, 36 000. Esse padrão de crescimento se
mantém para os meses subsequentes.
Quantas passagens foram vendidas por essa empresa em julho do ano passado?

A) 38 000 B) 40 500 C) 41 000 D) 42 000 E) 48 000

Solução. Observe que o aumento do número de passagens de janeiro para fevereiro foi de 34.500 −
33.000 = 1.500 passagens. Já o aumento do número de passagens de fevereiro para março foi de
36.000 − 34.500 = 1.500 passagens. O fato de que o padrão de crescimento se manteve constante
significa que o aumento do número de passagens de um mês para o seguinte foi sempre igual a 1.500.
De março a julho, temos 4 meses. A cada mês, foram vendidas 1.500 passagens a mais. Logo, nos
quatro meses, foram vendidas 4×1.500 = 6.000 passagens a mais. Portanto, em julho, foram vendidas
36.000 + 6.000 = 42.000 passagens. A alternativa correta é letra D). 

Observação 0.24 No exercício anterior, a variável tempo é discreta, pois são os meses do ano, e
podemos identificá-la por uma variável n, que varia de 1 a 12. O número de passagens no mês n é
simbolizado por an . Assim, temos

an = 33 000 + 1 500 · (n − 1)

Em março, por exemplo, que corresponde ao mês n = 3, temos

a3 = 33 000 + 1 500 · (3 − 1) = 33 000 + 3 000 = 36 000 passagens vendidas.

Este é um exemplo de uma progressão aritmética em que o valor da variável em n é dado por

an = a1 + r · (n − 1),

onde a1 é o valor da variável em 1 e r é uma constante chamada razão.

Os conceitos de funções afins, acréscimos simples e progressões aritméticas são naturalmente relacio-
nados entre si. Vimos, acima, que acréscimos simples, a uma taxa i, sobre um valor inicial C, definem
o montante M como função do tempo t dada por

M (t) = C + Cit.

98
Note que, tomando t = 1, 2, 3 . . ., definimos uma progressão aritmética de montantes em que a
razão r é igual a Ci. Assim, denotando r = Ci, temos

M (0) = C,
M (1) = C + Ci = C + r,
M (2) = C + 2Ci = C + 2r,

e assim por diante. O termo geral desta progressão aritmética de montantes é dado pelo montante no
tempo t = n, isto é, por
M (n) = C + Cin = C + rn,
onde r = Ci é o acréscimo (por exemplo, juros) somado a cada iteração.

Observação 0.25 Continuando a discussão sobre a relação entre funções afins, acréscimos simples e
progressões geométricas, dada uma função afim

f (x) = b + ax,

considere valores inteiros da variável x, dados por x = n, onde n = 1, 2, 3, . . . Então

f (n) = b + an.

Essa sequência de valores da função define uma progressão aritmética com termos

f (0) = b,
f (1) = b + a,
f (2) = b + 2a,

e assim por diante. O termo geral desta progressão aritmética é dado pela função f no valor da
variável x = n, isto é, por
f (n) = b + an,
onde r = a é a razão da progressão aritmética e b é seu termo inicial.

1 2 3 4 5

A figura ilustra o gráfico da função afim


4
f (x) = 1 + x,
5

99
em que destacamos os pontos que correspondem aos valores x = n, onde n = 1, 2, 3, 4, 5. Observe que
os valores
4
f (n) = 1 + n
5
formam uma progressão aritmética de razão r = 45 , o que, geometricamente, é representado pelo fato
de que os pontos correspondentes no gráfico estão todos alinhados.

Questão 98 — FGV 2015. A que taxa mensal de juros um capital deve ser aplicado a juros simples,
durante 250 meses, para que quadruplique?

A) 1,4% B) 1,5% C) 1,3% D) 1,6% E) 1,2%

Solução. Aplicando um dado capital C (cujo valor não é relevante para a resolução) à taxa mensal
i%, durante 250 meses, devemos ter 4 vezes esse capital como montante. Em termos algébricos, isso
pode ser escrito como
C + 250 · i% · C = 4C,
ou seja,
250i% = 3.
Portanto,
3 3·4 12
i% = = = = 0,012 = 1,2%.
250 250 · 4 1 000
A alternativa correta é letra E). 

Observação 0.26 Essa questão e algumas outras tratam de acréscimos simples. De fato, envolvem,
mais particularmente, a aplicação de juros simples. Nesse regime de capitalização, o montante
varia proporcionalmente ao intervalo de tempo. Ou seja, temos um crescimento aritmético do
montante.
Por exemplo, em uma situação em que um capital de 350 reais é ajustado a uma taxa de juros
simples de 2%, temos a seguinte progressão aritmética do montante:

350 + 7×4

350 + 7×3

350 + 7×2

350 + 7×1

350 + 7×0

0 1 2 3 4 t

No eixo horizontal, temos os intervalos de tempo, em dias, decorridos desde a data de vencimento
do boleto; no eixo vertical, vemos os valores sucessivos do montante, aumentando linearmente dia
após dia, com um acréscimo de
0,02×350 = 7 reais por dia.
Note que os pontos destacados no plano estão todos alinhados sobre uma reta: essa é a maneira
gráfica de expressar que os valores do montante M crescem em progressão aritmética em função

100
do tempo t, isto é,
M = C + C ×i×t, (15)
a “fórmula” do montante em um regime de juros simples, em que i é a taxa de juros (no exemplo,
aplicada ao dia) e C é o valor ou capital inicial.

Questão 99 — SAEPE - Item M110370H6, adaptado. Em março de 2017, Taís começou a trabalhar
como manicure e comprou 8 vidros de esmalte. Após isso, a cada mês, ela comprou 2 vidros de
esmalte a mais do que havia comprado no mês anterior. Em agosto de 2017, o preço de cada vidro
de esmalte era R$ 3,75. A quantia gasta por Taís, em agosto de 2017, na compra desses vidros de
esmalte foi

A) R$ 37,50. B) R$ 45,00. C) R$ 52,50. D) R$ 67,50. E) R$ 75,00.

Solução. As quantidades de vidros de esmalte comprados a cada mês, de março a agosto de 2017,
formam uma progressão aritmética:

8, 8 + 2 = 10, 8 + 4 = 12 8 + 6 = 14, 8 + 8 = 16, 8 + 10 = 18.

Em agosto de 2017, portanto, Taís comprou 18 vidros de esmalte a um preço unitário de R$ 3,75. Para
tanto, gastou

18×3,75 = 9×2×3,75 = 9×7,5 = 8×7,5 + 7,5 = 4×15 + 7,5 = 60 + 7,5 = 67,50,

ou seja, R$ 67,50, o que corresponde à alternativa D). 

Questão 100 — SAEPE - Item M120382H6. Cláudia foi a um teatro e observou que a distribuição das
cadeiras para a plateia foi feita da seguinte maneira: a primeira fileira, a mais próxima ao palco,
possui 6 assentos, a segunda fileira, 8 assentos e assim sucessivamente, de forma que as quantidades
de assentos em cada fileira seguem uma progressão aritmética. Cláudia sentou-se em uma cadeira
da última fileira dessa plateia, a qual continha 26 assentos. De acordo com essa distribuição, a
quantidade total de cadeiras para a plateia nesse teatro era de

A) 11. B) 40. C) 70. D) 176. E) 289.

Solução. Trata-se de uma progressão aritmética de razão 2, cujos termos inicial e final são,
respectivamente, a1 = 6 e an = 26. Logo, há
26 − 6
+ 1 = 10 + 1 = 11
2
temos na sequência, ou seja, 11 fileiras de cadeiras no teatro. A soma desse total de cadeiras é igual a
6 + 26
. + 22 + 24 + 26} = (6 + 26) · 5 +
6 + 8 + 10 + . .{z = 32 · 5 + 16 = 176
| 2
=11 termos

cadeiras. Assim, a alternativa correta é D). 

Questão 101 — SAEPE - Item M120241G5. Em um parque de diversões cobra-se R$ 12,00 de ingresso
para entrada no parque mais um valor de R$ 1,50 cada vez que o brinquedo for utilizado, conforme
representado na tabela abaixo.

101
Quantidade de brinquedos utilizados Preço a ser pago (em reais)
0 12,00
1 13,50
2 15,00
3 16,50
... ...
10 27,00

A função que melhor expressa a relação entre o valor total a ser pago (P ) e o número de vezes (n)
em que os brinquedos foram utilizados é

A) P = 12,00n.
B) P = 12,00n + 1,50.
C) P = 12,00 + 1,50n.
D) P = 1,50n.
E) P = 13,50n.

Solução. Uma vez mais, temos um contexto que pode ser representado matematicamente em
termos de uma progressão aritmética. O termo inicial (iniciando a contagem com 0 em vez de 1) é 12,00
e a razão desta progressão é 1, 50. Sendo assim, o termo de ordem n é dado por

P = 12 + 1,50n.

Deduzimos, assim, que a alternativa correta é C). 

Nesta próxima sequência, estudaremos o comportamento do gráfico de funções: os intervalos em


que uma função cresce (ou decresce), os pontos em que essa função se anula (os zeros da função, que
ocorrem quando o gráfico da função intersecta o eixo x) e pontos de máximo (ou de mínimo) da função,
quando existirem.
Questão 102 — SAEPE - Item M100098H6, adaptado. Observe abaixo o gráfico de uma função real,
definida no intervalo [−5, 7].

Essa função é estritamente decrescente

A) no intervalo [−5, 2] e no intervalo [0,3].


B) no intervalo [−5, 0].
C) no intervalo [−2, 0] e no intervalo [3,5].
D) no intervalo [1, 4].
E) no intervalo [5, 7].

Solução. A função representada no gráfico é crescente nos intervalos [−5, −2] e [0, 3], é decrescente
nos intervalos [−2, 0] e [3, 5] e constante no intervalo [5, 7]. A alternativa correta é C). 

102
Questão 103 Determine a expressão algébrica da função da questão 102, quando restrita aos intervalos
[−5, −2], [−2, 0], [0, 3], [3, 5] e [5,7].

Solução. A função y = f (x) representada no gráfico é linear por partes, ou seja, é definida por
múltiplas sentenças em diferentes intervalos de seu domínio. Temos

1


 3x + 14
3 , se x ∈ [−5, −2],




 −x + 2, se x ∈ [−2, 0],
1
3 x + 2, se x ∈ [0, 3],
f (x) =

−x + 5, se x ∈ [3, 5],






1, se x ∈ [5, 7],

Observação 0.27 Caro(a) professor(a), utilize o contexto dessas duas últimas questões e da próxima
para explorar funções definidas por várias sentenças; e/ou funções definidas por situações descritas
por textos ou tabelas.

Questão 104 — SAEPE - Item M100344ES. A previsão do tempo para uma cidade brasileira foi noticiada
da seguinte maneira: “Durante a madrugada a temperatura diminuiu, permanecendo constante pela
manhã, mas aumentou no período da tarde”. Qual dos gráficos abaixo melhor representa a situação
descrita nesse texto?

Solução. Considerando que “madrugada” corresponde ao intervalo [0, 6] no eixo das abscissas,
[6, 12] corresponde à “manhã” e “tarde” é correspondente ao intervalo [12,18], a alternativa correta é C).
Note que a variável temperatura “diminui” no intervalo em que essa função é decrescente e “aumenta”
no intervalo em que é crescente. 

103
Questão 105 — PAEBES - Item M110087E4. Observe abaixo o gráfico de uma função y = f (x) definida
no intervalo [−8,8].

Essa função é decrescente no intervalo

A) ]2, 5[ B) ] − 2, 1[ C) ] − 4, 1[ D) ] − 6, 2[ E) ] − 8, −6[

Solução. A função representada no gráfico é constante no intervalo [−8, −6], é crescente nos
intervalos [−6, −2] e [5, 8] e decrescente no intervalo [2, 5]. A alternativa correta é A).


Questão 106 — SAEPI - Item M120982E4, adaptado. Observe abaixo o gráfico de uma função real,
definida no intervalo [0, 7].

Essa função é decrescente em

A) (−0,25, 0,75).
B) (0; 7).
C) (0, 1,5) ∪ (2,5; 4).
D) (1,5; 2,5) ∪ (4,7).
E) (1,5; 4).

Solução. A função representada no gráfico é crescente nos intervalos [0; 1,5] e [2,5; 4] e decrescente
nos intervalos [1,5; 2,5] e [4, 7]. A alternativa correta é D).


Questão 107 — PAEBES - Item M120612H6. Observe abaixo o gráfico da função f : [−6, 8] → [−5, 4].

104
A função f é estritamente crescente

A) no intervalo [5, 8].


B) no intervalo [0,4].
C) no intervalo [−2, 7].
D) no intervalo [−6, 0] e no intervalo [4, 5].
E) no intervalo [−6, −2] e no intervalo [7, 8].

Solução. A função representada no gráfico é constante no intervalo [0, 4], é crescente nos intervalos
[−6, 0] e [4, 5] e decrescente no intervalo [5, 8]. A alternativa correta é D).


Questão 108 — SAEGO - Item M100100H6. Observe abaixo o gráfico de uma função real definida no
intervalo [−1, 4].

Quais são os zeros dessa função

A) −4 e 16.
B) −1, 0 e 4.
C) −1 e 4.
D) 0 e 3.
E) 4 e 16,

Solução. A função representada no gráfico tem zeros em x = 0 e x = 3. A alternativa correta é


D). 

105
Questão 109 — ENEM 2022, Caderno Amarelo, Questão 164. A esperança de vida ao nascer é o
número médio de anos que um indivíduo tende a viver a partir de seu nascimento, considerando
dados da população. No Brasil, esse número vem aumentando consideravelmente, como mostra o
gráfico.

Pode-se observar que a esperança de vida ao nascer em 2012 foi exatamente a média das registradas
nos anos de 2011 e 2013. Suponha que esse fato também ocorreu com a esperança de vida ao nascer
em 2013, em relação às esperanças de vida de 2012 e de 2014.
Caso a suposição feita tenha sido confirmada, a esperança de vida ao nascer no Brasil no ano de
2014 terá sido, em ano, igual a

A) 74,23 B) 74,51 C) 75,07 D) 75,23 E) 78,49

Solução. O gráfico apresentado na questão pode ser considerado como uma linha reta inclinada
com relação ao eixo horizontal. No eixo horizontal, estão indicados os anos de 2008 a 2013. A cada um
desses anos, está associado um ponto no gráfico e um valor da variável esperança de vida ao nascer.
Assim, os pontos no gráfico correspondentes aos anos de 2011, 2012 e 2013 têm as seguintes coordenadas

Ano (x) Esperança de vida ao nascer (y)


2011 73,67
2012 73,95
2013 74,23

Observe que de 2011 a 2012, a variável y passou de 73,67 a 73,95, ou seja, aumentou

73,95 − 73,67 = 0,95 − 0,65 − 0,02 = 0,30 − 0,02 = 0,28.

Já de 2012 a 2013, a variável y passou de 73,95 a 74,23, aumentando

74,23 − 73,95 = 74,28 − 74,00 = 0,28.

De acordo com o enunciado, essa tendência de aumento é mantida de 2013 para 2014. Sendo assim, a
variável y em 2014 deve ser igual a
74,23 + 0,28 = 74,51.
Logo, a resposta correta é B). 

106
Vimos que o aumento da variável y de um ano para o seguinte (a partir de 2011) é sempre igual a
0,28. Assim,

se a variável x aumenta 1, a variável y aumenta 0,28.

Portanto, no ano x após 2011, a variável y será dada por

y = 73,67 + 0,28(x − 2011). (16)

Por exemplo, quando x = 2012, temos

y = 73,67 + 0,28 · (2012 − 2011) = 73,67 + 0,28 = 73,95.

Tomando x = 2013, calculamos

y = 73,67 + 0,28 · (2013 − 2011) = 73,67 + 0,28 · 2 = 73,95 + 0,28 = 74,23.

Finalmente, para x = 2014, obtemos

y = 73,67 + 0,28 · (2014 − 2011) = 73,67 + 0,28 · 3 = 74,23 + 0,28 = 74,51,

como já havíamos calculado. Por fim, se x = 2015, temos

y = 73,67 + 0,28 · (2015 − 2011) = 74,23 + 0,28 · 4 = 74,51 + 0,28 = 74,79,

e assim por diante.


Note que a expressão (16) pode ser escrita como
y − 73,67
= 0,28.
x − 2011
Isso significa que a razão ou quociente entre a variação da variável y, isto é,

y − 73,67,

e a variação correspondente de x, ou seja,

x − 2011,

é sempre constante e igual a 0,28 (de 2011 em diante). Dizemos que 0,28 é a taxa de variação de
y com relação a x. Veremos, mais adiante, que essa taxa de variação é associada à inclinação do
gráfico de y como função de x.

Observação 0.28 — Nota ao professor. Utilize a discussão desta questão para verificar a fluência do
aluno quanto aos seguintes tópicos:

1) a compreensão do texto;
2) leitura de coordenadas;
3) adições e subtrações com números decimais;
4) relação entre multiplicação e divisão (inclusive em expressões algébricas);
5) a noção de variável;
6) a noção de razão expressa como fração, divisão e comparação;
7) a noção de proporcionalidade.

A expressão (16) é um exemplo de função afim ou função de primeiro grau. Em geral, uma

107
variável y é uma função afim de uma variável x se, dados dois valores distintos x0 e x1 da variável x, e
os valores correspondentes y0 e y1 da variável y, então existe uma constante a 6= 0 tal que
y1 − y0
= a,
x1 − x0
ou seja, a taxa de variação de y como função de x é constante, igual a a. Isso significa que a razão
entre a variação de y e a variação correspondente de x é sempre constante e igual a a. Na questão
anterior, tínhamos a = 0,28.

Uma maneira equivalente de modelar o problema posto na questão anterior é a seguinte: perceba que
os valores da variável y na tabela estão em uma progressão aritmética. Isso significa que, para
passar do valor de y em um ano para o valor de y no ano imediatamente seguinte, basta somar a
constante a = 0,28, ou seja,

73,67 + 0,28 = 73,95,


73,95 + 0,28 = 74,23,

e assim por diante. Em geral, tomando 2011 como o ano 1, para calcularmos o valor de y no ano n,
bastaria usar a seguinte expressão

y(n) = 73,67 + 0,28(n − 1) (17)

Dizemos que a constante a (a taxa de variação de y com relação a x) é a razão da progressão


aritmética y(n).

Observação 0.29 — Nota ao professor. Neste ponto, é importante retornar ao enunciado da questão.
Utilize o comentário sobre progressões aritméticas para elaborar a seguinte propriedade fundamental:

em uma progressão aritmética, um termo é dado pela média aritmética de seu antecessor e de seu
sucessor.

Considerando a progressão definida anteriormente, isso significa que

y(n − 1) + y(n + 1)
y(n) = ·
2
Essa propriedade pode ser tomada como a própria definição de progressão aritmética.
No enunciado, afirma-se que o valor de y em 2012 é a média dos valores de y em 2011 e 2013, ou
seja,

y(1) + y(3) 73,67 + 74,23 0,67 + 1,23 1,90


y(2) = = = 73 + = 73 + = 73 + 0,95 = 73,95,
2 2 2 2
exatamente o valor de y no ano 2, ou seja, em 2012. Assim, o valor de y no ano 4, ou seja, em 2014,
é dado por
y(2) + y(4)
y(3) = ,
2
ou seja
73,95 + y(4) = 2 · 74,23.
Assim,
y(4) = 2 · 74,23 − 73,95 = 74,23 + 0,28 = 74,51,
como já sabíamos.

108
Ao trabalhar uma sequência didática sobre funções afins e progressões aritméticas a partir dessa
questão, convém ressaltar o papel dessa propriedade fundamental, utilizando-a para deduzir as
expressões algébricas corriqueiras dessas funções e progressões. No caso de funções afins da forma
y = ax + b, a propriedade pode ser reformulada da seguinte maneira:

y(x − h) + y(x + h)
y(x) = ,
2
ou seja, o valor de y associado a x é igual a média aritmética dos valores de y nos pontos x − h e
x + h à mesma distância h > 0 de x. De fato,

a(x − h) + b + a(x + h) + b
= ax + b = y(x).
2
Finalmente, observe que, considerando os valores de y apenas para valores naturais de x, isto é, para
x = 1, 2, . . . , n, . . ., obtemos uma progressão aritmética, ou seja, os valores

y(n) = b + an = (a + b) + a(n − 1) = y(1) + a(n − 1)

estão em progressão aritmética com razão a. Assim

y(n − 1) + y(n + 1)
= y(n).
2
Para aprofundar esses conceitos, estudemos outra questão do ENEM.
Questão 110 — ENEM 2022, Caderno Amarelo, Questão 180. Uma loja de roupas fixou uma meta
de vendas de 77 000 reais para um determinado mês de 30 dias. O gráfico mostra o volume de vendas
dessa loja, em real, nos dez primeiros dias do mês e entre o dia dez e o dia vinte desse mês, nos
seus dois únicos setores (infantil e adulto). Suponha que a variação no volume de vendas, para o
período registrado, tenha se dado de forma linear, como mostrado no gráfico, e que essa tendência se
mantenha a mesma para os próximos dez dias.

Ao final do trigésimo dia, quanto faltará no volume de vendas, em real, para que a meta fixada para
o mês seja alcançada?

A) 5 000 B) 7 000 C) 11 000 D) 18 000 E) 29 000

Solução. Uma vez mais, a “chave” para resolver a questão está explícita no enunciado. Nesse tipo
de problema, frequente no ENEM, preste atenção às partes relevantes do enunciado, como as seguintes
(onde destacamos as palavras mais importantes):
“Suponha que a variação no volume de vendas, para o período registrado, tenha se dado de forma
linear, como mostrado no gráfico, e que essa tendência se mantenha a mesma (...)”
De acordo com a linha contínua (“adulto”) no gráfico, do dia x0 = 10 para o dia x1 = 20, o volume de
vendas no setor adulto passa de y0 = 18 000 para y1 = 15 000. Assim, a taxa de variação do volume

109
de vendas nesse setor é dada pela razão
y1 − y0 15 000 − 18 000 −3 000
a= = = = −300.
x1 − x0 20 − 10 10
Note que essa taxa de variação é negativa, o que reflete o fato de que o volume de vendas y diminui a
medida que aumentamos o número x de dias. Isso pode ser visto, em termos do gráfico, pela inclinação
negativa da reta que representa o volume de vendas no setor adulto.
Mantida essa tendência, como enunciado na questão, o valor y no dia x = 30 pode ser calculado a
partir da seguinte razão:
y − y1 y − 15 000
= = −300,
x − x1 30 − 20
ou seja,
y − 15 000 = −300 · 10 = −3 000
e, portanto,
y = 15 000 − 3 000 = 12 000 reais.
Esse é o valor, em real, do volume de vendas no setor adulto, acumulado do dia x1 = 20 ao dia x = 30.
Usando as informações do gráfico e os cálculos anteriores, concluímos que
• O volume de vendas no setor adulto acumulado do início do mês ao dia x = 10 é igual a 18 000
reais.
• O volume de vendas no setor adulto acumulado do dia x = 10 ao dia x = 20 é igual a 15 000 reais.
• O volume de vendas no setor adulto acumulado do dia x = 20 ao dia x = 30 é igual a 12 000 reais.
Logo, o volume total de vendas no setor adulto, no mês inteiro, é igual a 18 000 + 15 000 + 12 000 =
30 000 + 15 000 = 45 000 reais.
Agora, façamos cálculos similares a respeito do volume de vendas no setor infantil. Nesse caso, a
taxa de variação pode ser calculada a partir dos dados no gráfico pontilhado (“infantil”). Temos
y1 − y0 7 000 − 8 000 −1 000
a= = =− = −100.
x1 − x0 20 − 10 10
Dada essa taxa de variação, o valor de y quando x = 30 é calculado do seguinte modo:
y − y1 y − 7 000
= = −100,
x − x1 30 − 20
ou seja,
y − 7 000 = −100 · 10 = −1 000.
Assim, o volume de venda no setor infantil, acumulado do dia x1 = 20 ao dia x = 30 é igual a
y = 7 000 − 1 000 = 6 000 reais.
Usando as informações do gráfico e os cálculos anteriores, concluímos que
• O volume de vendas no setor infantil acumulado do início do mês ao dia x = 10 é igual a 8 000
reais.
• O volume de vendas no setor infantil acumulado do dia x = 10 ao dia x = 20 é igual a 7 000 reais.
• O volume de vendas no setor infantil acumulado do dia x = 20 ao dia x = 30 é igual a 6 000 reais.
Logo, o volume total de vendas no setor infantil, no mês inteiro, é igual a 8 000 + 7 000 + 6 000 =
14 000 + 7 000 = 21 000 reais.
Deduzimos que o total de vendas, nos dois setores (adulto e infantil) é igual a
45 000 + 21 000 = 66 000 reais,
faltando, portanto,
77 000 − 66 000 = 11 000 reais
para cumprir a meta almejada de 77 000 reais para o mês. Logo, a alternativa correta é C). 

110
Questão 111 — Baseada na questão 180, Caderno Amarelo, ENEM 2022. Uma loja de roupas fixou
uma meta de vendas no total de 770 000 reais para um determinado mês de 30 dias. O gráfico mostra
o volume de vendas por dia dessa loja, em real, entre o dia dez e o dia vinte desse mês, nos seus
dois únicos setores (infantil e adulto). Suponha que a variação no volume de vendas, para o período
registrado, tenha se dado de forma linear, como mostrado no gráfico, e que essa tendência tenha se
mantido a mesma durante os 30 dias do mês.

Ao final do trigésimo dia, quanto faltará no volume de vendas, em real, para que a meta fixada para
o mês seja alcançada?

Solução. Note que a variável y agora é o volume de vendas por dia. A informação principal no
enunciado continua a ser a seguinte:
“Suponha que a variação no volume de vendas, para o período registrado, tenha se dado de forma
linear, como mostrado no gráfico, e que essa tendência se mantenha a mesma (...)”
Em termos matemáticos, essa frase significa que a taxa de variação do volume de vendas diário (que
representamos pela variável y) se mantém constante em relação ao aumento do número de dias decorridos
desde o início do mês (uma variável que representamos por x). Logo, y é uma função afim de x da forma

y = ax + b.

De acordo com o gráfico dado pela linha contínua (“adulto”), vemos que o volume de vendas y, por
dia, no setor adulto é uma função decrescente de x cujo valor máximo ocorre quando x = 0 e cujo valor
mínimo ocorre quando x = 30. Como y é uma função afim, o valor médio entre os valores máximo e
mínimo ocorre quando
0 + 30 30
x= = = 15,
2 2
ou seja, passados 15 dias do mês. Observe, agora, que 15 é a média aritmética de 10 e 20. Logo, o valor
de y quando x = 15 é a média aritmética dos valores de y quando x = 10 e x = 20, ou seja,
y(10) + y(20) 18 000 + 15 000 33 000 30 000 + 3 000
y(15) = = = = = 15 000 + 1 500 = 16 500,
2 2 2 2
onde usamos as informações do gráfico “adulto”, ou seja, que y(10) = 18 000 e y(20) = 15 000.
Em resumo, deduzimos que o valor médio do volume de vendas por dia, no setor adulto, é igual a
16 500 reais. Esse valor médio é igual ao total de vendas no mês inteiro, dividido pela quantidade de dias
no mês, isto é, por 30. Sendo assim, o total de vendas no setor adulto nos 30 dias pode ser calculado por

16 500 · 30 = 495 000 reais.

De modo similar, considerando o gráfico dado pela linha pontilhada (“infantil”), vemos que o volume de
vendas y, por dia, no setor infantil é também uma função decrescente de x cujo valor máximo ocorre
quando x = 0 e cujo valor mínimo ocorre quando x = 30. Novamente, como y é uma função afim, o
valor médio entre os valores máximo e mínimo ocorre quando x = 15, ou seja, na metade do mês, o

111
ponto médio entre 0 e 30. Como 15 é a média aritmética de 10 e 20, o valor de y quando x = 15 é a
média aritmética dos valores de y quando x = 10 e x = 20, ou seja,
y(10) + y(20) 8 000 + 7 000 15 000
y(15) = = = = 7 500,
2 2 2
onde usamos as informações do gráfico “infantil”, ou seja, que y(10) = 8 000 e y(20) = 7 000. Em resumo,
deduzimos que o valor médio do volume de vendas por dia, no setor infantil, é igual a 7 500 reais. Esse
valor médio é igual ao total de vendas no mês inteiro, dividido pela quantidade de dias no mês, isto é,
por 30. Sendo assim, o total de vendas no setor infantil nos 30 dias pode ser calculado por
7 500 · 30 = 225 000 reais.
Portanto, o total de vendas nos dois setores (adulto e infantil) foi igual a
495 000 + 225 000 = 500 000 + 220 000 = 720 000.
Concluímos, assim, que a meta do total mensal de vendas não foi atingida, faltando R$ 50 000 para
alcançá-la. 

Usamos fortemente a propriedade de que a “soma” de todos os valores diários das vendas, do dia
x = 0 ao dia x = 30, daria o mesmo resultado total que teríamos, caso vendêssemos, todo dia, um
valor igual à média dos valores diários. Para verificar isso, lembre que a variável y (valor diário do
volume de vendas) é uma função afim de x (número de dias desde o início do mês) da forma

y = ax + b,

onde a é a taxa de variação e b é o valor de y quando x = 0. Por exemplo, na questão 110, o gráfico
dado pela linha contínua inclinada (setor “adulto”) representava a função afim com b = 21 000 e
a = −300, ou seja,
y = −300x + 21 000.
Assim,
y(0) + y(30) b + 30a + b
= = 15a + b = y(15).
2 2
Da mesma forma,

y(1) + y(29) a + b + 29a + b 30a + 2b


= = = 15a + b = y(15),
2 2 2
e assim por diante. Somando as vendas diárias do dia x = 0 ao dia x = 30, temos

y(0) + y(1) + . . . + y(14) + y(15) + y(16) + . . . + y(29) + y(30)


  
= y(0) + y(30) + y(1) + y(29) + . . . + y(14) + y(16) + y(15)
= 2y(15) + 2y(15) + . . . + 2y(15) +y(15)
| {z }
7 vezes
= 7 · 2 · y(15) + y(15) = 15 · y(15)

Portanto, a soma dos valores das vendas diárias de x = 0 a x = 30 e dada por

15 · y(15),

ou seja, por 30 vezes o valor médio, isto é,

y(0) + y(30)
30 · , (18)
2

112
como havíamos afirmado. O valor médio
y(0) + y(30)
2
é a média aritmética entre o valor máximo (neste caso, y(0)) e o valor mínimo (neste caso, y(30)) da
variável y.
De modo geral, a soma S(n) dos valores da variável y de x = 0 a x = n é dada por

y(0) + y(n)
S(n) = n · · (19)
2
Como vimos, essa expressão decorre do fato de que os termos y(n) formam uma progressão aritmé-
tica com termo inicial (quando n = 1) igual a a + b e razão igual a a, ou seja,

y(n) = b + an = (a + b) + a(n − 1).

Observe que a expressão (18) determina a área do trapézio T na seguinte figura, delimitado pelas
retas horizontais x = 0 e x = 30, pelo gráfico da função y = ax + b (a linha contínua inclinada) e pelo
eixo horizontal.
variável y

22 000
20 000
18 000
16 000
14 000
12 000
10 000
8 000 T
6 000
4 000
2 000

0 10 20 30 variável x

De fato, as bases de T medem y(0) = 21 000 e y(30) = 12 000 enquanto a altura é igual a 30. Logo, a
área de T é dada por

y(0) + y(30) 21 000 + 12 000


30 · = 30 · = 30 · 16 500 = 495 000.
2 2
Note que essa área é igual a área do retângulo R cuja base é dada pela média das bases de T , ou seja,
por
y(0) + y(30) 21 000 + 12 000
= = 16 500
2 2
e cuja altura é 30, como mostra a seguinte figura.

113
variável y

22 000
20 000
18 000
16 000
14 000
12 000
10 000
8 000 R
6 000
4 000
2 000

0 10 20 30 variável x

Observação 0.30 Nas soluções e comentários das questões, estamos abordando fatos fundamentais
sobre funções afins e progressões aritméticas. O ponto de vista é sempre de que, particularizando a
função afim y = ax + b para valores naturais da variável x, isto é, para x = n com n ∈ N, obtemos
uma progressão aritmética
y(n) = (a + b) + a(n − 1).
Convém, motivados pela discussão anterior sobre somas e médias aritméticas dos termos de uma
progressão aritmética, representar geometricamente a expressão (19) para a soma dos termos de
x = 0 a x = n da progressão aritmética em termos de áreas de trapézios.
Propomos, a seguir, um roteiro para apresentação gradual dessas ideias sobre representações
algébricas e gráficas de funções afins e de progressões aritméticas, ao longo do qual surgem várias
oportunidades para a discussão, com os alunos, sobre conhecimentos e habilidades básicos que precisem
ser recompostos. Esse roteiro será pontuado por questões ou tarefas que devem ser trabalhadas pelos
alunos. É importante que o professor acompanhe os alunos na execução dessas tarefas, identificando
as eventuais dificuldades manifestas por eles e que podem ser as razões primordiais de problemas de
aprendizagem entre alguns alunos. Iniciamos, aproveitando os gráficos apresentados no contexto das
questões 110 e 111, retomando a noção de coordenadas cartesianas. Em seguida, avançamos para
questões que envolvem a representação algébrica ou gráfica de relações de proporcionalidade (e
de linearidade) entre variáveis (ou entre suas variações). Em particular, apresentamos exercícios e
problemas relativos à identificação e interpretação dos coeficientes na expressão algébrica de funções
afins. Finalizamos com a discussão sobre somas e áreas, como antecipada na discussão anterior.
Veremos como a área delimitada pelo gráfico de uma função afim pode ser expressa em termos de
funções quadráticas, o que nos conduz ao início de um novo percurso.

As próximas atividades trabalham conhecimentos e habilidades relativas aos seguintes descritores da


Matriz de Referência do SAEB (terceira série do Ensino Médio):

• D29 - Resolver problema que envolva função exponencial.


• D27 - Identificar a representação algébrica e/ou gráfica de uma função exponencial.
• D28 - Identificar a representação algébrica e/ou gráfica de uma função logarítmica reconhecendo-a
como inversa da função exponencial.

Questão 112 Os cientistas usam modelos matemáticos para estudar como uma doença causada por
vírus se espalha em uma população, usando dados sobre contágio. Em um modelo bastante simples,

114
suponhamos que o número de pessoas contagiadas dobra a cada cinco dias. Sendo assim, havendo
512 contagiados em um dado dia, quantos haverá 20 dias depois?

A) 2 048 B) 2 560 C) 4 096 D) 8 192 E) 10 240

Solução. Em um intervalo de 20 dias, temos 4 grupos de 5 dias. Portanto, ao fim desse intervalo,
temos
512 · 2| · 2{z
· 2 · 2}
=4 vezes
contagiados. Logo, há
29 · 24 = 210 · 23 = 1 024 · 8 = 8 192
contagiados. A alternativa correta é D). 

Questão 113 Marcos solicitou, em março de 2022, financiamento de R$ 2 000, a uma taxa de juros
compostos de 10% ao mês, aplicada pela administradora do seu cartão de crédito. Caso realize o
pagamento 2 meses após o financiamento, isto é, em maio de 2022, o valor a ser pago por Marcos é
igual a

A) R$ 2 420. B) R$ 2 400. C) R$ 2 040. D) R$ 2 020.

Solução. Ao fim de dois meses, o montante é dado por

2 000 · (1 + 0,1) · (1 + 0,1) = 2 000 · 1,21 = 2 420

reais. A alternativa correta é A). 

Questão 114 Marcos (veja o enunciado da questão 113) gostaria de calcular quanto deveria pagar à
administradora do cartão de crédito, caso pudesse realizar o pagamento da dívida apenas 3 meses
após a solicitação do financiamento com taxa de juros compostos de 10% ao mês. Qual das seguintes
expressões matemáticas ele deve usar para fazer esse cálculo?

A) 2 000 · 3,3 B) 2 000 · 1,13 C) 2 000 · 0,13 D) 2 000 · 1,3

Solução. A expressão correta, que permite determinar o montante gerado pela aplicação de juros
composto de 10% = 0,1 ao fim de 3 meses, é

2 000 · (1 + 0,1) · (1 + 0,1) · (1 + 0,1) = 2 000 · 1,13


| {z }
=3 vezes

reais. A alternativa correta é B). 

Observação 0.31 Acréscimos compostos são diferentes de acréscimos simples. No caso de


acréscimos simples sobre um valor inicial C a uma taxa i igual a, por exemplo 5% = 0,05, os
acréscimos são sempre calculados sobre C, isto é,

C + |C ×i + .{z
. . + C ×}i,
t vezes

ou seja, o montante M , isto é, o valor final após t acréscimos, é dado por

M = C + Cit = C(1 + it).

Esses valores definem uma progressão aritmética.

115
No caso de acréscimos compostos, cada acréscimo é calculado sobre o valor imediatamente anterior
e não sobre o valor inicial. Logo, acréscimos compostos a uma taxa i, sobre o valor inicial C, definem
uma progressão geométrica:

C + Ci = C(1 + i),
C(1 + i) + C(1 + i)i = C(1 + i)2 ,
C(1 + i)2 + C(1 + i)2 i = C(1 + i)3 ,

e assim por diante. De um modo geral, o acréscimo composto t vezes resulta no montante

M = C(1 + i)t .

A figura seguinte ilustra a diferença entre os crescimentos aritmético e geométrico, respectivamente


associados aos acréscimos simples e acréscimos compostos:

0 1 2 3 4 t

Os pontos alinhados em uma reta correspondem aos acréscimos simples, a partir do valor inicial
C = 5/32 a uma taxa de acréscimos simples igual a i = 100%. Já os pontos que seguem ao longo de
uma linha exponencial são dados pela progressão geométrica de acréscimos compostos que também
iniciam com C = 5/32, a uma taxa de acréscimos compostos igual a i = 100%.

Questão 115 — SAEPI - Item M120282H6. Em determinado período, um pecuarista constatou que
a população P , em milhares, de caprinos e ovinos da empresa onde atuava variava de acordo com
a função P (t) = 14 · 2t , em que t representa o tempo, em anos, a partir do início do registro dessa
população.
Depois de 6 anos do início desse registro, a população, em milhares, de caprinos e ovinos será de

A) 2. B) 3. C) 9. D) 12. E) 16.
1
Solução. Fixando t = 6 na função exponencial P (t) = 4 · 2t , temos

1 6 26
P (6) = · 2 = 2 = 24 = 16
4 2
milhares da caprinos. A alternativa correta é D). 

Questão 116 Observe os gráficos nas seguintes figuras e faça as atividades sugeridas nesta questão e
na próxima.

116
I
5
III
4 4

2 d
II
1
c

−1 1 2 3 −2 −1 1 2 3
−1

i) O gráfico I representa a função f (x) = ax . Determine o valor de a.


ii) Os pontos (−1, y0 ) e (2, y1 ) pertencem ao gráfico I. Determine o valor de y1 − y0 .
iii) O gráfico II representa a função g(x) = logb x. Determine o valor de b.
iv) Calcule g(1/4) e g(4). Calcule f (−2) e f (2).
v) Mostre que f (x + 1) = 2f (x), para todo x real. Conclua que f (x) = 2x .
vi) Mostre que g(xx0 ) = g(x) + g(x0 ), para quaisquer números reais x, x0 positivos.

Solução. i) Note que o ponto (1, 2) pertence ao gráfico I. Logo, se x = 1, então y = f (1) = 2.
Portanto 2 = f (1) = a1 = a, ou seja, a = 1.
ii) Para que o par (−1, y0 ) pertença ao gráfico I, é preciso que y0 = f (−1). Como a = 2 (vide questão i)
anterior), temos
1
f (−1) = 2−1 = .
2
1
Portanto y0 = 2 . Da mesma forma, devemos ter
y1 = f (2) = 22 = 4.
1
Portanto, y1 − y0 = 4 − 2 = 27 .

Note que f (x) = 2x não é uma função afim, pois não tem taxa de variação constante De fato, a taxa
de variação da função f de x = −1 para x = 0 é
1
f (0) − f (−1) 1− 2 1
= = ,
0 − (−1) 1 2

enquanto a taxa de variação da função f de x = −1 para x = 2 é


1
f (2) − f (−1) y1 − y0 4− 2 7/2 7
= = = = ·
2 − (−1) 2 − (−1) 3 3 6

Portanto, a taxa de variação entre dois pontos do gráfico I não se mantém constante.

iii) Por definição, dados um número real x > 0 e um número real positivo b, diferente de 1, o logaritmo
de x na base b é o expoente ao qual devemos elevar b para obtermos x.
De acordo com a figura da esquerda, o ponto (2, 1) pertence ao gráfico da função logarítmica
g(x) = logb x. Então, g(2) = 1, ou seja,
logb 2 = 1.
Portanto,
b1 = 2,

117
ou seja, b = 2.
iv) Temos
1
g(1/4) = log2 1/4 = ` ⇐⇒ 2` = ⇐⇒ ` = −2
4
e
g(1/4) = log2 4 = m ⇐⇒ 2m = 4 ⇐⇒ m = 2.
Note que, de acordo com esses cálculos,
1
f (−2) = 2−2 = e f (2) = 22 = 4,
4
ou seja,
g(f (−2)) = −2 e f (g(1/4)) = 1/4
bem como
g(f (2)) = −2 e f (g(4)) = 4.

v) Por conta das propriedades operatórias de potências, temos

f (x + 1) = 2x+1 = 2x · 21 = 2x · 2 = 2 · f (x),

seja qual for o valor de x.


vi) Por conta das propriedades operatórias de potências, temos
0
g(x) = ` e g(x0 ) = `0 ⇐⇒ 2` = x e 2` = x0 .

Portanto
0 0
xx0 = 2` · 2` = 2`+` ,
o que implica que
log2 (xx0 ) = ``0
sejam quais forem os valores (positivos) de x e x0 .


Questão 117 Observe o gráfico III na figura da questão 116 e faça as seguintes atividades.

i) O gráfico III representa a função h(x) = Ax . Determine o valor de A.


ii) Calcule h(−3) e h(3).
iii) Os pontos (−1, d) e (1,c) pertencem ao gráfico III. Qual o valor de d − c?
iv) Para qual valor de x devemos ter h(x) = 1/16?
v) Para qual valor de x devemos ter h(x) = 16?
vi) Mostre que h(x + x0 ) = h(x)h(x0 ), para quaisquer números reais x, x0 .

Solução. i) Note que o ponto (−2, 4) pertence ao gráfico III. Logo, se x = −2, então y = h(−2) = 4.
Portanto 4 = h(−2) = A−2 = 1/A2 , ou seja, A2 = 1/4, o que implica A = 1/2.
ii) Temos h(−3) = (1/2)−3 = 23 = 8 e h(3) = (1/2)3 = 1/8.
iii) Para que o par (−1, d) pertença ao gráfico III, é preciso que d = h(−1). Como A = 1/2 (vide
questão i) anterior), temos
h(−1) = (1/2)−1 = 2.
Portanto d = 2. Da mesma forma, devemos ter

c = f (1) = (1/2)1 = 1/2.

118
1
Portanto, d − c = 2 − 2 = 32 .
iv) Temos
h(x) = (1/2)x = 1/16 ⇐⇒ x = 4
e
h(x) = (1/2)x = 16 ⇐⇒ x = −4

v) Por conta das propriedades operatórias de potências, temos


0 0
h(x + x0 ) = (1/2)x+x = 1/2x · (1/2)x = h(x) · h(x0 ).

sejam quais forem os valores de x e x0 .




Seção 4. Quarto percurso: leitura e interpretação de gráficos e tabelas

As tarefas a seguir envolvem conhecimentos prévios fundamentais para desenvolver as habilidades nos
seguintes descritores da Matriz de Referência do SAEB (terceira série do Ensino Médio):

• D34 - Resolver problema envolvendo informações apresentadas em tabelas e/ou gráficos.


• D35 - Associar informações apresentadas em listas e/ou tabelas simples aos gráficos que as
representam e vice-versa.
• D16 - Resolver problema que envolva porcentagem.
• D33 - Calcular a probabilidade de um evento.

Questão 118 — SAEPE - Item M090067ES. Observe na tabela abaixo o valor faturado com as compras
realizadas pela internet no Brasil no período de 2006 a 2010.
Ano Faturamento (em reais)
2006 4,4 bilhões
2007 6,3 bilhões
2008 8,2 bilhões
2009 10,6 bilhões
2010 14,8 bilhões

Disponível em: Fonte:www.e-commerce.org.br.

O gráfico que melhor representa os dados dessa tabela é

119
Solução. A alternativa correta é B). 

Questão 119 — SAEPE - Item M100079A9. Um grupo de pessoas respondeu a uma pesquisa sobre a
forma de escolha de seus médicos. As respostas obtidas foram registradas no quadro a seguir.
Como você escolhe seu médico?
Motivos Porcentagem
Proximidade 22%
Indicação 31%
Disponibilidade 19%
Atendimento telefônico 13%
Outros motivos 15%

De acordo com os dados desse quadro, o gráfico que melhor representa essas informações é

Solução. A alternativa correta é D). 

120
Questão 120 — SPAECE, Item M120976E4. O dono de uma locadora fez uma pesquisa para saber a
quantidade de filmes que os clientes alugam por mês. Os resultados dessa pesquisa estão representados
na tabela abaixo.
QUANTIDADE DE FILMES ALUGADOS POR MÊS QUANTIDADE DE CLIENTES
2 OU MENOS 42
3 35
4 87
5 95
6 OU MAIS 58

De acordo com essa tabela, quantos clientes alugam menos de 5 filmes por mês?

A) 87 B) 95 C) 164 D) 259 E) 317

Solução. Devemos somar as quantidades na coluna da direita das linhas da tabela correspondentes
a “2 ou menos”, “3” e “4”, obtendo

42 + 35 + 87 = 40 + 35 + 85 + 2 + 2 = 40 + 120 + 4 = 164.

Logo, a alternativa correta é a C). 

Questão 121 A escola realizou uma pesquisa entre os 480 estudantes da terceira série sobre suas
escolhas de cursos na universidade. As respostas indicaram cursos em quatro áreas: exatas, tecnológica,
biomédica e humanidades, conforme representado no seguinte gráfico de setores.

Com base no gráfico, podemos afirmar que

A) Mais de 50% escolheram cursos na área biomédica.


B) Cerca de 320 estudantes escolheram cursos na área de humanidades.
C) Aproximadamente 1/3 dos estudantes escolheu cursos na área de exatas.
D) Menos de 33% dos estudantes escolheu cursos na área tecnológica.

Solução. A alternativa correta é D). 

Questão 122 — PAEBES - Item M120169ES, adaptado. João anotou os gastos extras que teve no mês
de janeiro em uma tabela, como a representada abaixo.
Despesas Valor (R$)
Material escolar 150
IPVA 750
IPTU 245
IR 978

O gráfico que melhor representa os dados dessa tabela é

121
Solução. Apenas as alternativas C), D) e E) representam corretamente as despesas com material
escolar, que são menores que R$ 200,00. Essas alternativas também representam adequadamente as
despesas com IR (imposto de renda), que são aproximadamente iguais a R$ 1 000,00.
No entanto, a D) não representa corretamente as despesas com IPVA, iguais a R$ 750,00 e, portanto,
entre R$ 600,00 e R$ 800,00. A alternativa C) representa as despesas com IPTU como maiores do que
R$ 800,00, o que não condiz com os dados. A alternativa E) representa corretamente as despesas com
IPTU, aproximadamente iguais a R$ 250,00. Logo, a alternativa correta é a E). 

Questão 123 — SPAECE - Item M100281E4. No gráfico abaixo está representado o resultado de uma
pesquisa realizada com um grupo de pessoas para saber de que forma elas descobriram que precisavam
do uso de óculos.

Disponível em: http://olhosartificiais.blogspot.com.br/p/graficos-e-tabelas.html. Acesso em: 17 jun. 2013.

De acordo com esse gráfico, a quantidade de pessoas que descobriu que precisava usar óculos por

122
causa da vista embaçada é igual a

A) 15 B) 18 C) 27 D) 29. E) 32

Solução. Precisamos somar os dados das colunas que correspondem à legenda “vista embaçada”, o
que resulta em
9 + 5 + 15 + 3 = 12 + 20 = 32.
Logo, a alternativa correta é a E). 

Questão 124 — SPAECE - Item M120964E4, Adaptado. O gráfico abaixo apresenta a quantidade de
alunos matriculados, por turma, no 3◦ ano do Ensino Médio de uma escola.

A tabela que representa as informações contidas nesse gráfico é

Solução. As únicas alternativas que representam corretamente a informação sobre a turma 1 são
as de letras A) e E). No entanto, a alternativa A) não representa corretamente as informações sobre as

123
turmas 4 e 5. Logo, a alternativa correta é E). 

Questão 125 O gráfico de colunas seguinte mostra o número de medalhas obtidas pelo Brasil em
cinco olimpíadas.

Qual o número médio de medalhas obtidas pelo Brasil nessas olimpíadas?

A) 10 B) 15 C) 17 D) 19 E) 75

Solução. O número médio de medalhas é dado por


12 + 10 + 17 + 17 + 19 75
= = 15
5 5
A alternativa correta é B). 

Questão 126 — SPAECE - Item M120350H6, daptado. A tabela abaixo apresenta os percentuais de
demanda brasileira por fertilizante, separados por cultura agrícola, no ano de 2013.
Cultura agrícola Fertilizante demandado
Cana-de-açúcar 15%
Algodão 4%
Café 6%
Soja 38%
Milho 17%
Outras 20%

Disponível em: http://ruralcentro.uol.com.br/analises/uso-de-fertilizantes-no-brasil-por-cultura-agricola-4696 Acesso em: 24 fev. 2017.

*Adaptada para fins didáticos.

O gráfico que apresenta a mesma relação entre os dados dessa tabela é

124
Solução. Podemos observar alguns erros nos seguintes gráficos de setores: no gráfico da alternativa
A), por exemplo, os percentuais de algodão e café estão trocados; no gráfico B), algodão aparece com
20%; no gráfico da alternativa C), cana-de-açúcar está associada ao percentual 17% e, no gráfico da
alternativa E), ao percentual 20%. Constatamos que a alternativa correta é D). 

Questão 127 — SAEPE - Item M110241H6, adaptado. A tabela abaixo apresenta o percentual de
consumo de energia elétrica em nível nacional, por região geográfica, no ano de 2015.
Consumo de energia elétrica em nível nacional no ano de 2015

Região geográfica Percentual (em %)


Norte 7
Nordeste 17
Sudeste 50
Sul 18
Centro-Oeste 8

Disponível em: http://www.epe.gov.br/mercado/Documents. Acesso em: 15 jul. 2016. Adaptado para fins didáticos.

Qual é o gráfico que se relaciona com os dados fornecidos nessa tabela?

125
Solução. Podemos observar alguns erros nos seguintes gráficos de setores: no gráfico da alternativa
A), por exemplo, os percentuais das regiões Sul e Centro-Oeste estão trocados; no gráfico B), a região
Nordeste aparece com o percentual 8%; no gráfico da alternativa C), a região Norte está associada ao
percentual 50% e, no gráfico da alternativa D), ao percentual 17%. Constatamos que a alternativa
correta é a E). 

Questão 128 — SPAECE - Item M100522E4, Adaptado. Foi feita uma pesquisa em uma determinada
empresa para encontrar um horário alternativo para a entrada dos funcionários. Cada um escolheu o
horário que era mais conveniente para iniciar o trabalho e o resultado está representado na tabela
abaixo.
Horários de chegada Percentual de funcionários
7h 31%
8h 14%
9h 46%
10h 9%

Qual dos gráficos abaixo apresenta as informações dessa tabela?

126
Solução. Podemos observar alguns erros nos seguintes gráficos de setores: no gráfico da alternativa
A), por exemplo, os percentuais das 7h e 8h estão trocados; no gráfico B), o horário 8h aparece com o
percentual 46%; nos gráficos das alternativas C) e E), o horário 7h está associada ao percentual 46% .
Constatamos que a alternativa correta é a D). 

Questão 129 — SAEPE - Item M100112H6, adaptado. O gráfico abaixo apresenta a média das notas
nas avaliações de cada bimestre dos estudantes de uma turma do primeiro ano do Ensino Médio nas
disciplinas Matemática e Língua Portuguesa.

De acordo com os dados desse gráfico, os aumentos verificados nas médias das notas do segundo para
o terceiro bimestre, nas disciplinas de Matemática e de Língua Portuguesa, foram, respectivamente,
iguais a

A) 0,7 e 1,3.
B) 1,1 e 1,3.
C) 1,2 e 1,4.
D) 1,9 e 2,7.
E) 2,1 e 3,0.

Solução. Devemos observar os dados informados no segundo e no terceiro par de colunas, da


esquerda para a direita. De acordo com os dados, as médias em Matemática no segundo e terceiro

127
bimestres são, respectivamente, iguais a 6,1 e 7,3. Logo, o aumento (absoluto, não percentual) da média
em Matemática, foi igual a
7,3 − 6,1 = 1,2.
Da mesma forma, somos informados de que as médias em Língua Portuguesa no segundo e terceiro
bimestres são, respectivamente, iguais a 7,2 e 8,6. Logo, o aumento (absoluto, não percentual) da média
em Matemática, foi igual a
8,6 − 7,2 = 1,4.
Portanto, a alternativa correta é C). 

Questão 130 Com base nos dados da questão 129, calcule a média anual da turma em Língua
Portuguesa e em Matemática. Em qual dessas duas disciplinas as médias bimestrais variaram mais?

Solução. A média anual em Matemática é a média aritmética das 4 médias bimestrais, ou seja,
5,4 + 6,1 + 7,3 + 7,5 5,5 + 6 + 7,3 + 7,5 26,3 24 + 2,3
= = = = 6,575 ≈ 6,6.
4 4 4 4
Da mesma forma, a média anual em Língua Portuguesa é dada pela média aritmética das 4 médias
bimestrais, isto é,
5,9 + 7,2 + 8,6 + 8,9 6 + 7,1 + 8,5 + 9 13,1 + 17,5 30,6 28 + 2,6
= = = = = 7,65.
4 4 4 4 4
As variações das médias bimestrais em relação à média anual em Matemática são dadas por

|5,4 − 6,6| = 1,2 |6,1 − 6,6| = 0,5 |7,3 − 6,6| = 0,7 |7,5 − 6,6| = 0,9

Já as variações das médias bimestrais em relação à média anual em Língua Portuguesa são dadas por

|5,9 − 7,65| = 1,75 |7,2 − 7,65| = 0,45 |8,6 − 7,65| = 0,95 |8,9 − 7,65| = 1,25

Portanto, a variabilidade das médias bimestrais em Língua Portuguesa, em relação à média anual, é
maior que nas médias bimestrais de Matemática. 

Questão 131 — SPAECE - Item MM120896E4, adaptado. O gráfico a seguir apresenta as atribuições
de um funcionário durante uma semana de trabalho.

Corrija a seguinte tabela para que melhor represente os dados desse gráfico.

128
Atribuições Porcentagem
Outros 10%
Projeto Interno 25%
Projeto Externo 20%
Reunião 26%
Tarefa da Diretoria 11%
Treinamentos 8%

Solução. A alternativa correta é a A). 

Questão 132 — SAEGO - M120087ES. Em em envelope existem 4 perguntas sobre Geografia, 5 sobre
História, 3 sobre Biologia e 2 sobre Inglês. Uma pergunta foi sorteada aleatoriamente.
A probabilidade de ser sorteada uma pergunta sobre Biologia é, aproximadamente,

A) 21,43% B) 25% C) 27,27% D) 33,33% E) 75%

Solução. O total de perguntas é

4 + 5 + 3 + 2 = 14.

Desse total, temos 3 pergunta de Biologia. Logo, uma seleção aleatória pode extrair uma pergunta de
Biologia com probabilidade
3
= 21, 428 . . . ≈ 21,43%.
14
Logo, a alternativa correta é E). 

Questão 133 — PAEBES - Item M120708H6. Para realizar um sorteio, Rosana vai utilizar uma urna
contendo 10 bolinhas idênticas numeradas de 1 a 10.
Qual é a probabilidade de a primeira bolinha retirada por Rosana dessa urna ser a de número 3?

1 1 3 9 10
A) B) C) D) E)
10 9 10 10 9

Solução. A alternativa correta é A). 

Questão 134 — SAEPI - Item M120196H6. Em um saco havia 15 bolas idênticas numeradas de 1 a 15.
Uma bola foi retirada aleatoriamente.
Qual é a probabilidade da bola retirada estar marcada com um número maior que 9?

1 6 7 8 9
A) B) C) D) E)
15 15 15 15 15

Solução. Há 6 números maiores que 9 nos números de 1 a 15. Logo, a probabilidade é

6
·
15

A alternativa correta é B). 

129
Questão 135 Considerando o contexto da questão 134, qual a probabilidade da bola retirada estar
marcada com um número maior que 9, sabendo-se que esse número é ímpar?

Solução. A informação de que o número retirado é ímpar restringe nosso conjunto de possibilidades,
ou seja, restringe o espaço amostral, que passa a ser

I = {1, 3, 5, 7, 9, 11, 13, 15}.

Desses números, aqueles maiores que 9 são

M = {11, 13, 15}.

Portanto, a probabilidade de retirar-se um número maior que 9, dentre os números ímpares, é

número de elementos de M 3
= ·
número de elementos de I 8
O fato de termos uma informação disponível sobre o evento aleatório modifica a probabilidade: temos
um exemplo de probabilidade condicional. Podemos calculá-la segundo o seguinte raciocínio: o
conjunto dos números ímpares de 10 a 15 representa

3
Probabilidade do número ser ímpar e ser maior que 9 =
15
do espaço amostral de todos os números naturais de 1 a 15, enquanto que os números maiores que 9
(não necessariamente ímpares) representam

8
Probabilidade do número ser ímpar =
15
do espaço amostral. Assim, a probabilidade condicional seria

3/15 3
= ·
8/15 8

Questão 136 Os seguintes gráficos de setores mostram os percentuais de desempenho dos alunos de
duas turmas, A e B, do nono ano em um teste de Matemática na escola:

Sabendo que 2/5 dos alunos estão na turma A e 3/5 dos alunos na turma B, qual o percentual de
alunos com desempenho adequado nas duas turmas ao todo?

A) 45 % B) 46 % C) 50 % D) 90 %

130
Solução. Sabemos que 2/5 dos alunos estão na turma A. Desses, o gráfico de setores do lado
esquerdo informa que 40% estão no nível adequado, ou seja,
2 80
40% · = % = 16%.
5 5
Da mesma forma, sabemos que 3/5 dos alunos estão na turma B. Desses, o gráfico de setores do lado
direito informa que 50% estão no nível adequado, ou seja,
3 150
50% · = % = 30%.
5 5
Portanto, a alternativa correta é a letra B). 

Questão 137 — PUC-RJ 2021. Um dado comum é um cubo com faces numeradas de 1 a 6. Jorge joga
dois dados comuns e soma os pontos. Qual é a probabilidade de que a soma total seja igual a 5?

1 1 1 5
A) B) C) D)
12 9 6 36
Solução. Os eventos favoráveis correspondem aos pares ordenados (1, 4), (2, 3), (3, 2), (4, 1). Logo,
há 4 eventos favoráveis. No entanto, há 6 · 6 = 36 pares possíveis, ou seja, há 36 eventos possíveis.
Assim, a probabilidade desejada é
4 4:4 1
= = ,
36 36 : 4 9
o que completa a solução. 

Questão 138 — PUC-RJ 2022. Uma caixa contém 10 bolas azuis, de mesmo tamanho. Três delas têm
o logotipo PUC escrito, e as outras, não. Dispõem-se, ao acaso, as dez bolas azuis, lado a lado, em
linha reta. Qual é a probabilidade de que as três bolas com o logotipo desenhado fiquem juntas?

1 1 1 1
A) B) C) D)
30 15 12 10
Solução. O número de eventos possíveis corresponde às possíveis permutações de 10 objetos, isto
é, ao fatorial de 10. Para calcular-se o número de eventos favoráveis, isto é, das permutações em que
as 3 bolas azuis ficam juntas, podemos tratar o conjunto de 3 bolas (observe que existem 3! formas
desse conjunto, dadas pelas permutações das 3 bolas) como um único elemento. Assim, teríamos a
permutação de 7 + 1 = 8 objetos, ou seja, 8!. Sendo assim, a probabilidade é dada por
# número de eventos favoráveis 3! · 8! 3·2 1
= = = ,
# número de eventos possíveis 10! 10 · 9 15
o que completa a solução. 

Questão 139 — UFRGS 2016, adaptado. Dardos são lançados em direção a um alvo com a forma de
um quadrado de lado 10, como representado na figura abaixo, tendo igual probabilidade de atingir
qualquer região do alvo.

x
10

131
Se todos os dardos atingem o alvo e 50% atingem o quadrado de lado x, o valor inteiro mais próximo
de x é

A) 4. B) 5. C) 6. D) 7. E) 8.

Solução. A probabilidade de que o dardo atinja o quadrado de lado x é igual a razão entre a área
desse quadrado e a área total, ou seja, igual a

x2
·
100
Portanto,
x2 50
= 50% = ·
100 100
Logo, x2 = 50 = 2 · 25. Assim, √
x = 5 2 ≈ 5 · 1,4 = 7,
o que corresponde à alternativa D). 

Questão 140 — SAEPI - Item M120124H6. Em uma academia de dança foram inscritos 5 homens e 9
mulheres para as aulas de forró. Quantos pares diferentes constituídos por um homem e uma mulher
poderão ser formados com esses alunos?

A) 5 B) 45 C) 91 D) 126. E) 183.

Solução. Este é um exemplo de questão em que podemos usar o princípio multiplicativo de


contagem: para cada um dos 5 homens inscritos, há 9 mulheres inscritas. Portanto, a quantidade de
pares formados por um homem e uma mulher é igual a

9
|
9 + 9 + 9} = 5 · 9 = 45.
+ 9 + {z
5 vezes

Portanto, a alternativa correta é B). 

Questão 141 — ENEM 2019 - PPL, Caderno 7 - Azul, Questão 140, adaptada. Considere que a safra
nacional de cereais, leguminosas e oleaginosas, em 2012, aponte uma participação por região conforme
indicado no gráfico. Em valores absolutos, essas estimativas indicam que as duas regiões maiores
produtoras deveriam produzir juntas um total de 119,8 milhões de toneladas em 2012.

De acordo com esses dados, a produção estimada, em milhão de tonelada, de cereais, leguminosas e
oleaginosas, em 2012, na Região Sudeste do país, foi um valor mais aproximado de

A) 11,4. B) 13,6. C) 15,7. D) 18,1. E) 35,6.

132
Solução. De acordo com o gráfico de setores na figura (forma de gráfico que, muitas vezes, chamamos
de gráfico de pizza), as duas regiões com maior produção, em 2012, foram as regiões Sul e Centro-Oeste,
com percentuais de 37,2% e 38,3%, respectivamente, do total da produção nacional. Portanto, juntas,
essas regiões produziram
37,2% + 38,3% = 75,5%
da produção nacional neste ano. Segundo o enunciado, a produção das duas regiões foi igual a 119,8
milhões de toneladas, ou seja, aproximadamente 120 milhões de toneladas. Com essas duas informações,
concluímos que 75,5% da produção nacional equivale a cerca de 120 milhões de toneladas.
Logo, aproximadamente 25% da produção nacional equivalem a cerca de
120
= 40 milhões de toneladas.
3
Portanto, 100% da produção nacional equivalem a cerca de

40×4 = 160 milhões de toneladas.

Como a Região Sudeste produziu 11,4% deste total, sua produção equivale a
11
×160 = 18,24 milhões de toneladas
100
o que corresponde à letra D). 

Poderíamos ter resolvido a questão da seguinte forma: indicamos por x a produção nacional em
milhões de toneladas, em 2012. Vimos que cerca de 75% desta produção equivalem a cerca de 120
milhões de toneladas, ou seja, que
75
x = 120 milhões de toneladas.
100
Uma vez que
75 3
=
100 4
temos
3
x = 120 milhões de toneladas.
4
Logo, multiplicando os dois lados desta equação por 4 e dividindo os dois lados desta equação por 3,
obtemos
4
x = ×120 milhões de toneladas,
3
ou seja, concluímos que a produção nacional, em 2012, foi igual a

x = 160 milhões de toneladas.

Agora, precisamos calcular 11,4% desta produção, ou seja,


11,4 11,4
x= ×160 = 18,24 milhões de toneladas,
100 100
o que finaliza a solução.

Questão 142 — ENEM 2014, Caderno 7 - Azul, Questão 137, adaptada. A taxa de fecundidade é um
indicador que expressa a condição reprodutiva média das mulheres de uma região, e é importante
para uma análise da dinâmica demográfica dessa região. A tabela apresenta os dados obtidos pelos
Censos de 2000 e 2010, feitos pelo IBGE, com relação à taxa de fecundidade no Brasil.

133
Ano Taxa de fecundidade no Brasil
2000 2,38
2010 1,90

Disponível em: www.saladeimprensa.ibge.gov.br. Acesso em: 31 jul. 2013.

Suponha que a variação percentual relativa na taxa de fecundidade no período de 2000 a 2010 se
repita no período de 2010 a 2020. Nesse caso, em 2020 a taxa de fecundidade no Brasil estará mais
próxima de

A) 1,14. B) 1,42. C) 1,52. D) 1,70. E) 1,80.

Antes de passarmos à resolução da questão, revisemos este conceito de variação percentual: trata-se
de comparar o valor de uma variável Q, digamos, em dois instantes, que denominaremos antes e
depois. A variação de Q é dada pela diferença

Q(depois) − Q(antes).

Para medirmos a intensidade desta variação, ou seja, o quanto esta variação é mais ou menos
significativa, comparamos a variação com o valor inicial da variável, isto é, calculamos o quociente ou
razão
Q(depois) − Q(antes)
·
Q(antes)
Para fixar ideias, calculemos as variações percentuais de duas variáveis, a variável Q e a variável R,
de acordo com a tabela abaixo:

Variável Valor antes Valor depois Variação


Q 10 30 30 − 10 = 20
R 20 40 40 − 20 = 20

Note que as variações das variáveis são ambas iguais a 20. No entanto, a variação percentual de Q é
dada por
Q(depois) − Q(antes) 30 − 10 20
= = = 2,
Q(antes) 10 10
ou seja, 200%, ao passo que a variação percentual de R é dada por

R(depois) − R(antes) 40 − 20 20
= = = 1,
R(antes) 20 20

ou seja, 100%. Portanto, a variação percentual de Q é maior que a variação percentual de R. Isto
quer dizer que o aumento de 10 para 30 (da variável Q) é mais significativo que o aumento de 20 para
40 (da variável R).
Finalmente, um comentário para justificar o adjetivo “percentual”: considere que o valor inicial
da variável, por exemplo o valor Q(antes) corresponde ao total, isto é, a 100%. Calculemos, então,
qual a porcentagem deste total que corresponde à variação Q(depois) − Q(antes). Para isto, usamos
a seguinte regra de três ou proporção:

Q(antes) está para 100 %

assim como

Q(depois) − Q(antes) está para x %

134
Portanto,
x% Q(depois) − Q(antes)
= ·
100% Q(antes)
Logo,
Q(depois) − Q(antes)
x% = ·
Q(antes)

Solução. Considerando a definição de variação percentual que revisamos no quadro anterior,


calculemos a variação da taxa de fecundidade de 2000 (antes) a 2010 (depois). Temos

Taxa em 2010 − Taxa em 2000 = 1,90 − 2,38 = −0,48,

uma variação negativa, já que a taxa de fecundidade diminuiu de 2000 para 2010. Agora, vejamos o
quão significativamente diminuiu. Para isso, calculamos a variação percentual

Taxa em 2010 − Taxa em 2000 −0,48


≈ = −0,20 = −20%.
Taxa em 2000 2,38

No enunciado, afirma-se que a variação percentual de 2010 a 2020 deve ser a mesma. Portanto, a taxa
de fecundidade de 2020 deve ser 20% menor do que em 2010. Ou seja, a taxa de fecundidade em 2020
deve ser igual a 80% da taxa de fecundidade em 2010, isto é,
80
Taxa em 2020 = ×Taxa em 2020
100
80
= ×1,90
100
= 1,52.

Outra maneira de calcularmos a taxa em 2020 é a seguinte: como a variação percentual de 2010 a 2020
é também de −20%, temos

Taxa em 2020 − Taxa em 2010


= −20%.
Taxa em 2010
Logo
Taxa em 2020
− 100% = −20%
Taxa em 2010
e, sendo assim,
Taxa em 2020
= 80%,
Taxa em 2010
donde concluímos, como antes, que
80 80
Taxa em 2020 = ×Taxa em 2020 = ×1,90 = 1,52.
100 100
Portanto, a alternativa correta é a de letra C). 

Questão 143 — ENEM 2013, Caderno Azul, Questão 150. Numa escola com 1 200 alunos foi realizada
uma pesquisa sobre o conhecimento desses em duas línguas estrangeiras: inglês e espanhol. Nessa
pesquisa constatou-se que 600 alunos falam inglês, 500 falam espanhol e 300 não falam qualquer um
desses idiomas. Escolhendo-se um aluno dessa escola ao acaso e sabendo-se que ele não fala inglês,
qual a probabilidade de que esse aluno fale espanhol?

A) 1/2. B) 5/8. C) 1/4. D) 5/6. E) 5/14.

135
Solução. Como há 1 200 − 300 = 900 alunos que falam pelo menos uma das duas línguas; e há
600 + 500 = 1 100 que falam inglês ou espanhol (“ou”, aqui, não significa alternativas excludentes, mas
união de conjuntos), concluímos que 1 100 − 900 = 200 alunos falam as duas línguas (estão na intersecção
dos conjuntos). Há, portanto, 500 − 200 alunos que falam apenas espanhol. Considerando como espaço
amostral o conjunto dos alunos que não falam inglês, vemos que esse conjunto tem 300 + 300 = 600
alunos, os 300 que não falam nenhuma das duas línguas e os 300 que falam apenas espanhol. Assim, a
probabilidade desejada é igual a
300 1
= ,
300 + 300 2
o que corresponde à alternativa A). 

Questão 144 — ENEM 2013, Caderno Amarelo, Questão 141. Uma loja acompanhou o número de
compradores de dois produtos, A e B, durante os meses de janeiro, fevereiro e março de 2012. Com
isso, obteve este gráfico:

A loja sorteará um brinde entre os compradores do produto A e outro brinde entre os compradores
do produto B. Qual a probabilidade de que os dois sorteados tenham feito suas compras em fevereiro
de 2012?

A) 1/20. B) 3/242. C) 5/22. D) 6/25. E) 7/15.

Solução. A probabilidade de sortear um comprador do produto A que tenha feito a compra em


fevereiro é igual a
30 30 3
= = ,
10 + 30 + 60 100 10
ao passo que a probabilidade de sortear um comprador do produto B que tenha feito a compra em
fevereiro é igual a
20 20 1
= = ·
20 + 20 + 80 120 6
Como esses sorteios são (hipótese tácita no enunciado) independentes, a probabilidade de que ambos
ocorram é igual ao produto dessas probabilidades acima, ou seja, igual a
3 1 1
· = ,
10 6 20
o que corresponde à alternativa A). 

Questão 145 A tabela seguinte mostra a distribuição, em percentuais, da população brasileira em


diferentes grupos de idade, nos anos de 1980 e 2010.

136
Grupos de idade Ano 1980 Ano 2010
0 a 14 anos 38,20% 24,08%
15 a 64 anos 57,68% 68,54%
65 anos ou mais 4,01% 7,38%

Fonte: IBGE. Disponível em https://brasilemsintese.ibge.gov.br/populacao/distribuicao-da-populacao-por-grandes-grupos-de-idade.html

Faça as seguintes atividades com base nessa tabela.

i) Calcule o percentual de habitantes com 15 anos ou mais em 2010.


ii) Calcule o percentual de aumento da população de 65 anos ou mais de 1980 para 2010.
iii) Determine qual dos grupos teve maior aumento percentual de 1980 para 2010.
iv) Construa gráficos de barras para representar os dados na tabela.
v) Construa gráficos de setores para representar os dados na tabela.
vi) Com base nos dados, podemos concluir que a população brasileira ficou “menos jovem” no
período de 1980 a 2010?

Solução. i) Somamos os percentuais de habitantes com idades de 15 a 64 anos e com idades com
65 anos ou mais, obtendo

68,54% + 7,38% = 68,52% + 7,40% = 75,92%.

ii) O aumento de percentual da população com 65 anos de idade ou mais, de 1980 para 2010, é igual a

7,38% − 4,01% = 4,37%.

Portanto, a variação percentual do percentual dessa população, de 1980 para 2010, é igual a

4,37%
≈ 1,09,
4,01%

ou seja, aproximadamente 109%.

iii) O aumento de percentual da população com 0 a 14 anos de idade, de 1980 para 2010, é igual a

24,08% − 38,20% = −14,12%.

Portanto, a variação percentual do percentual dessa população, de 1980 para 2010, é negativa e igual a

−14,12%
≈ −0,37,
38,20%

ou seja, aproximadamente −37%. Da mesma forma, a variação percentual do percentual da população


com idades de 15 a 64 anos, de 1980 para 2010, é negativa e igual a

68,54% − 57,68% 10,86%


= ≈ 0,19,
57,68% 57,68%

isto é, 19%.

iv) Temos um gráfico com duas colunas por grupo, uma representando os percentuais em 1980, outra
com os dados de 2010:

137
v) Temos dois gráficos de setores, um representando os percentuais em 1980, outro com os dados de
2010:

vi) Houve um decréscimo no percentual da população com idades de 0 a 14 anos, um acréscimo percentual
do grupo com idades de 15 a 64 anos, mas um aumento significativamente maior da população com 65
anos ou mais. Podemos concluir que há maior frequência de idades mais elevadas na população em 2010
do que havia em 1980.


Questão 146 A tabela seguinte informa taxas de câmbio e o preço, na moeda nacional, de um famoso
sanduíche em alguns países.

País Moeda Taxa de câmbio Preço do sanduíche


Brasil Real (R$) 1,00 R$ 22,90
EUA Dólar (US$) 4,78 US$ 5,15
Inglaterra Libra (£) 6,10 £ 3,69

Com base nesses dados, em que país o sanduíche é mais caro para um turista brasileiro?

Solução. O preço do sanduíche, nos EUA, em reais, é

5,15 · 4,78 = 24,617 ≈ 24,62 reais,

138
enquanto o preço do mesmo sanduíche, na Inglaterra, em reais, é

3,69 · 6,10 = 22,509 ≈ 22,51 reais.

Logo, o preço mais alto do sanduíche, em reais, é nos EUA. 

Questão 147 — ENEM 2019, Caderno Amarelo, Questão 177. A taxa de urbanização de um município
é dada pela razão entre a população urbana e a população total do município (isto é, a soma das
populações rural e urbana). Os gráficos apresentam, respectivamente, a população urbana e a
população rural de cinco municípios (I, II, III, IV, V) de uma mesma região estadual. Em reunião
entre o governo do estado e os prefeitos desses municípios, ficou acordado que o município com maior
taxa de urbanização receberá um investimento extra em infraestrutura.

Segundo o acordo, qual município receberá o investimento extra?

A) I B) II C) III D) IV E) V

Solução. Na questão 148, iii), calculamos o percentual da população urbana, relativamente à


população total, de cada um dos cinco municípios. O maior percentual é o do município III. Portanto, a
resposta correta é a alternativa C). 

Questão 148 Com base no enunciado e nos dados da questão 147, faça as seguintes atividades.

i) Determine os municípios que têm população (urbana mais rural) maior que 20 000 hanitantes.
ii) Determine a população total dos cinco municípios.
iii) Determine os percentuais da população urbana e da população rural de cada um dos cinco
municípios.
iv) Calcule a média da população urbana dos cinco municípios.
v) Construa uma tabela para representar os dados dos gráficos de barras.
vi) Construa gráficos de setores para as populações urbanas, as populações rurais e as populações
totais dos cinco municípios.

Solução. i) e iii) Os municípios IV e V têm, respectivamente, 18 000 + 10 000 = 28 000 e 17 000 +


12 000 = 29 000 habitantes. Portanto, são os que têm mais de 20 000 habitantes ao todo. De fato, os
municípios I, II e III têm, respectivamente

8 000 + 4 000 = 12 000, 10 000 + 8 000 = 18 000, 11 000 + 5 000 = 16 000

habitantes.
iii) Os percentuais de populações urbana e rural do município I são, respectivamente, iguais a

8 000 2 4 000 1
= ≈ 66,67% e = ≈ 33,33%.
8 000 + 4 000 3 8 000 + 4 000 3

139
Os percentuais de populações urbana e rural do município II são, respectivamente, iguais a
10 000 5 18 000 1
= ≈ 36% e = ≈ 64%.
10 000 + 18 000 14 10 000 + 18 000 3
Os percentuais de populações urbana e rural do município III são, respectivamente, iguais a
11 000 11 5 000 5
= = 68,75% e = = 31,25%.
11 000 + 5 000 16 11 000 + 5 000 16
Os percentuais de populações urbana e rural do município IV são, respectivamente, iguais a
18 000 10 000
≈ 64% e ≈ 36%.
18 000 + 10 000 18 000 + 10 000
Os percentuais de populações urbana e rural do município V são, respectivamente, iguais a
17 000 17 12 000 12
= ≈ 59% e = ≈ 41%.
17 000 + 12 000 29 17 000 + 12 000 29

iv) A média da população urbana dos cinco municípios é


8 000 + 10 000 + 11 000 + 18 000 + 17 000 29 000 + 35 000 30 000 + 34 000
= = = 32 000 habitantes.
5 2 2

v) e vi) Professor, oriente os alunos no uso de planilhas eletrônicas para confeccionar esses gráficos.


Questão 149 A tabela abaixo mostra dados de 2021 sobre os percentuais da população em cada
região brasileira abaixo das seguintes linhas de pobreza, definidas pelo Banco Mundial:

• extrema pobreza: renda abaixo de 1,90 dólares por dia;


• pobreza: renda acima de 1,90 dólares e abaixo de 5,50 dólares por dia.

Rendimento/Região Rendimento
Região Extrema pobreza Pobreza População
Norte 13,0% 13,3% 8,7%
Nordeste 53,2% 44,8% 27,0%
Centro-Oeste 3,3% 5,5% 7,8%
Sudeste 29,5% 25,5% 42,1%
Sul 5,0% 6,9% 14,3%

De acordo com os dados na tabela, qual o percentual da população em extrema pobreza nas regiões
Norte e Nordeste juntas?

A) 13,0 % B) 35,7% C) 53,2% D) 58,1% E) 66,5%

Solução. Basta somar os percentuais na coluna “Extrema Pobreza” e nas linhas “Norte” e “Nor-
deste”, obtendo
13,0% + 53,2% = 66,2%.


140
Questão 150 a) De acordo com os dados da tabela na questão 149, qual região tem a maior proporção
de pessoas em condição de extrema pobreza?
b) Qual o percentual da população brasileira nessa condição?
Dica: para responder, observe que a coluna “População” na tabela indica o porcentual da população
brasileira em cada uma das regiões.

Solução. a) A população em extrema pobreza na região Norte representa

8,7% · 13% = 11,31%

da população brasileira em extrema pobreza. A população em extrema pobreza na região Nordeste


representa
27% · 53,2% ≈ 14,36%
da população brasileira em extrema pobreza. A população em extrema pobreza na região Centro-Oeste
representa
7,8% · 3,3% ≈ 0,26%
da população brasileira em extrema pobreza. A população em extrema pobreza na região Sudeste
representa
42,1% · 29,5% ≈ 12,42%
da população brasileira em extrema pobreza. A população em extrema pobreza na região Sul representa

14,3% · 5,0% ≈ 0,71%

da população brasileira em extrema pobreza. Portanto, o Nordeste é a região brasileira que têm maior
percentual de pessoas em extrema pobreza, relativamente ao total de habitantes da região.
b) Concluímos também que

11,31% + 14,36% + 0,26% + 12,42% + 0,71% ≈ 39%

da população brasileira vivia em extrema pobreza, de acordo com os dados referentes a 2021. 

Questão 151 Corrija o seguinte gráfico de barras para que corresponda aos dados expostos nas
colunas “extrema pobreza” e “pobreza” da tabela na questão 149.

Solução. O gráfico de colunas corrigido é o seguinte:

141


Questão 152 Construa gráficos de setores que representem os dados na questão 149 correspondentes
aos percentuais de extrema pobreza e pobreza nas cinco regiões.

Solução. Professor, oriente os alunos no uso de planilhas eletrônicas para confeccionar esses
gráficos. 

142

Você também pode gostar